Está en la página 1de 120

Introducción al Pensamiento Científico

UNIVERSIDAD DE BUENOS AIRES


Rector Ruben Hallu
Vicerrector Alberto Edgardo Barbieri
Secretaria de Asuntos Académicos María Catalina Nosiglia
Subsecretaría de innovación y Calidad Académica Marilina Lipsman
PROGRAMA UBA XXI Directora Claudia Lombardo
Vicedirectora Diana Mazza

Coordinación Desarrollo Pedagógico María Alejandra Codazzi


Coordinación Producción Multimedial
Liliana Castillo María Alejandra Batista Ariadna Pou Patricia Bucich Ariel Guglielmo

Coordinación Comunicación Integral Marcela Gamberini


GUÍA DE ESTUDIO
Equipo Docente
Profesora titular de IPC Cristina Ambrosini
Especialistas en contenidos Cristina Ambrosini Gastón Beraldi
Introducción al pensamiento científico : guía de estudio . - 1a ed. - Buenos Aires :
Eudeba, 2012. - (UBA XXI; 0) E-Book.

ISBN 978-950-23-2026-7
1. Epistemología. CDD 121

Eudeba
Universidad de Buenos Aires
1ª edición: agosto de 2012
© 2012, Editorial Universitaria de Buenos Aires
Sociedad de Economía Mixta
Av. Rivadavia 1571/73 (1033) Ciudad de Buenos Aires
Tel.: 4383-8025 / Fax: 4383-2202
www.eudeba.com.ar
UBA XXI es el Programa de Educación a Distancia de la Universidad de Buenos Aires,
dirigido a alumnos interesados en adoptar esta modalidad de enseñanza para cursar
algunas materias del Ciclo Básico Común. Es una propuesta multimedial que brinda
distintos materiales y recursos para organizar el estudio de modo autónomo, sin la
obligación de asistir a clases. En la página web institucional
http://www.uba.ar/academicos/uba21 se encuentra la información sobre esta
propuesta de estudio y las primeras orientaciones para incorporarse al Programa.
Además, UBA XXI cuenta con un Campus virtual,
http://www.ubaxxivirtual.uba.ar, un entorno en el cual los alumnos interactúan
con docentes y compañeros, así como también acceden a actividades, materiales
didácticos y recursos multimediales para acompañar el cursado de las asignaturas
elegidas.

Introducción al Pensamiento Científico en UBA XXI

Para abordar los contenidos de la materia IPC en esta modalidad, usted cuenta con
los siguientes materiales y recursos didácticos: - Textos impresos de lectura
obligatoria: Guía de Estudio de IPC y Bibliografía. - Un espacio en el Campus vir tual
desde el cual podrá acceder al correo electrónico para consultas sobre la materia,
actividades de aprendizaje como foros de discusión y de orientación pedagógica y a
otros recursos didácticos que lo acompañarán en su estudio. Se recomienda escuchar
los programas grabados de radio, disponibles en la Mediateca. - Tutorías presenciales
que son encuentros optativos para abordar los temas del programa, aclarar dudas
sobre los contenidos y sobre las actividades de aprendizaje. - Programas de radio
sobre diversos temas de la asignatura, que se emiten por: FM Radio UBA 90.5 - Sala
de lectura en la que se puede consultar la bibliografía obligatoria para el estudio de la
materia, en la Sede Central de Pte. J. E. Uriburu 950, 1° piso, correo electrónico:
bibliotecaubaxxi@rec.uba.ar.

La Guía de Estudio de IPC es una herramienta didáctica que acompaña la lectura de


la bibliografía obligatoria de la asignatura. El objetivo principal es guiar a los alumnos
en la comprensión de la materia. Está organizada siguiendo el desarrollo del Programa.

En cada unidad se anticipan los temas y la bibliografía, se sugieren preguntas y


actividades de aprendizaje y se incluyen los "Documentos de Cátedra", que son textos
elaborados por los profesores sobre algunos contenidos del programa.

Este material didáctico es de lectura obligatoria.

Las ilustraciones correspondientes a Alicia en el país de las maravillas han sido basadas
en los originales de Sir John Tenniel. Se agradece a Ariel F. Guglielmo por los dibujos
realizados.

Recomendamos ingresar con frecuencia al espacio que la materia tiene en el Campus


virtual para participar de los foros y otras actividades que allí se proponen.
Objetivos generales

QUE LOS ALUMNOS:


- comprendan las características generales del conocimiento científico;
- conozcan los fundamentos del lenguaje lógico y del lenguaje científico;
- comprendan las principales orientaciones de la epistemología contemporánea;
- desarrollen un lenguaje riguroso y preciso;
- desarrollen la capacidad de análisis y el pensamiento crítico;
- valoren al pensamiento y a la actividad científica como dimensiones fundamentales
de la cultura y la sociedad;
- valoren el papel y el impacto social de la ciencia y la tecnología;
- valoren el compromiso social de los científicos y del resto de los ciudadanos.

Objetivos específicos

QUE LOS ALUMNOS:


- reconozcan las estructuras del lenguaje como paso indispensable para la
comprensión de temas metodológicos;
- establezcan diferencias entre el plano formal o empírico del conocimiento científico;
- evalúen las diferentes comprensiones de las estructuras científicas: enfoque
sincrónico (estático: las teorías vistas como unas estructuras lógicas) o diacrónico
(dinámico: el proceso histórico de génesis, mejoramiento y cambio de teorías);
- comprendan los aspectos salientes de los debates contemporáneos en el terreno de
la epistemología y el impacto social de la ciencia y la tecnología en el mundo
contemporáneo; - identifiquen las diferencias teóricas y metodológicas entre las
ciencias formales y las ciencias fácticas.

Unidad 1. Consideraciones sobre el lenguaje


1.1. Lenguaje y teorías científicas
1.2. Lenguaje y realidad
1.3. Uso y mención del lenguaje
1.4. La Semiótica
1.5. Nombrar y clasificar. Vaguedad y ambigüedad
1.6. La definición

Unidad 2. Argumentación: el escenario formal y el escenario informal


2.1. Las leyes lógicas
2.2. Tautologías, contradicciones, contingencias
2.3. Los razonamientos. Verdad y validez
2.4. Argumento y consecuencia lógica. Reglas lógicas
2.5. La inducción y la analogía
2.6. ¿Es formal la inducción? El intento de Carnap
2.7. Lógica informal y falacias materiales
2.8. Toulmin, sobre los usos argumentativos

Unidad 3. Las ciencias formales


3.1. La matemática: constructos formales y realidad
3.2. Sistemas axiomáticos
3.3. Propiedades de los sistemas axiomáticos
3.4. Interpretación y modelos de los sistemas axiomáticos

Unidad 4. La cuestión del método en las ciencias fácticas


4.1. Estructura de las teorías empíricas
4.2. El problema del método científico
4.3. Estrategias metodológicas básicas de las ciencias fácticas
4.4. Método inductivo: inductivismo "estrecho" e inductivismo "sofisticado"
4.5. Método hipotético-deductivo

Unidad 5. Explicación y predicción en ciencias fácticas


5.1. ¿Qué significa "explicar"?
5.2. Concepto de explicación científica
5.3. Modelos de explicación científica
5.4. La especificidad de las ciencias sociales: intencionalidad y explicación por
mecanismos
5.5. Dimensión explicativa y dimensión predictiva
5.6. El enfoque comprensivista en ciencias sociales

Unidad 6. Corrientes epistemológicas contemporáneas


6.1. La epistemología del siglo XX: problemas fundamentales
6.2. La concepción epistemológica del Positivismo lógico
6.3. Las perspectivas falsacionistas: Popper y Lakatos
6.4. La ruptura: del racionalismo de justificación al análisis de la historia de la ciencia
6.5. El pensamiento de Kuhn en perspectiva

Unidad 7. Ciencia y sociedad


7.1. Política científica
7.1.1. Características generales
7.1.2. La planificación de la política científica. Debates contemporáneos
7.1.3. La formación del investigador
7.1.4. Etapas de la política científica argentina
7.1.5. Institutos de investigación y Universidad
7.2. Ciencia, tecnología y sociedad
7.2.1. Ciencia, tecnología y ética: la responsabilidad social del científico y el tecnólogo
7.2.2. Ciencia y ética: debates en torno a la neutralidad valorativa de la ciencia
7.2.3. La ética de la investigación científica
7.2.4. Ciencia básica, ciencia aplicada y tecnología. Cientificismo y anticientificismo
Unidad 1. Consideraciones sobre el lenguaje
BIBLIOGRAFÍA OBLIGATORIA: Unidad 1 en la Guía de Estudio de IPC, producida por
UBA XXI y editada por Eudeba, a partir de la edición de 2010. Asti Vera, C. y
Ambrosini, C. (2009). Capítulo 1. En Argumentos y teorías. Aproximación a la
Epistemología. Buenos Aires: Educando.
BIBLIOGRAFÍA COMPLEMENTARIA: Copi, I. (1984). Capítulos 2 y 4. En Introducción a
la lógica. Buenos Aires: Eudeba.
Unidad 2. Argumentación: el escenario formal y el escenario informal
BIBLIOGRAFÍA OBLIGATORIA: Unidad 2 en la Guía de Estudio de IPC, producida por
UBA XXI y editada por Eudeba, a partir de la edición de 2010. Asti Vera, C. y
Ambrosini, C. (2009). Capítulos 2 y 3. En Argumentos y teorías. Aproximación a la
Epistemología. Buenos Aires: Educando.
BIBLIOGRAFÍA COMPLEMENTARIA: Copi, I. (1984). Capítulos 1 y 3. En Introducción a
la lógica. Buenos Aires: Eudeba. Díez, J. A. y Moulines, C. U. (1999). Capítulo 2. En
Fundamentos de filosofía de la ciencia. Barcelona: Ariel. Bunge, M. (1997). Capítulo 3.
En Epistemología. México: Siglo XXI. Klimovsky, G. (1994). Capítulo 18. En Las
desventuras del conocimiento científico. Buenos Aires: AZ.
Unidad 3. Las ciencias formales
BIBLIOGRAFÍA OBLIGATORIA: Unidad 3 en la Guía de Estudio de IPC, producida por
UBA XXI y editada por Eudeba, a partir de la edición de 2010. Asti Vera, C. y
Ambrosini, C. (2009). Capítulo 4. En Argumentos y teorías. Aproximación a la
Epistemología. Buenos Aires: Educando.
BIBLIOGRAFÍA COMPLEMENTARIA: Díez, J. A. y Moulines, C. U. (1999). Capítulos 4 y
5. En Fundamentos de filosofía de la ciencia. Barcelona: Ariel. Klimovsky, G. (1994).
Capítulos 2, 3, 4 y 5. En Las desventuras del conocimiento científico. Buenos Aires: AZ.
Unidad 4. La cuestión del método en las ciencias fácticas
BIBLIOGRAFÍA OBLIGATORIA: Unidad 4 en la Guía de Estudio de IPC, producida por
UBA XXI y editada por Eudeba, a partir de la edición de 2010. Asti Vera, C. y
Ambrosini, C. (2009). Capítulo 5. En Argumentos y teorías. Aproximación a la
Epistemología. Buenos Aires: Educando.
BIBLIOGRAFÍA OBLIGATORIA: Cohen, I. y Nagel, E. (1980). Capítulos X, XI, XIV y XVI.
En introducción a la lógica y al método científico (vol. II). Buenos Aires: Amorrortu.
Hempel, C. G. (1985). Capítulos 2, 3 y 4. En Filosofía de la ciencia natural. Madrid:
Alianza. Klimovsky, G. (1994). Capítulos 9, 13 y 14. En Las desventuras del
conocimiento científico. Buenos Aires: AZ.
Unidad 5. Explicación y predicción en ciencias fácticas
BIBLIOGRAFÍA OBLIGATORIA: Unidad 5 en la Guía de Estudio de IPC, producida por
UBA XXI y editada por Eudeba, a partir de la edición de 2010. Asti Vera, C. y
Ambrosini, C. (2009). Capítulo 6. En Argumentos y teorías. Aproximación a la
Epistemología. Buenos Aires: Educando. Beraldi, G. (2010). Documento de Cátedra: La
tensión entre explicación y comprensión. El problema de la explicación en las ciencias
sociales. En esta Guía de Estudio de IPC, Buenos Aires: Eudeba; y en el Campus virtual
de UBA XXI. BIBLIOGRAFÍA COMPLEMENTARIA: Díez, J. A y Moulines, C. U. (1999).
Capítulo 7. En Fundamentos de filosofía de la ciencia. Barcelona: Ariel. Schuster, G.
(1986). Capítulos 2, 3, 4 y 5. En Explicación y predicción. Buenos Aires: Clacso. Wright,
G. H. von (1979). Explicación y comprensión. Madrid: Alianza.
Unidad 6. Corrientes epistemológicas contemporáneas
BIBLIOGRAFÍA OBLIGATORIA: Unidad 6 en la Guía de Estudio de IPC, producida por
UBA XXI y editada por Eudeba, a partir de la edición de 2010. Asti Vera, C. y
Ambrosini, C. (2009). Capítulo 7. En Argumentos y teorías. Aproximación a la
Epistemología. Buenos Aires: Educando.
BIBLIOGRAFÍA COMPLEMENTARIA: Echeverría, J. (1999). Capítulos 1, 2, 3, 4, 5 y 6.
En introducción a la metodología de la ciencia. La filosofía de la Ciencia en el siglo XX.
Madrid: Cátedra. Klimovsky, G. (1994). Capítulos 21, 22 y 23. En Las desventuras del
conocimiento científico. Buenos Aires: AZ.
Unidad 7. Ciencia y sociedad
BIBLIOGRAFÍA OBLIGATORIA: Unidad 7 en la Guía de Estudio de IPC, producida por
UBA XXI y editada por Eudeba, a partir de la edición de 2010. Contratti, M. B. (2010).
Documento de Cátedra: Política científica: problemas y perspectivas. En esta Guía de
Estudio de IPC, Buenos Aires: Eudeba; y en el Campus virtual de UBA XXI. Contratti,
M. B. (2010). Documento de Cátedra: Ética y ciencia. En esta Guía de Estudio de IPC,
Buenos Aires: Eudeba; y en el Campus virtual de UBA XXI.
A través del curso a distancia de esta materia, introducción al Pensamiento Científico,
se busca poner al alcance de los estudiantes de la Universidad el conocimiento de la
epistemología, aquella región de la filosofía que problematiza los métodos y prácticas
de la ciencia así como evalúa sus resultados. Esta propuesta está animada por la idea
de establecer una estrecha relación entre "construcción de la ciudadanía" y "enseñanza
de epistemología". ¿Por qué, desde la perspectiva que sostenemos en esta asignatura,
enseñar epistemología para posibilitar la constitución de la ciudadanía plena? Porque,
por un lado, el aprendizaje de la epistemología permite, entre otras cuestiones,
distinguir formas de argumentación racional de otras que solamente parecen serlo, y
analizar y debatir en torno de los criterios que permiten esa distinción. Por otro lado,
favorece el perfeccionamiento de habilidades de pensamiento necesarias para
interactuar en la vida social. Mas para que esta posibilidad se concrete, es
imprescindible el acceso a determinados bienes simbólicos: el uso adecuado del
lenguaje; la competencia discursiva en diversas esferas de la vida social; una
disponibilidad de información acorde, no sólo con las demandas profesionales, sino que
posibilite participar de distintas interacciones en las que las personas se constituyen
como ciudadanos porque están en condiciones de tomar la palabra y sostenerla,
evitando así ser víctimas de manipulaciones. Desarrollar las habilidades para pensar y
actuar con flexibilidad a partir de lo que ya se sabe es apenas el comienzo de esta
tarea. Implementar estrategias para la construcción del conocimiento implica
establecer un puente entre el producto y el productor, porque un conocimiento sólo es
un producto acabado cuando el alumno logra pensar en él como un producto propio.
La generación de este conocimiento autocrítico y consciente, que conforma un
instrumento importante para la transformación de la realidad, resulta prioritaria. En la
actualidad observamos dos tipos de situaciones: el prejuicio contra la teoría, que a
menudo es presentada ante la opinión pública como un inútil pasatiempo para
intelectuales, y una cultura ágrafa, basada en estímulos intensos y pasajeros (al estilo
de los videoclips), en la que la capacidad crítica es cuidadosamente desactivada para
ponerla al servicio de algunos prejuicios, sintetizados en unas pocas frases hechas.
Arriesgarse a tomar la palabra es un gesto propio de una ciudadanía activa; estar
impedido de hacerlo revela al lenguaje como barrera social. Por ello creemos que, en el
caso de esta propuesta, es esencial armonizar la enseñanza de habilidades de
pensamiento y contenidos, donde podemos apreciar la centralidad de un uso reflexivo
del lenguaje para propiciar, a su vez, la conformación de un conocimiento pluralista,
capaz de estimular en los estudiantes, desde el inicio de su formación, el desarrollo de
un pensamiento crítico y autónomo. Creemos firmemente que la enseñaza de esta
materia, de raigambre filosófica, no es simple transmisión de conocimientos (en este
caso de lógica, nociones de metodología e historia de la ciencia), sino que es
producción y, en tanto producción filosófica, es producción autocrítica y reflexiva de
conocimientos. Actualmente se menciona la "sociedad del conocimiento" para aludir al
tipo de sociedades donde el conocimiento es un factor principal en la producción de
riquezas, de allí que sea cada vez más alta la demanda social de educación superior en
sociedades como la nuestra. Esta propuesta didáctica, entonces, busca introducir a los
alumnos en la revisión de las estructuras del lenguaje y los distintos escenarios
argumentativos para que, con estas herramientas, puedan luego evaluar las distintas
posiciones epistemológicas y el mundo de debates que se genera alrededor de estas
particulares y poderosas manifestaciones de la creatividad y de la inteligencia humana
que llamamos "ciencias".
¿Por qué Introducción al Pensamiento Científico en el inicio de los estudios
superiores?

En este curso se trata de tomar en cuenta las particularidades del conocimiento


científico tal como los problematiza la epistemología. Para ello, y antes de iniciar el
estudio de esta disciplina, conviene recordar que nuestro concepto de ciencia es un
producto histórico de ascendencia griega. Los babilonios, los egipcios, los aztecas y los
incas reunieron y registraron mucha información sobre los fenómenos naturales, con
estos conocimientos pudieron alcanzar grandes logros culturales y, sin embargo, es
discutible si este conocimiento puede considerarse "científico". Por otra parte, podemos
admitir que la ciencia es un producto social bastante reciente si lo comparamos con
otros logros culturales más antiguos como el arte o la literatura. Dentro de esta
tradición, para que haya ciencia, es necesario que estos conocimientos formen un
sistema ordenado según criterios lógicos. En la conformación de la idea de ciencia, el
epistemólogo argentino Juan Samaja[1] reconoce dos momentos: a) Un primer
momento que correspondería al nacimiento de la ciencia en el sentido más amplio de
la episteme tal como la concibe Platón (427-347 a.C.), como un conocimiento acorde a
la razón o logos, como un poder que permite controlar sus fundamentos. De allí el uso
del término logos para designar disciplinas científicas como antropología, psicología,
etc. En este período se puede ubicar el pasaje de la concepción mitológica del mundo a
la concepción epistemológica y los héroes de esta conquista fueron los filósofos griegos
Tales de Mileto (639-547 a.C.), Parménides de Elea (nacido entre el 530 y el 515 a.C.),
Pitágoras de Samos (aproximadamente 582-507 a.C.) y, especialmente, Aristóteles
(384-322 a.C.), entre otros. b) Un segundo momento que corresponde al nacimiento
de las ciencias positivas según se concibe a partir del desarrollo de las ciencias
experimentales con Galileo Galilei (1564-1642), Isaac Newton (1643-1727), Blaise
Pascal (1623-1662), René Descartes (1596-1650), para nombrar sólo a los primeros.
Desde este punto de vista, el conocimiento científico es el resultado de una práctica
que consiste básicamente en "teorizar" acerca de distintas entidades, empíricas o
formales. "Argumentar" y "teorizar" no son sinónimos, sin embargo con estos términos
señalamos dos actividades íntimamente conectadas ya que la construcción de una
teoría supone utilizar argumentos que la fundamenten o justifiquen. Realizar una
actividad no implica necesariamente estar capacitado para formular las reglas de estas
prácticas ni para criticar algunas de las creencias vigentes y menos para conocer su
historia y las distintas interpretaciones que puedan hacerse de ellas. Está claro que se
puede ser un excelente director de cine y un pésimo crítico de arte o un destacado
jugador de tenis y un mal comentarista deportivo. Salvando las distancias, de un modo
parecido teorizar, como hablar o argumentar, es una actividad que puede realizarse
correctamente sin realizar explícitamente una revisión filosófica de sus presupuestos.
Los científicos y los estudiantes de las distintas ciencias, a menudo, suelen mirar con
desconfianza a los epistemólogos y esta desconfianza en parte se justifica por lo
abstracto de sus formulaciones que, a veces, parecen conducir a discusiones bizantinas
completamente alejadas de las prácticas científicas concretas y de los acuciantes
dilemas morales a los que se enfrentan. Por otra parte, algunos epistemólogos
sostienen que para hacer epistemología es necesario ser científico o estar familiarizado
con la producción de teorías en el interior de alguna disciplina, mientras que otros
defienden la autonomía y especificidad del discurso filosófico y el científico. Volviendo a
la analogía anterior, equivale a la discusión acerca de si, para ser crítico literario, es
necesario haber escrito una novela o para discutir sobre cine, ser cineasta. Más allá de
estas polémicas, podemos admitir que es deseable que un buen científico sea capaz de
poder reflexionar críticamente acerca de su objeto de estudio y de evaluar los
resultados sociales de su práctica científica. La epistemología se ocupa de formular
teorías filosóficas sobre teorías científicas buscando así evitar la aceptación dogmática
y acrítica de los logros de las distintas ciencias. Desde esta concepción, renunciamos a
asentar una única definición de "ciencia". Por el contrario, de lo que se tratará es de
presentar distintas posiciones epistemológicas que en el desarrollo del conocimiento
occidental dieron lugar a distintas concepciones de las ciencias. Un curso de
epistemología en el inicio de la formación universitaria de los futuros científicos o
profesionales involucrados en el tratamiento de teorías científicas tiene el sentido de
hacer explícitos los problemas que afectan a la producción de teorías y a las
particularidades de esta actividad de tan alto y controvertido impacto social. A menudo
se asocia el conocimiento científico al progreso de la humanidad y al logro de
desarrollos tecnológicos que permiten aumentar nuestra calidad de vida, como son la
elaboración de vacunas o drogas para paliar enfermedades antes incurables, pero
también es cierto que puede presentarse a la ciencia como responsable de la
contaminación del planeta y de las distintas amenazas bélicas. En este curso no
pretendemos tomar partido por ninguna concepción pesimista u optimista acerca de la
ciencia y sus promesas o amenazas sino, por el contrario, ya antes de adoptar una
posición, esperamos que el alumno refuerce las competencias argumentativas para
fundamentar alguna. En nuestro tiempo, constatamos que el viejo ideal griego del
conocimiento del mundo como parte de una empresa desinteresada no da cuenta del
concepto actual de ciencia, ya que de ella se espera alguna "utilidad" para ordenar los
fenómenos de un modo "eficaz". Desde este punto de vista, la aceptabilidad de las
teorías se sustenta en la obtención de éxitos tecnológicos y la verdad científica queda
asimilada a la utilidad. La ciencia adquiere el alto grado de prestigio a partir de su
utilidad social, en especial a partir de su interdependencia con el sistema productivo.
Desde la concepción positivista (que recorreremos en varios capítulos de este curso) se
concibe a la ciencia como un producto de la razón, que suministra el conocimiento
necesario para dominar la naturaleza y controlar el orden social. Según esta idea, la
ciencia es la principal herramienta del progreso humano. No se trata de negar este
supuesto, sino de constatar su presencia y llevarlo al campo reflexivo. Básicamente, en
nuestras concepciones acerca de la ciencia creemos que el conocimiento científico no
nos fue otorgado como un don de los dioses, sino que es el resultado de las
actividades humanas que, al igual que otras adquisiciones, son productos falibles y
perfectibles, que en ningún caso sus resultados son inocuos, por el contrario, que
deben ser cuestionados y revisados tanto por la comunidad científica como por el resto
de la comunidad. Esperamos que este curso brinde herramientas conceptuales a las
nuevas generaciones de científicos y profesionales en esta tarea central para la
construcción de una ciudadanía activa.

Un curso, una aventura llevados de la mano de Alicia

Recurrir a la figura de Alicia en el país de las maravillas[2] en un curso de epistemología


no es una originalidad. Por el contrario, los personajes presentes en el cuento son
íconos de nuestra cultura y su autor es objeto de culto en el mundo de los filósofos y
científicos desde la aparición del cuento, en Inglaterra, en 1864. Podemos advertir que
en esa época había una fuerte revisión de los conceptos centrales de la física y la
biología y que esta obra es contemporánea a la publicación de El origen de las
especies por medio de la selección natural, o la preservación de las razas preferidas en
la lucha por la vida, publicada en 1859 por otro científico revolucionario inglés, Charles
Darwin. Para el gran público, especialmente el infantil, el éxito editorial de Alicia fue
inmediato. Tanto en el mundo de la literatura como en el de la ciencia, distintos
autores reconocieron en el uso y la creación de nuevos lenguajes la puerta de entrada
a un experimento revolucionario. Si optamos por definir al lenguaje como un conjunto
de símbolos regidos por reglas, la lógica que usamos es la cárcel del lenguaje ya que
nos confina al uso de algunas, consagradas como las reglas "correctas". Lewis Carroll
nos abre las puertas de la celda para que salgamos a "jugar", a usar otras reglas, otras
leyes, otros principios, mostrando así todo tipo de situaciones paradójicas y
demenciales. A juicio del escritor argentino Jorge Luis Borges (1899-1986), las
aventuras de Alicia parecen arbitrarias e irresponsables, luego advertimos que
encierran "el secreto rigor del ajedrez y de la baraja", es decir, denuncian la naturaleza
convencional y arbitraria de estas reglas, lo que permite experimentar mentalmente,
imaginativamente, la posibilidad de transitar distintos órdenes. La inocente mirada de
Alicia, perpleja pero irreverente, consigue un efecto desestructurador sobre nuestras
convicciones profundas al poner en descubierto el absurdo de las situaciones que no
encuadran perfectamente en las reglas conocidas. Bajo un disfraz de locura, el relato
disimula, detrás de las actitudes candorosas de una niña, la agudeza de una
inteligencia crítica capaz de ridiculizar y mostrar en toda su insignificancia y
precariedad algunas de las categorías más respetadas por las ciencias y la lógica.
Acompañando a Alicia en los pasajes cruciales del cuento, encontramos la figura tutelar
del gato de Cheshire que, al modo de un alter ego, aparece y desaparece a voluntad
(hasta quedar solamente su sonrisa) y es el único que toma en broma todo lo que
ocurre. En este curso aparecerán varias veces alusiones a pasajes de la obra de Lewis
Carroll así como fragmentos de textos de otro admirador de Alicia, Borges. Ambos
consagraron gran parte de su genio creativo a la imaginación de situaciones donde
aparecen fuertemente cuestionadas las reglas y presupuestos del lenguaje. Conmover
las creencias profundas acerca de nuestras habilidades lingüísticas provoca perplejidad
y una cierta sensación de inseguridad, ya que con ello ponemos en tela de juicio
nuestras certezas y seguridades acerca del mundo. Revisar los mecanismos íntimos de
la construcción de teorías implica superar las concepciones intuitivas para acceder
(haciendo un esfuerzo contraintuitivo) a la comprensión de concepciones alternativas
acerca de la relación entre el lenguaje y la realidad. Poner una cuña entre estos dos
planos es uno de los logros de la epistemología contemporánea. Este pasaje es
necesario para acceder al territorio de la epistemología, especialmente para deslindar
entre el plano del lenguaje (formal) y el plano de la realidad (empírico) al momento de
diferenciar entre las ciencias formales y fácticas y advertir que, a lo largo de todo el
curso, estamos haciendo afirmaciones acerca de teorías y que el conocimiento de la
realidad está mediado por el conocimiento de teorías científicas, que no se da de un
modo inmediato ni natural y que, por el contrario, implica la "desnaturalización" de
nuestras certezas para transformarlas en hipótesis conjeturales y provisorias.
[3]
Alicia en el país de las maravillas: reseña de la obra

¿Quién escribió un libro de cuentos dedicado a su amiga predilecta, Alice Liddell?


Podemos afirmar que fue el Reverendo Charles Lutwidge Dodgson (1832-1898),
profesor de Lógica y Matemática de la Universidad de Oxford, autor de libros técnicos,
y/o podemos inclinarnos a pensar que el seudónimo "Lewis Carroll" [4] esconde a otro
personaje, no yuxtapuesto sino distinto y dividido del anterior. En este caso decidir por
la "y" o la "o" no es un detalle. Bajo el seudónimo de Lewis Carroll encontramos, en las
aventuras de Alicia, plasmadas las pesadillas de su autor, el lógico-matemático inglés
Charles Dodgson, también autor de tratados académicos de geometría y de lógica.

Las aventuras de Alicia comienzan cuando, en una soleada tarde de verano, la niña ve
pasar a un conejo -con chaleco, reloj y guantes- y movida por la curiosidad lo sigue
hasta una madriguera en la que desciende lentamente hasta caer en un pozo muy
profundo sobre un montón de palos y hojas secas. Carroll, como matemático,
manifestaba enorme interés por las teorías de la gravitación universal, que estaban
siendo objeto de gran desarrollo en su tiempo. Se ha señalado la similitud entre la
caída de Alicia en la madriguera y un ejemplo empleado por Einstein para ilustrar
algunos aspectos de su teoría. Podemos decir que las aventuras se inician a partir de lo
que podríamos denominar un "resorte moral", un impulso que sobrepasa la capacidad
reflexiva y voluntaria del personaje. Alicia se ve superada por la rigidez mecánica de la
gravedad que la coloca al margen de una acción deliberada y libre puesto que queda
reducida a la condición de "cosa que cae", aunque esta caída parece responder a otras
leyes distintas a las conocidas por la física de la época ya que cae lentamente en una
situación que llamaríamos "antigravitatoria", hasta llegar al fondo de la madriguera.

Alicia comienza, entonces, una serie de metamorfosis a partir de lo que bebe o come:
se achica hasta casi desaparecer o se agranda y pierde de vista sus propios pies.
Tantos cambios imprevistos provocan el llanto de la niña, la que comienza a dudar de
si seguirá siendo la que era antes de caer en la madriguera. La angustia de Alicia se
produce frente a la posibilidad de la pérdida de identidad o de unidad ante los
sorpresivos cambios, que ella identifica con la pérdida del nombre. Este tema es
retomado, en la conversación entre Alicia y la Oruga, la que le da consejos para
sobrellevar los cambios corporales radicales, ya que ella es experta en metamorfosis
profundas.

El efecto "bola de nieve", de acumulación de situaciones, presente en toda la obra, se


produce a partir de varias metamorfosis que trastocan la condición inicial y que
consigue neutralizar el asombro y la perplejidad del lector, el que se ve obligado a
dejarse llevar por una dinámica vertiginosa.

El encuentro de Alicia con la Duquesa y con el gato de Cheshire es uno de los


capítulos más conocidos de la obra. El gato de Cheshire puede ser visto como el animal
emblemático de la pequeña heroína de esta historia, ya que la acompaña desde un
trasfondo fantasmal donde aparece y desaparece; entre todos los animales es el único
capaz de enfrentar a la Reina con su sonrisa sardónica. Alicia se encuentra por
segunda vez con el sonriente gato de Cheshire posado sobre la rama de un árbol -la
primera vez lo ve en la cocina de la Duquesa- y éste le aconseja conocer al Sombrerero
loco y a la Liebre de marzo. En este episodio aparece cuestionada la idea del tiempo
(recordemos que al inicio aparece cuestionado el espacio) y se presentan curiosos
relojes que miden el año o los días del mes en lugar de las horas. Luego de una
conversación disparatada, Alicia da por terminada "la merienda de locos", en la que el
Tiempo se ha detenido a las seis en punto de la tarde, lo que condena a los personajes
a repetir eternamente la ceremonia del té sin darles tiempo siquiera a lavar las tazas.

Nuevamente, luego de esta extraña merienda, la niña se ve arrastrada por un impulso


o resorte que la lleva al Juego de Croquet de la Reina. En este capítulo, Carroll
ridiculiza los aspectos esenciales del espíritu inglés de su época y presenta a una Reina
despótica (recordemos que estaba en el trono la Reina Victoria y que su personalidad
impregnó la época dando lugar al adjetivo "victoriano" para referirse a cosas que
sucedieron en su reinado), dispuesta a cortar la cabeza de todo el mundo. En el campo
de Croquet de la Reina de Corazones, las pelotas son erizos vivos y los palos unos
incómodos flamencos que intentan escapar, los arcos los forman los soldados de la
Reina curvando sus cuerpos de naipes. Todos juegan al mismo tiempo sin esperar su
turno, peleando con los erizos que se mueven caprichosamente por todo el campo
haciendo desaparecer el resultado de la jugada. Para alegría de Alicia se hace visible la
cabeza del gato de Cheshire y, cuando terminan de aparecer las orejas, le cuenta entre
asombrada y divertida: "No están jugando limpiamente", empezó diciendo Alicia con
tono quejumbroso, "y se están peleando todo el tiempo, de forma que no hay quién
oiga nada..., y además, nadie hace demasiado caso a las reglas del juego; parece
como si no tuviera ninguna, o, en todo caso, si las hay, nadie parece que las esté
siguiendo...[...] es tan seguro que va a ganar la Reina que no vale la pena que los
demás sigan jugando."

Un juego de esta índole, sin reglas, donde se sabe de antemano quién va a ganarlo,
donde no se distingue entre la destreza y la pura casualidad, no puede ser puesto
como modelo sino como contrafigura del juego social identificado entre los ingleses
como fairplay. En los tres capítulos que siguen ("Historia de la Tortuga Artificial", "La
cuadrilla de la Langosta" y "¿Quién robó las tartas?") se multiplican los juegos de
palabras, la ridiculización de los cuentos con moraleja y el sistema escolar inglés.
Carroll explota al máximo los efectos cómicos que crea el lenguaje a través de distintos
mecanismos que, en muchos de los casos, son intraducibles. La crítica social se ve
reforzada por las caracterizaciones zoomórficas de los personajes. Estos animales
(reales o fantásticos) que hablan con la niña, cuestionan los parámetros de la
racionalidad en un territorio donde se diluye el límite entre la razón y la locura. En este
sentido puede asociarse a Carroll con la tradición iniciada por Esopo y continuada por
La Fontaine, hasta el mismo Walt Disney. Los animales que rodean al personaje son
puestos como contrafiguras, unas veces, o como prolongación o alter ego del
protagonista en otros casos. En el último episodio, Alicia se ve involucrada en un
proceso judicial disparatado que presenta algunas notas en común con el Juego de
Croquet de la Reina. La acusación recae sobre la Sota de Corazones por robar unas
tartas de la Reina. Nuevamente la ausencia de reglas preexistentes junto al arbitrario y
caprichoso ejercicio del poder por parte de la Reina posibilita la aparición del absurdo:

"En este momento, el Rey, que había estado muy ocupado escribiendo
apretadamente en su libreta de notas, exclamó: "¡Silencio!", y leyó a continuación lo
que acababa de anotar: "Regla Número Cuarenta y Dos: Todas las personas que midan
más de una milla de altura habrán de abandonar la Sala". Todos miraron a Alicia.
"¡pero si yo no mido una milla de altura!", dijo Alicia.
"¡Ciertamente que sí!", declaró el Rey.
"Casi dos millas", añadió la Reina.
"pues lo que es yo, no me marcharé en ningún caso", anunció Alicia; "además, esa
regla no vale porque se la acaba de inventar usted".
"Es la regla más antigua de todo el libro", aseguró el Rey.
"Entonces sería la primera y no la cuarenta y dos", acusó Alicia.
Las actitudes contradictorias del Rey y la Reina, junto a la sorpresa de la protagonista
frente a un nuevo cambio de tamaño, precipitan el final cuando Alicia asume una
decisión vital, transformadora, que revela la precariedad y fragilidad de las
convenciones admitidas:

"¡A callar!", vociferó la Reina poniéndose morada de rabia. "pues no me callo",


respondió Alicia.
"¡Que le corten la cabeza!", chilló la Reina con toda la fuerza de sus pulmones; pero
nadie hizo el menor movimiento.
"¿Quién les va a hacer caso?", dijo Alicia (que para entonces ya había recobrado su
estatura de todos los días). "¡Si no son más que un mazo de cartas!"

Alicia despierta del sueño y vuelve al punto de partida. El mundo del absurdo, en el
que estuvo sumergida, queda abolido. La seguridad amenazada, la tranquilidad perdida
por la irrupción de un "otro mundo" incomprensible se diluyen en una sonrisa cuando
logra restablecerse el orden conocido y todo se encarrila nuevamente en la
normalidad. Podemos ver, en el final de la obra, que el personaje recupera la condición
de persona a partir de un acto de voluntad, de rebeldía: la pesadilla termina en el
momento en que una decisión de no-sumisión, de sublevación aniquila la coacción de
un mundo donde estaba reducida a la condición de títere y le permite recuperar su
libertad, su autonomía.

Propuesta de un plan de estudio para los alumnos


La lectura del material de estudio requiere la puesta en práctica de distintas
habilidades por parte de los alumnos. Una competencia necesaria es la de deslindar los
temas centrales de aquellos que los sustentan o justifican. Es decir, es necesario
identificar dentro de la bibliografía, los conceptos y nociones que vertebran los
contenidos del curso y aquellos que son comentarios sobre otras posiciones, polémicas,
distintos criterios de clasificación, enfoques, etc. En este sentido, el abordaje del
material de estudio supone una "estrategia de lectura", una jerarquización de los
conceptos. En virtud de un mejor modo de organización para el estudio, sugerimos
que: - Tome contacto con el programa de la materia, en el que encontrará los
contenidos y la bibliografía, identificados por unidad. Lea la Introducción a la materia y
las presentaciones de cada unidad y luego la bibliografía obligatoria. - Realice las
actividades propuestas en esta guía de estudio, buscando la información que se
requiera. En cada unidad, además del libro de lectura obligatoria, se encuentra
señalada una bibliografía de consulta. Tome en cuenta que en el nivel de estudios
superiores es necesario recurrir a fuentes bibliográficas reconocidas. Internet es un
recurso muy accesible para encontrar información pero, como toda herramienta, debe
ser usada con cierta destreza, en este caso, para poder determinar aquellas fuentes
bien documentadas de otras poco autorizadas. La búsqueda de información requiere
un entrenamiento y éste puede ser el momento de adquirir esta competencia,
necesaria para continuar estudios universitarios. Cuando dude acerca de la
confiabilidad de la fuente encontrada puede consultar a los docentes en las distintas
instancias de tutorías presenciales y virtuales. - A partir de las situaciones
problemáticas, propuestas por las actividades, recurra al material de estudio y realice
esquemas, mapas conceptuales, resúmenes y todo tipo de actividades que permitan
organizar estos contenidos, unidad por unidad. En esa tónica, conviene localizar en
espacio y tiempo a los autores principales que aparezcan mencionados. Los ideales
científicos corresponden a períodos históricos determinados y se relacionan
estrechamente con ideales religiosos, estéticos, políticos y sociales. Conviene tomar en
cuenta el contexto histórico de los autores y de las concepciones epistemológicas que
sostienen para poder comprender mejor sus planteos y el mundo de ideas que les da
sentido. Para ello, se propone realizar una línea histórica que podrá ir completándose
con el orden cronológico de los autores, a medida que vaya avanzando en el curso.
Advierta que la primera vez que se los nombra aparecen entre paréntesis sus datos de
nacimiento y muerte. - Prepare con anticipación las actividades y téngalas a mano para
despejar sus dudas a través de los distintos medios de comunicación ofrecidos por UBA
XXI (correo electrónico, tutorías presenciales, foros, etc.). Recuerde que el mejor
resumen o esquema es el que realiza usted mismo y que el modo de aprender a
hacerlo es intentándolo hasta adquirir esta destreza. Si se nos permite una analogía
con la adquisición de habilidades físicas como "andar en bicicleta" o "nadar", la
adquisición de las competencias para estudiar es también cuestión de "entrenamiento".
La construcción del conocimiento es un logro social que se alcanza en el contacto con
los materiales de estudio y en el diálogo con otras personas. Realizar un curso a
distancia no es equivalente a estudiar en solitario. Las tutorías presenciales y el
campus virtual permiten el encuentro con otros alumnos, con los docentes a cargo del
dictado de la materia y con los pedagogos encargados de orientar la resolución de
problemas de distinta índole. Recomendamos enfáticamente usar estos recursos
previstos para la realización de este curso y, en la medida de lo posible, conformar una
comunidad de estudio con otros compañeros del curso. Los invitamos, además, a
escuchar los programas de radio y los recursos sonoros disponibles en la Mediateca del
Campus, en los que se abordan diversos temas de la materia.

Respecto a las Actividades de aprendizaje


Las actividades propuestas en cada unidad están orientadas a la problematización de
los contenidos del curso y no son indicativas del tipo de ejercitación que se tomará en
los parciales. Por el contrario, tienen el sentido de presentar problemáticamente
distintos "casos" para ilustrar diversos aspectos de las teorías abordadas y profundizar
en la comprensión de los temas. Lo importante, en estos ejercicios, no es "dar con la
respuesta correcta", sino pensar y poder justificar. "Justificar", en este caso, tiene el
sentido de hacer explícitos los aspectos teóricos de los distintos temas (definiciones,
reglas o criterios de clasificación vinculados a los conceptos). En ningún caso esta
aplicación será meramente mecánica ya que se tratará de interpretar estos conceptos
en base a algunos criterios y cabrá la posibilidad de que se privilegien algunos por
encima de otros. Por ejemplo: en base a una clasificación de distintas funciones del
lenguaje preguntamos en dónde ubicaría distintos casos, se debe tratar de poner en
juego esta clasificación y después dar cuenta del criterio que se adoptó en cada
elección. Aquí lo importante es constatar si se consideró el criterio de la clasificación.
Sabemos que hay una brecha entre la definición de un concepto y su aplicación a
casos individuales. Para usar un ejemplo conocido, digamos que, por estrictamente
definida que esté la definición de "gol" en un reglamento de fútbol, es bastante común
que se generen fuertes polémicas al momento de decidir si una jugada fue un gol o
no, es decir, cuál es la interpretación del reglamento. Más que acertar con la respuesta
correcta, lo que buscan estos ejercicios es problematizar los conceptos presentados en
el curso y mostrar la posibilidad de aplicación sobre casos donde, más allá de lo
anecdótico y particular de la situación presentada, pueda llegarse a una comprensión
más profunda de la teoría y no la mera asimilación de datos. Conviene recordar que los
temas presentados son de índole filosófica y motivo de disputas teóricas en las que
pueden sostenerse posiciones distintas, o abordarse los problemas desde ángulos
diferentes según distintas decisiones metodológicas.
Temas de la Unidad
1.1. Lenguaje y teorías científicas
1.2. Lenguaje y realidad
1.3. Uso y mención del lenguaje
1.4. La Semiótica
1.5. Nombrar y clasificar. Vaguedad y ambigüedad
1.6. La definición

Bibliografía obligatoria Unidad 1 en la Guía de Estudio de IPC, producida por UBA


XXI y editada por Eudeba, a partir de la edición de 2010. Asti Vera, C. y Ambrosini, C.
(2009). Capítulo 1. En Argumentos y teorías. Aproximación a la Epistemología. Buenos
Aires: Educando.
BIBLIOGRAFÍA COMPLEMENTARIA Copi, I. (1984). Capítulos 2 y 4. En Introducción a
la lógica. Buenos Aires: Eudeba.
Presentación de la Unidad

Los doce jurados estaban muy atareados escribiendo en sus pizarras.


"¿Qué están haciendo?" susurró Alicia a Grifo; "no pueden estar
nada, puesto que el juicio no ha empezado todavía ".

"Están escribiendo sus nombres ", cuchicheó Grifo, "no vaya a ser que los
olviden antes de que acabe el juicio ".
"¿Pero es que son imbéciles?", empezó a decir Alicia, con una voz muy
indignada pero se calló a tiempo al ver que el Conejo Blanco gritaba

"¡Silencio en la Sala!", y al ver que el rey se calaba las gafas y miraba


severamente de un lado para otro intentando descubrir quién era el que
estaba hablando.
(Carroll, L., Alicia en el país de las maravillas)

Por qué comenzamos este curso abordando el tema del lenguaje?

En principio, aceptamos que toda teoría científica es una construcción lingüística. El


lenguaje es el instrumento básico para la construcción de teorías ya que es impensable
una teoría "inefable", es decir, una teoría que no pudiera expresarse en algún
lenguaje. La revisión de las estructuras lingüísticas, presentes en las teorías científicas,
se hará desde dos disciplinas emparentadas: la Semiótica (estudio de los signos) y la
Lógica (estudio de las estructuras del lenguaje). Estas dos disciplinas actualmente
tienen desarrollos teóricos autónomos aunque remiten a un origen común en las
teorías griegas. Comenzamos este recorrido con la concepción mágica que identifica al
nombre con el alma o espíritu de la cosa nombrada, antes de mencionar las ideas de
Platón presentes en la obra Cratilo para caracterizar la teoría esencialista. A
continuación estudiaremos la posición revolucionaria del filósofo y teólogo franciscano
inglés Guillermo de Ockham (1298-1349), quien introduce, en el siglo XIV, una
novedosa teoría acerca del signo lingüístico, en oposición a la teoría esencialista de
Platón. La posición de Ockham, caratulada como "nominalismo", nos introduce en las
nociones de la Semiótica al proponer un enfoque rival a la concepción del lenguaje del
esencialismo utilizando una nueva teoría acerca de los signos. Advertimos en este
punto que, en el texto de la bibliografía, aparecen distintas referencias al semiólogo
italiano contemporáneo Umberto Eco (n. 1932) quien rinde homenaje a Ockham en su
novela El nombre de la rosa y problematiza allí y en otros tratados de semiología
algunos de los problemas ligados al campo semántico y pragmático del lenguaje. Es
importante comprender la diferencia entre uso y mención del lenguaje para destacar el
nivel metalingüístico de la Epistemología, ya que no advertir la distinción entre las
teorías, consideradas como entidades lingüísticas, y la realidad, da lugar a todo tipo de
confusiones y paradojas. En el terreno de la Semiótica veremos, en primer lugar, la
distinción del filósofo y científico estadounidense Charles Peirce (1839-1914) entre
signo natural, ícono y símbolo a los efectos de llegar a comprender el sentido de la
definición del lenguaje como "conjunto reglado de símbolos". En segundo lugar, nos
aproximaremos a los distintos temas que aluden a la división del estudio de los signos:
la dimensión sintáctica (acerca de las relaciones entre signos y las reglas que los
ordenan), la semántica (acerca de las relaciones entre el signo y sus significados) y la
pragmática (acerca de la relación entre el signo y sus intérpretes o usuarios). Al
estudiar la dimensión semántica, nos detendremos en la noción de "término" usado
para nombrar a una de las estructuras lógicas y con ello entramos ya en el territorio de
la Lógica (para la cual los términos son unidades de significación). En el terreno de los
términos (signos lingüísticos, nombres, símbolos) se distingue entre designación,
extensión y denotación, considerados como partes del significado. Con la dimensión
pragmática y la distinción entre las funciones del lenguaje, se introduce el concepto de
"proposición", otra de las estructuras lógicas que permite predicar el concepto de
"verdadero" o "falso" a partir de tener alguna teoría acerca de la verdad proposicional.
Para ello, tomaremos la definición de proposición del filósofo austríaco Ludwig
Wittgenstein (18891951) citado en la bibliografía obligatoria: Lo que engrana con el
concepto de verdad (como una rueda dentada) eso es una proposición [...] Y lo que es
una proposición está en un sentido determinado por las reglas de formación oracional
(de la lengua castellana, por ejemplo) y en otro sentido por el uso del signo en el
juego del lenguaje. (Asti Vera y Ambrosini, 2009. 26-27) El concepto semántico de
verdad del lógico polaco Alfred Tarski (19011983) es presentado para destacar uno de
los intentos, en la semántica contemporánea, de definir el concepto de verdad
proposicional evitando la incursión en paradojas. Además de los temas mencionados,
tendrá que estudiar las distintas operaciones lingüísticas, fundamentales en el uso de
un lenguaje, en especial en el caso de los lenguajes científicos: nombrar, clasificar,
definir. Es muy importante reconocer sus particularidades antes de entrar en el análisis
de la estructura de las teorías científicas. La clasificación de las ciencias que se
encuentra al inicio del capítulo 1 de la bibliografía, servirá de parámetro o mapa
conceptual útil para anticipar los temas que se trabajarán en las diferentes unidades
del programa. Como sostiene Borges, toda clasificación es arbitraria y conjetural. Esta
clasificación de las ciencias, entre formales y fácticas, no se la presenta con la
intención de legitimarla como la clasificación correcta, sino como la que,
metodológicamente, utilizamos en este curso para ordenar los distintos temas que se
presentan.
Actividades de aprendizaje

ACTIVIDAD 1. LENGUAJE Y REALIDAD


En el punto 1.2. del capítulo 1, se presentan dos posiciones antagónicas acerca del
modo de justificar la relación entre el lenguaje y la realidad: Platón y Guillermo de
Ockham. Estas actividades están orientadas a destacar la diferencia entre estas dos
posiciones.

1.1. La suppositio materialis en El nombre de la rosa de Umberto Eco[5]


Lea los siguientes párrafos y luego responda las preguntas que aparecen a
continuación.
En la novela histórica El nombre de la rosa, el semiólogo italiano Umberto Eco (1932)
rinde homenaje a la figura de Guillermo de Ockham a través del personaje de un
monje franciscano inglés, Guillermo de Baskerville. Para Francisco Bertelloni, [6] el tema
central del relato es el totalitarismo de la verdad que puede mover a los hombres a
matarse unos a otros. Según Bertelloni, este tema le permite a Eco vincular una trama
filosófica e histórica -las luchas doctrinales en la Edad Media- con una trama policial,
una serie de asesinatos cuyas víctimas tienen un rasgo en común: están todos
interesados en el mismo libro y mueren a causa de él.

El relato comienza una mañana de noviembre de 1327 en el norte de Italia y se


desarrolla en siete días. La historia gira alrededor de la búsqueda del autor de los
crímenes, un fanático que mata por extremo amor a la verdad pues no tolera que la
cosmovisión cristiana sea sustituida por las doctrinas de un Aristóteles recién
descubierto, distinto al apropiado por la doctrina oficial de la iglesia. Se torna peligroso
para la Teología un Aristóteles "que mira la tierra antes que el cielo" y debe ser
detenido incluso mediante el crimen. Eco propone mostrar que en la Edad Media la
filosofía no fue inofensiva, sino que tuvo efectos sociales relevantes. Eco apela al
nominalismo empirista de Guillermo de Ockham, la nueva filosofía del siglo XIV, y la
coloca como trasfondo de los diálogos de Guillermo de Baskerville y su discípulo Adso.

El trasfondo histórico se relaciona con las pretensiones del Papa Bonifacio VIII de
conseguir la plenitudo potestatis, la totalidad del poder espiritual y terrenal en la Bula
Unam Sanctam de 1302, promulgada 25 años antes del inicio de la historia contada
por Eco, donde define una teocracia papal: la iglesia como un cuerpo con una cabeza,
el Papa. La Bula concluye que los reyes y emperadores deben subordinarse al poder
espiritual para alcanzar la salvación. Los franceses consiguen que se fije la sede del
papado en Aviñón. La rebelión contra esta imposición papal se concentra en la corte
imperial de Luis de Baviera en Munich donde encuentran refugio Ockham, Marsilio de
Padua y otros franciscanos que proponen volver a la iglesia primitiva y separar la
iglesia del Estado. Al cuestionar el derecho de propiedad como parte del derecho
natural, Ockham procura desmontar la estructura jerárquica de la iglesia eliminando el
papel del clero como intermediario entre Dios y los hombres del mismo modo que, en
el plano de la lógica, había eliminado las entidades metafísicas como intermediarias
entre Dios y los individuos, siempre singulares. En la abadía, Guillermo se involucra en
una trama policial (al modo de las novelas de detectives) donde se suceden los
crímenes. Para resolver el problema de contestar ¿quién es el asesino? Eco, que partió
de Ockham, va más allá del contexto medieval y hace de Guillermo de Baskerville un
Sherlock Holmes con alusiones a Ludwig Wittgenstein y a la semiótica contemporánea
sin dejar de homenajear, en la imagen del bibliotecario español ciego, Jorge de
Burgos, a Jorge Luis Borges. Como no podía ser de otro modo, el semiólogo Eco, ahora
puesto en novelista, hace recaer la solución del enigma y la resolución del caso sobre
un detalle lingüístico, la suppositio materialis. Para ello, se trata de develar el sentido
de la frase que permi- tiría abrir una puerta de la biblioteca laberíntica y encontrar la
clave para ubicar al asesino.

La frase dice: Secretum finis Africae manus supra idolum age primum et septimum de
quatuor.[7] Según una primera interpretación, la traducción sería "el secreto del finis
Africae consiste en que la mano sobre el ídolo opera sobre el primero y el séptimo de
los cuatro". Ahora el mensaje no tiene sentido ya que no existe el séptimo entre cuatro
cosas. a. ¿Cuál podría ser el sentido del mensaje? b. ¿Dónde oprimiría para abrir la
puerta? c. ¿Cómo formularía el enigma para que fuese fácilmente comprensible?
Ayuda: La puerta está oculta tras un espejo (ídolo-imagen). Sobre la puerta finis
Africae hay una leyenda que reproduce el versículo 4.4 del Apocalipsis de San Juan.
"Super thronos viginti quatuor", y la palabra "quatuor" tiene siete letras.

1.2. La navaja de Ockham y el Dr. House


El siguiente texto presenta un episodio de la serie de televisión Dr. House. Léalo
atentamente y luego responda las preguntas que están a continuación: La serie
norteamericana de televisión Dr. House muestra a un médico misógino y adicto al
consumo de fármacos que, con una mente deductiva, resuelve los casos problemáticos
al modo de un detective o un semiólogo: interpretando signos. Como en las novelas de
Sherlock Holmes, el Dr. House interroga a los síntomas, como si fueran las pistas que
conducen hasta hallar al culpable: la causa de la enfermedad. Las analogías entre
Sherlock Holmes y el Dr. House son evidentes, incluso el Dr. Wilson representa una
versión del Dr. Watson. Al igual que Holmes, House es un solitario, se relaciona de
modo conflictivo con las demás personas y llega a ser incluso cortante y ofensivo con
sus pacientes. Lo único que le interesa es "resolver el caso" y todas sus emociones se
concentran en ese fin. En el capítulo 3 de la primera temporada de la serie, titulado
originalmente "Occam's Razor", traducido al español como "Principio de parsimonia",
se muestra el caso de Brandon, un chico de 22 años que ha sufrido un colapso y es
ingresado al hospital. En los últimos días ha estado tosiendo y tiene un sarpullido.
Ahora siente un dolor fuerte en el abdomen, náuseas, fiebre y su presión arterial es
baja. Un escáner rápido y un examen no revelan nada, así que House y su equipo
buscan alternativas. Coinciden en que ninguna enfermedad se corresponde con los
siete síntomas que presenta el paciente. La teoría de Foreman, uno de los médicos del
equipo que dirige House, es que el chico tiene una endocarditis, lo que explicaría
varios de los síntomas, excepto dos: la tos y el sarpullido. Aún así, es una posibilidad
entre un millón. House examina la lista de síntomas del paciente y sugiere que dos
enfermedades coinciden con los síntomas de Brandon, pero sigue siendo una
posibilidad entre un millón. House apuesta 50 dólares con Foreman a que si el
recuento de leucocitos del paciente aumenta, estará en lo cierto al decir que el chico
tiene una infección. Cuando el número de glóbulos blancos de Brandon disminuye, los
dos médicos se dan cuenta de que sus hipótesis eran erróneas. House tiene una
revelación y le pregunta al doctor Wilson cuál fue el primer síntoma de Brandon y sus
sospechas se confirman: fue la tos. House tiene entonces otra revelación, irrumpe en
la habitación del paciente y anuncia su diagnóstico: intoxicación por colchicina,
sustancia con la que se combate la "gota", una enfermedad que Brandon no sufre ni
toma medicación contra ella. Ahora el problema es saber: ¿cómo ingirió la colchicina?,
¿quién le administró esa sustancia?
El enigma se resuelve cuando la madre de Brandon confiesa que le administró pastillas
para la tos, sin consultar a los médicos. Primero Foreman y luego House afirman en
momentos cruciales: "Principio de parsimonia, la mejor explicación es la más simple".
a. ¿En qué sentido se alude aquí a la navaja de Ockham? b. ¿Qué enuncia, en su
versión más conocida, este principio? c. Una navaja se usa, normalmente, para cortar
o rasurar la barba. Investigue a la barba de quién alude el nombre de "navaja de
Ockham" y cuál sería la analogía que propone la posición nominalista y el hecho de
cortar las barbas.

ACTIVIDAD 2. USO Y MENCIÓN DEL LENGUAJE


Lea los siguientes versos y luego conteste:
Si, (como afirma el griego en el Cratilo)
El nombre es arquetipo de la cosa,
En las letras de "rosa" está la rosa
Y todo el Nilo en la palabra "Nilo". (Jorge Luis Borges, "El Golem")[8]
a. ¿A qué griego se refiere Borges y cómo interpreta la frase "el nombre es arquetipo
de la cosa"?
b. Tomando en cuenta la distinción entre uso y mención del lenguaje, justifique por
qué las palabras "rosa" y "Nilo" aparecen en un caso escritas con comillas y en otro sin
comillas
c. En este párrafo: ¿Borges afirma que en las letras de "rosa" está la rosa y todo el Nilo
en la palabra "Nilo"?

ACTIVIDAD 3. SEMIÓTICA. ACERCA DE LOS SIGNOS.


En esta actividad se trata de reconocer distintos tipos de signos según una clasificación
canónica, para llegar al concepto de "símbolo" que es mencionado como parte de la
definición del lenguaje, tema central en esta unidad. Que el lenguaje sea definido
como un "conjunto reglado de símbolos" implica, básicamente, que no hay ningún
vínculo natural ni necesario entre el nombre y el concepto u objeto nombrado, sino
que este vínculo es arbitrario y convencional. Dice Charles Morris: "Los signos y la
ciencia están inextricablemente conectados, habida cuenta de que la ciencia,
simultáneamente, ofrece a los hombres signos más fiables y expresa sus resultados en
sistemas de signos. La civilización humana depende de los signos y de los sistemas de
signos, y al propio tiempo la mente humana es inseparable del funcionamiento de los
signos, si es que, en verdad, la mentalidad misma no debe identificarse con ese
funcionamiento."[9] Charles Morris llama "semiosis" al proceso por el cual algo funciona
como signo. Es necesario que concurran tres factores para que algo funcione como
signo: El vehículo sígnico, la señal, fenómeno o cosa que actúa como signo (S). El
designatum, el significado del signo (D). El intérprete (I). Ejemplo: Un perro (I)
responde al sonido del silbato (S) que implica la caza de ardillas (D). Le proponemos
que en cada uno de los siguientes ejemplos identifique al signo (S), al designatum (D)
y al intérprete (I), y coloque (S), (D) ó (I) en los paréntesis, según corresponda.
Indique de qué clase de signo se trata. a. Al ver la seña del policía ( ), un conductor ( )
frena el auto ( ). b. Un peatón ( ) cruza la calle ( ) al ver el color de la luz del semáforo
( ). c. Al ver una nube de humo negro que sale de una ventana del primer piso ( ), los
vecinos de un edificio ( ) salen de sus departamentos corriendo ( ). d. La fiebre del
nene ( ) lleva al médico ( ) al diagnóstico de una hepatitis ( ).

ACTIVIDAD 4. SEMIÓTICA. ACERCA DE LOS TÉRMINOS.


Uno de los temas centrales de esta unidad es la distinción entre designación y
denotación. En el siguiente párrafo de Alicia en el país de las maravillas, Lewis Carroll
ilustra la idea de que los atributos no pueden andar solos (no tienen
denotación), es decir, la imposibilidad de concebir la subsistencia de un atributo
luego de la desaparición de la cosa.

Dale!", dijo el Gato, y esta vez se desvaneció muy paulatinamente,empezando por la


punta de la cola y terminando por la sonrisa, que permaneció flotando en el aire un
rato después de haber desaparecido todo el resto.
"¡Bueno!Muchas veces he visto a un gato sin sonrisa", pensó Alicia,
"pero ¡una sonrisa sin gato!... ¡Esto es lo más raro que he visto en toda mi
vida!"
(Carroll, L. 1990)

La posibilidad de poder definir términos sin denotación habilita, en el campo de las


ciencias formales, la posibilidad de armar sistemas en los que tengamos términos sin
denotación, es decir, sin ninguna representación en el mundo empírico (como la
sonrisa sin gato), pero que con ellos podemos construir sistemas en la geometría, en la
aritmética o en la lógica. En el capítulo segundo de la obra Symbolic Logic (1892),
Carroll escribió que entre las clases de cosas están las cosas imposibles y dio, como
ejemplo, el caso de las cosas que pesan más de una tonelada y un niño puede
levantar. Advierta aquí que el autor sustancializa el lenguaje al hablar de cosas
imposibles. Lo correcto sería decir "términos que nombran cosas imposibles".
Suponemos que lo hace para crear perplejidad ya que en el uso común del lenguaje
sustancializamos los conceptos. Estos términos que nombran cosas imposibles serían
los términos sin denotación de las ciencias formales (números, triángulos, líneas, etc.),
entre otros casos. Hemos tomando como ejemplo el caso imposible de una "sonrisa sin
gato". Le pedimos, ahora, que: a. Mencione al menos tres ejemplos de términos que
nombren atributos que no tienen denotación. Justifique. b. Proponga otros tres
ejemplos de términos que tengan designación en las ciencias formales y justifique por
qué puede afirmarse que no tienen denotación.

ACTIVIDAD 5. NOMBRAR Y CLASIFICAR. VAGUEDAD Y AMBIGÜEDAD.


La vaguedad (significado impreciso de un término) y ambigüedad (más de un
significado para el mismo término) son algunos de los temas tratados por la dimensión
semántica de la semiótica. Luego de leer el siguiente párrafo del libro Alicia en el país
de las maravillas, determine si el término "pican" es un caso de vaguedad o
ambigüedad. Seguro que estarás pensando que por qué no te paso el brazo por la
cintura", dijo la Duquesa tras una pausa; "la razón es que tengo mis dudas sobre el
humor de ese pájaro flamenco que llevas ahí. ¿Qué te parece si probamos el
experimento?"

'A lo mejor le da un picotazo", replicó Alicia con cautela, sin ninguna gana de
intentar el experimento.

"Muy cierto ", concedió la duquesa; "los flamencos y la mostaza, ambos pican;
y la moraleja de esto es.... Dios los cría y ellos se juntan'".

(Carroll, L. 1990)
ACTIVIDAD 6. LA DEFINICIÓN
En esta actividad se busca llamar la atención y trabajar las nociones de "definiendum"
y "definiens" para advertir que un mismo término puede ser parte de un definiendum o
de un definiens. Elabore definiciones para los siguientes términos, haciendo
corresponder al definiendum un género y una diferencia apropiados.

Definiendum Definiens
Género Diferencia
a.- soltero
b.- banquete
c.- muchacho
d.- hermano
e.- niño
f.- potro
g.- hija
h.- oveja
i.- padre
j.- gigante
k.- muchacha
l.- marido
ll.- cordero
m.- yegua
n.- enanito
ñ.- madre
o.- pony
p.- carnero
q.- hermana
r.- porción
s.- hijo
t.- soltero
a.- vástago
b.- caballo
c.- hombre
d.- comida
e.- progenitor
f.- oveja
g.- hermano
h.- mujer

a.- hembra
b.- macho
c.- casado
d.- no casado
e.- muy grande
f.- muy pequeño
g.- joven

ACTIVIDAD 7. DEFINICIÓN. DESIGNACIÓN Y EXTENSIÓN.


Entre designación y extensión hay una relación inversa: a mayor designación (notas
definitorias), menor extensión (cantidad de ejemplares). "Hombre" tiene más
designación que "animal" pero hay menos ejemplares de hombres que de animales.
Dicho de otro modo: la especie tiene mayor designación que el género y menor
extensión.
Le proponemos que ordene los siguientes términos según la mayor cantidad de
ejemplares (extensión):
a. Argentino, hombre, americano, misionero.
b. Físico, hombre, científico, ser vivo.
c. Hombre, futbolista, deportista, Martín Palermo.
Temas de la Unidad
2.1. Las leyes lógicas
2.2. Tautologías, contradicciones, contingencias
2.3. Los razonamientos. Verdad y validez
2.4. Argumento y consecuencia lógica. Reglas lógicas
2.5. La inducción y la analogía
2.6. ¿Es formal la inducción? El intento de Carnap
2.7. Lógica informal y falacias materiales
2.8. Toulmin, sobre los usos argumentativos

Bibliografía obligatoria

Unidad 2 en la Guía de Estudio de IPC, producida por UBA XXI y editada por Eudeba, a
partir de la edición de 2010. Asti Vera, C. y Ambrosini, C. (2009). Capítulos 2 y 3. En
Argumentos y teorías. Aproximación a la Epistemología. Buenos Aires: Educando.
BIBLIOGRAFÍA COMPLEMENTARIA
Copi, I. (1984). Capítulos 1 y 3. En Introducción a la lógica. Buenos Aires: Eudeba.
Díez, J. A. y Moulines, C. U. (1999). Capítulo 2. En Fundamentos de filosofía de la
ciencia. Barcelona: Ariel. Bunge, M. (1997). Capítulo 3. En Epistemología. México: Siglo
XXI. Klimovsky, G. (1994). Capítulo 18. En Las desventuras del conocimientocientífico.
Buenos Aires: AZ.
Presentación de la Unidad

"Pero es que a mí me gusta estar entre locos", observó Alicia.


"Eso sí que no lo puedes evitar", repuso el Gato; "todos estamos locos por
aquí. Yo estoy loco; tú también lo estás".
"Y ¿cómo sabes tú si yo estoy loca?", le preguntó Alicia.
"Has de estarlo a la fuerza", le contestó el Gato; "de lo contrario no habrías
venido aquí".
Alicia pensó que eso no probaba nada; pero continuó de todas formas: "Y
¿cómo sabes que tú estás loco?"
"Para empezar", repuso el Gato, "los perros no están locos, ¿de acuerdo?''
"Supongo que no", dijo Alicia.
"Bueno, pues entonces", continuó diciendo el Gato, "verás que los perros
gruñen cuando algo no les gusta, y mueven la cola cuando están contentos. En
cambio, yo gruño cuando estoy contento y muevo la cola cuando me enojo;
luego estoy loco."

(Carroll, L. Alicia en el país de las maravillas)

Esta unidad está reservada a una revisión de la tercera estructura lógica: el


"razonamiento". En la primera unidad ya hemos visto la primera estructura lógica: los
"términos" y la segunda: las "proposiciones". Aquí estudiaremos el escenario formal y
el escenario informal. En el primero, encontraremos los principios lógicos como paso
necesario para introducir los conceptos de tautologías, contradicciones y contingencias
que aluden a tipos distintos de proposiciones. Esta distinción entre proposiciones
apunta a deslindar firmemente entre ciencias formales y fácticas dado el distinto tipo
de enunciados que involucran sus teorías. Es importante reconocer la pertinencia del
uso de los conceptos de verdad y validez que también apuntan a mostrar una
distinción entre proposiciones y razonamientos.
Es muy importante reconocer las reglas lógicas y el concepto de razonamiento
deductivo. A continuación, estudiaremos la propuesta del filósofo alemán Rudolf
Carnap (1891-1970) y su esfuerzo por dotar a la inducción (una forma de
razonamiento inválido desde el punto de vista deductivo) de la rigurosidad propia de
los procedimientos formales para establecer una "lógica inductiva". También veremos
el razonamiento analógico como un tipo de razonamiento no deductivo. Este tipo de
razonamiento es un recurso argumentativo muy usado en el campo de la ciencia. En el
escenario informal veremos diferentes manifestaciones de lo que se llama "lógica
informal" o "teorías de la argumentación". Para ello estudiaremos, en principio,
distintos tipos de falacias materiales, donde se distingue entre dos grupos con
características particulares: a) falacias de inatinencia (o de inatingencia) y b) falacias
de ambigüedad. Es importante la revisión de este tipo de argumentaciones falaces ya
que es muy provechoso reconocerlas en el lenguaje social, para no ser víctimas de
manipulaciones retóricas como así también para detectarlas en las argumentaciones
científicas. Esta unidad se completa con uno de los aportes relevantes en el terreno de
las teorías de la argumentación, surgidas a partir de la segunda mitad del siglo XX: la
teoría de los usos argumentativos del filósofo estadounidense Stephen Toulmin
(1922-2009). Aquí es importante que comprenda la distancia que toma Toulmin del
planteo de la Lógica formal cuando parte de una analogía jurídica: los argumentos son
comparables a las demandas judiciales y esta lógica que apunta a la "práctica"
argumentativa sería una suerte de "jurisprudencia generalizada". El esquema
argumentativo básico distingue entre D (datos), C (conclusiones) y G (garantías). En el
ejemplo del libro de la bibliografía, se muestra el esquema según el cual la afirmación
"Juan Carlos S. nació en Salta, que es una provincia argentina" (D), permite inferir la
conclusión (C) "Juan Carlos S. es ciudadano argentino", a partir de la garantía (G) "Si
una persona nace en una provincia de la Argentina, es argentina". En un esquema
posterior, se agregan elementos al análisis del argumento con los conceptos de
"modalizador" (M), de la fuerza de la conclusión (presuntamente, probablemente) y las
condiciones de excepción o refutación (E) donde pueden presentarse casos de
excepción ("a menos que haya sido naturalizado español"). Una noción central que
debe ser tomada en cuenta para comprender la propuesta de Toulmin es la de "campo
argumentativo" ya que esta noción permite deslindar entre "argumentos analíticos",
llamados también "teóricos", que serían independientes del contexto y asimilables a los
razonamientos deductivos de la lógica formal, y los "argumentos sustanciales", también
llamados "prácticos", que serían dependientes del contexto y de importante aplicación
en el mundo práctico. Debemos advertir que la presentación del escenario informal se
completa, en el texto de la bibliografía, con otras dos propuestas destacadas como son
la del filósofo del derecho belga Chaím Perelman (1912-1984), y la de los pensadores
contemporáneos holandeses Frans Van Eemeren y Rob Grootendorst que en este
programa no se incluyen como lectura obligatoria ni forman parte del material de
lectura obligatoria pero que, de todos modos, y más allá de las exigencias temáticas
del curso, conviene leer y confrontar con la propuesta de Toulmin para completar la
comprensión de las diferencias entre el escenario formal y el escenario informal.

Actividades de aprendizaje

ACTIVIDAD 1. ACERCA DE LA ESTRUCTURA DE LOS RAZONAMIENTOS


El razonamiento es la estructura lógica que se compone de premisas y conclusión en
las que, dada una serie de enunciados que actúan como premisas, se infiere una
conclusión. Los razonamientos pueden ser "válidos" o "inválidos". En el libro Alicia en el
país de las maravillas, el gato de Cheshire irrumpe en las escenas de modo inesperado.
En el párrafo citado en la presentación de esta unidad, que aquí transcribimos, se
argumenta acerca de la inevitabilidad de la locura en el país donde se encuentra Alicia
y para ello recurre a un extraño razonamiento.

A. Lea atentamente el razonamiento.


B. Identifique las premisas y la conclusión.
C. Determine si este razonamiento es válido o inválido y justifique.

"Pero es que a mí no me gusta estar entre locos", observó Alicia.


"Eso sí que no lo puedes evitar", repuso el Gato; "todos estamos locos por aquí. Yo
estoy loco; tú también lo estás".
"Y ¿cómo sabes tú si yo estoy loca?", le preguntó Alicia.
"Has de estarlo a la fuerza", le contestó el Gato; "de lo contrario no habrías venido
aquí".
Alicia pensó que eso no probaba nada; pero continuó de todas formas: "Y ¿cómo
sabes que tú estás loco?"
"Para empezar", repuso el Gato, "los perros no están locos, ¿de acuerdo?"
"Supongo que no", dijo Alicia.
"Bueno, pues entonces", continuó diciendo el Gato, "verás que los perros gruñen
cuando algo no les gusta, y mueven la cola cuando están contentos. En cambio, yo
gruño cuando estoy contento y muevo la cola cuando me enojo; luego estoy loco. "

(Carroll, L., 1990)

ACTIVIDAD 2. ACERCA DE LOS CONCEPTOS DE VERDAD Y VALIDEZ


Como afirmamos en la bibliografía de esta unidad, predicamos la verdad o falsedad de
las proposiciones y la validez o invalidez de los razonamientos. A menudo, en el
lenguaje común, usamos estos conceptos de un modo impreciso y mezclado. Es
necesario reconocer la pertinencia del uso de estos dos conceptos para reconocer dos
estructuras lógicas distintas: las proposiciones y los razonamientos. Complete las
siguientes expresiones de modo que se conviertan en enunciados verdaderos:

a. Si un enunciado tiene premisas falsas y conclusión verdadera, el razonamiento


puede ser ...............................................................................

b. Si un razonamiento es válido y tiene premisas falsas, su conclusión puede ser


..................................................................................................
c. Si un razonamiento tiene premisas verdaderas y conclusión verdadera, su forma
puede ser ................................................................................
d. Si un razonamiento tiene premisas falsas y conclusión falsa, su forma puede ser
.........................................................................................
e. Si un razonamiento tiene premisas verdaderas y conclusión falsa, su forma es
...................................................................................................
f. Para obtener una conclusión verdadera se requiere que el razonamiento
sea....................................... y las premisas......................................

ACTIVIDAD 3. REGLAS LÓGICAS Y FALACIAS


3.1. En los cuatro razonamientos siguientes, determine qué formas son deductivas
(poner el nombre a la regla) y cuáles son falacias.

a. Si sumerjo un cubito de hielo en un vaso de agua caliente, entonces el cubito se


derrite. Sumergí el cubito, por lo tanto, se derritió.
b. Si sumerjo un cubito de hielo en un vaso de agua caliente, entonces el cubito se
derrite. No lo sumergí, por lo tanto, no se derrite.
c. Si sumerjo un cubito de hielo en un vaso de agua caliente, entonces el cubito se
derrite. El cubito no se derrite, por lo tanto, no lo sumergí.
d. Si sumerjo un cubito de hielo en un vaso de agua caliente, entonces el cubito se
derrite. El cubito se derrite, por lo tanto, lo sumergí.

3.2. En los dos razonamientos presentados a continuación hay uno deductivo y otro
inválido, a pesar de que ambos concluyen en el mismo enunciado. Identifique cada
caso y explique la diferencia entre ambos.

a. Si Juan no repite el año, entonces se va a Bariloche.


Juan se va a Bariloche. Juan no repitió el año.

b. Si Juan repite el año, entonces no va a Bariloche.


Juan fue a Bariloche (no es cierto que no va). Juan no repitió el año.
3.3. Reglas lógicas y falacias formales
Empleando el enunciado: "Si me anoto en el Ciclo Básico Común, entonces curso el
primer año de mi carrera", qué conclusión se obtiene construyendo:

a. un Modus ponens,
b. un Modus tollens,
c. una falacia de afirmación del consecuente y
d. una falacia de negación del antecedente.

3.4. Conclusión correcta de decir "Si p entonces q"


a. Si alguien promete a Juan: "Si terminás el CBC este año, te llevo de viaje a Europa"
y, luego, lo lleva a Europa a fin de año, aunque no haya terminado el CBC en un año,
¿habrá mentido? Explique por qué.
b. Si no termina el CBC en un año y no lo lleva a Europa, ¿habrá mentido? Explique por
qué.
c. Si termina el CBC en un año y no lo lleva a Europa, ¿habrá mentido? Explique por
qué.

3.5. Diferencias entre decir "si p entonces q" a decir "si y sólo si p entonces
q" ¿Qué consecuencias distintas podrían tener las siguientes afirmaciones?
a. "Si me recibo de contador este año, me caso".
b. "Si y sólo si me recibo de contador este año, me caso".

ACTIVIDAD 4. LA INDUCCIÓN
4.1. Suponiendo que las siguientes proposiciones son las conclusiones de distintos
razonamientos inductivos, en cada caso determine cuáles podrían ser las premisas.

a. "Los planetas se mueven alrededor del sol".


b. "Todas las modelos se casan con jugadores de fútbol".
c. "Todos los mamíferos tienen pelos".
d. "Todas las aves tienen plumas".
e. "Los argentinos toman mate".

4.2. Dados los siguientes enunciados, consideradas las premisas de algún


razonamiento inductivo, proponga la conclusión.

a. "Las peras, manzanas, bananas y frutillas sirven para hacer un postre".


b. "Los números 2, 4, 10 y 22 son divisibles por dos".
c. "El mejillón, las almejas, las ostras, los pulpos, los calamares y las sepias viven en el
mar".
d. "El Aedes aegypty y el Anopheles transmiten enfermedades".
e. "Las ratas, los conejos y los ratones son vivíparos".
f. "Los peruanos, los bolivianos y los coreanos venden ropa en la feria de La Salada".

4.3. Luego de leer los tres textos de Aristóteles citados, determine de qué tipo de
argumento se trata en cada caso. Decía Aristóteles:

a. "Si el mejor de los pilotos es el más diestro y el mejor de los aurigas es también el
más diestro, podemos afirmar, en general, que en cada profesión el mejor es el más
diestro". (Tópicos, I, 12)
b. "Los animales sin hiel son de larga vida; el hombre, el caballo, el mulo, son todos
animales sin hiel; por lo tanto, todos los animales sin hiel son de larga vida". (Primeros
Analíticos, II, 23)
c. "Todo ocurre según una finalidad, todo movimiento tiende a un fin, a veces ese fin
se consigue y otras veces no. En el arte, el escriba, a veces, comete faltas en la
escritura y el médico, a veces, administra equivocadamente un remedio. Así ocurre
también que la naturaleza, cuando yerra la generación, en vez de reproducir el tipo del
engendrador, se desvía de él y nace un monstruo (térata). El monstruo es un ser
inacabado, informe, mal logrado (anaperon). Cuando esto ocurre, la simiente traiciona
la forma desprendida del esperma del engendrador, que no logró dominar
perfectamente la materia suministrada por la hembra." (Física, II, 8)

ACTIVIDAD 5. LA ANALOGÍA
La analogía es un modo de razonar que, a partir de la comparación entre dos o más
casos que tienen notas comunes, permite concluir para alguno de esos casos, algo
admitido únicamente en alguno de los otros casos. Al igual que la inducción, es una
forma de razonamiento inválido aunque en muchos casos puede resultar de interés
pragmático. Dados los siguientes casos, reconstruya un razonamiento por analogía
determinando cuál es la conclusión y cuáles son las premisas.

a. "Ludwig Wittgenstein (Viena, Austria, 1889 - Cambridge, Reino Unido, 1951) solía
comparar el pensar con la natación. Así como en ésta nuestros cuerpos tienen una
tendencia natural a flotar sobre la superficie del agua, de modo que se necesita un
gran esfuerzo físico para sumergirse en el fondo, de igual modo en el pensar se
necesita un gran esfuerzo mental para alejar a nuestras mentes de lo superficial."[10]
b. "Uno de los atributos más naturales de la mujer es el cuidado de los niños. De
hecho, es correcto afirmar que los grupos en los que los hombres, y no las mujeres,
crían a los hijos pequeños son totalmente excepcionales. Puesto que el enfermo y el
discapacitado se asemejan en muchos aspectos a los niños, pues no solamente son
físicamente débiles y desvalidos, sino también psicológicamente dependientes y
narcisísticamente regresivos, era bastante fácil suponer que las mujeres están
especialmente capacitadas también para el cuidado del enfermo."[11]

ACTIVIDAD 6. LÓGICA INFORMAL Y FALACIAS MATERIALES


En el diario La Vanguardia de Barcelona, el 4 marzo de 2010 aparece la siguiente
noticia. Históricamente la comunidad de Madrid y la de Barcelona son antagonistas en
muchos sentidos. En Catalunya, el Parlament está tratando la prohibición de la corrida
de toros a partir de los reclamos de las sociedades defensoras de los derechos de los
animales, mientras que en Madrid hay una gran resistencia ya que representa un
punto de interés también turístico. Veamos el argumento de la presidenta de la
comunidad de Madrid en defensa de las corridas de toros.

A. Lea atentamente la noticia.


B. Identifique el argumento de Esperanza Aguirre.
C. Determine si incurre en alguna falacia. Justifique.

Los toros, a debate

AGUIRRE: "LOS TOROS SON UN ARTE Y MERECEN ESTAR BIEN


PROTEGIDOS"
La presidenta de la comunidad de Madrid justifica así la intención de su gobierno de
declarar los toros Bien de Interés Cultural, que coincide con el debate de prohibición
que se está haciendo en el Parlament de Catalunya El consejero de Cultura de la
Comunidad, Ignacio González, reconoce que "es evidente que tiene mucha relación"
con el debate que se está produciendo en Catalunya.
Madrid (EFE).- La presidenta de la Comunidad de Madrid, Esperanza Aguirre, ha
defendido que "los toros son un arte" y que "merece la pena declararlos Bien de
Interés Cultural para que estén bien protegidos". Aguirre ha hecho estas
declaraciones en los pasillos de la Asamblea, después de que su Gobierno haya
acordado iniciar el expediente para declarar la Fiesta de los toros Bien de Interés
Cultural (BIC). "A nadie le puede extrañar, porque la cultura taurina es algo que desde
tiempo inmemorial está en la cultura española y en la cultura mediterránea", ha
afirmado la presidenta madrileña.
A su juicio, "no hay más que ver que, desde Goya, Picasso o García Lorca o, fuera de
nuestras fronteras, Hemingway y Orson Welles, todos se han ocupado de los toros
como arte". "Qué duda cabe que es un bien cultural a proteger y, como dice nuestra
ley, no solamente los monumentos o museos deben protegerse, también los bienes
inmateriales", ha argumentado. [...]
Aguirre: "Los toros son un arte y merecen estar bien protegidos". (2010, marzo 4). La
Vanguardia. Disponible en:
http://www.lavanguardia.es/ciudadanos/noticias/20100304/
53896639502/aguirre-los-toros-son-un-arte-y-merecen-estar-bien-protegidos-madrid-e
speranza-aguirre-ignacio-gonza.html

ACTIVIDAD 7. EL CÍRCULO DE VIENA


Rudolf Carnap fue uno de los miembros destacados del Círculo de Viena (Wiener Kreis)
y director de la Revista Erkenntnis donde se publicaron los principales aportes de los
integrantes de este círculo de científicos y epistemólogos destacados. Dada la siguiente
lista de nombres de personalidades del siglo XX, investigue acerca de sus trayectorias y
luego determine qué rol cumplieron en relación con el Círculo de Viena. Moritz Schlick,
Ludwig Wittgenstein, Otto Neurath, Otto Weininger, Sigmund Freud, Hans
Reichenbach, Adolf Hitler, Kurt Gódel, Bertrand Russell, Carl Hempel, Alfred Einstein,
Alfred Tarski, Karl Popper, A. J. Ayer.

ACTIVIDAD 8. FALACIAS MATERIALES. EL ESCENARIO INFORMAL


Identifique la falacia en la que se incurre en los siguientes argumentos:
a. "Te dije el año pasado que si te casabas con tu novio, al año estarías separada.
Ahora estás separada y es por lo que te dije."
b. "Las mujeres no tienen alma. Los chinos, desde la más remota antigüedad, han
negado que la mujer tenga alma."[12]
c. "Usted no tiene derecho a tomar la palabra porque es funcionario de un gobierno
corrupto."
d. "No puede hacerme la boleta por exceso de velocidad porque, si tengo que pagar la
boleta, pierdo la ganancia del día y no tengo para darles de comer a mis hijos."
e. "Todo abogado tiene derecho a la libertad de consultar libros cuando busca
fundamentar mejor una defensa y todo médico tiene derecho a consultar sus libros
cuando quiere curar mejor a un enfermo. Del mismo modo, debe permitirse a los
alumnos ejercer la libertad de consultar sus libros para contestar mejor en un parcial."
f. "Las esposas de los hombres exitosos usan ropa cara, de modo que la mejor manera
que tiene una mujer de ayudar a su marido en los negocios es gastar mucha plata en
ropa."
g. "Señor Gerente, mi marido merece que le aumente el sueldo porque lo que gana
apenas alcanza para alimentar a los niños."
h. "En los últimos 20 años ha aumentado la asistencia de adolescentes en la
Universidad de Buenos Aires y también ha aumentado la delincuencia juvenil. Por lo
tanto, para combatir el delito, es necesario limitar el ingreso de jóvenes a la
Universidad de Buenos Aires."
i. "Está comprobado que los seres extraterrestres ayudaron a construir las pirámides
de Egipto porque hasta ahora nadie ha podido demostrar que ellos no intervinieron."
j. "Si un automóvil funciona, entonces tiene nafta en el tanque. Pero ahora el auto no
funciona, eso quiere decir que no tiene nafta en el tanque."
k. "Si Marilyn Monroe fue asesinada, entonces está muerta y, efectivamente, está
muerta. Eso quiere decir que fue asesinada."
l. "Si el amor es ciego y Dios es amor, entonces, Dios es ciego."

ACTIVIDAD 9. TOULMIN. MAPAS ARGUMENTATIVOS


Stephen Toulmin afirmó que las argumentaciones cotidianas no siguen el clásico
modelo riguroso del silogismo. Para él, en una argumentación directa, un "sujeto
argumentador" presenta explícitamente una "tesis" u opinión y expone una serie de
argumentos o "razones lógicas" que deben desembocar en una "conclusión" que
confirma la tesis propuesta.

Una exclamación como "Tienes una infección y por tanto debes tomar penicilina" es
un argumento que parte de una dato ("tienes una infección") y postula una conclusión
("debes tomar penicilina"), pero su credibilidad depende de una garantía (la penicilina
cura infecciones, por ejemplo) que a su vez se basa en un respaldo (pruebas de
laboratorio, experiencia médica, etc.), condición que hace posible el uso del
modalizador ("por lo tanto"). Y el argumento puede tener también una refutación ("a
menos que seas alérgico" podría ser una). Todos los argumentos parten de un
esquema lógico de este tipo, en el que la capacidad persuasiva depende de las
premisas en que se funda y de su capacidad de vincularse a las creencias previas del
público. Por ello, una misma conclusión puede ser defendida o refutada desde diversos
puntos de partida. El diagrama de Toulmin puede ayudarnos a esquematizar esta
actividad en el momento de organizar la comunicación. Por ejemplo, yo puedo exhortar
a alguien a adquirir un libro con distintos argumentos como:

"Este libro es barato, adquiérelo." (Basado en una premisa cuantitativa)


"Este libro está muy bien escrito, adquiérelo." (Basado en una premisa cualitativa)
"Este libro no volverás a verlo, adquiérelo." (Basado en una premisa que remite al
valor de lo único)
"Si no compras el libro, te reprueban." (Basado en una premisa que remite al criterio
del mal menor) [13]

9.1. Tomando lo estudiado en la unidad, conteste:

a. Según Toulmin, ¿en qué se diferencian los argumentos deductivos de la lógica


formal y los argumentos prácticos de la vida social?
b. ¿Por qué la lógica práctica que propone Toulmin incluye el concepto de campos
argumentativos? ¿Qué críticas recibe su teoría?
9.2. Señale con una (X) cuáles de las siguientes características corresponden a los
argumentos prácticos de la vida social según Toulmin:

a. Son analíticos.
b. Son independientes del contexto.
c. Dependen del contexto.
d. Fundamentan la conclusión de un modo absoluto.
e. Fundamentan la conclusión de un modo probabilístico.
Temas de la Unidad
3.1. La matemática: constructos formales y realidad
3.2. Sistemas axiomáticos
3.3. Propiedades de los sistemas axiomáticos
3.4. Interpretación y modelos de los sistemas axiomáticos

Bibliografía obligatoria Unidad 3 en la Guía de Estudio de IPC, producida por UBA


XXI y editada por Eudeba, a partir de la edición de 2010. Asti Vera, C. y Ambrosini, C.
(2009). Capítulo 4. En Argumentos y teorías. Aproximación a la Epistemología. Buenos
Aires: Educando.

BIBLIOGRAFÍA COMPLEMENTARIA Díez, J. A. y Moulines, C. U. (1999). Capítulos 4 y 5.


En Fundamentos de filosofía de la ciencia. Barcelona: Ariel. Klimovsky, G. (1994).
Capítulos 2, 3, 4 y 5. En Las desventuras del conocimiento científico. Buenos Aires: AZ.
Presentación de la Unidad

“¿Te apetece un poco de vino?", insinuó meliflua la Liebre de Marzo.


Alicia miró por toda la mesa sin ver más que té, por lo que observó: "No veo ese vino
por ninguna parte. "
"No lo hay ", replicó enseguida la Liebre de Marzo.
"Entonces, no ha sido nada amable el ofrecérmelo ", dijo Alicia enojada
"Tampoco lo ha sido el sentarse a esta mesa sin haber sido invitada " repuso la
Liebre.
(Carroll, L. Alicia en el país de las maravillas)

De acuerdo con el cuadro de clasificación de las ciencias presentado en la Unidad 1,


esta unidad está destinada a completar las características señaladas allí como propias
de las ciencias formales. Es decir, estudiaremos el "método axiomático" y los conceptos
que incluye su formulación. En este punto conviene detenerse y prestar atención a la
distinción entre la concepción clásica de las ciencias formales, identificada con
Aristóteles y la geometría de Euclides en el siglo III a.C. y la ruptura que se produjo en
este terreno a partir del cuestionamiento del V Postulado de Euclides, conocido como
el Postulado de las paralelas, a mediados del siglo XIX de nuestra era. Para
comprender mejor esta ruptura y cambio radical de teorías en el campo de las ciencias
formales, conviene que considere los principales aportes a las geometrías no
euclidianas, señaladas en este capítulo. Para ejemplificar los componentes de un
sistema axiomático, donde como en todo lenguaje, desde el punto de vista sintáctico,
encontramos términos (definidos y no definidos), proposiciones (demostrables:
teoremas y no demostrables: axiomas) y razonamientos (reglas deductivas), deberá
detenerse en la presentación de estos conceptos a partir de la Aritmética de Peano
(matemático y filósofo italiano, 1858-1932). En la revisión de las propiedades de los
sistemas axiomáticos, es necesario advertir la importancia del aporte del lógico,
matemático y filósofo estadounidense Kurt Gódel (1906-1978) y su cuestionamiento al
requisito de completitud para algunas regiones de las ciencias formales. Finalmente,
revisaremos las nociones de interpretación y modelo en las ciencias formales. Estas
nociones serán retomadas en los capítulos siguientes ya que una importante corriente
epistemológica considera a las teorías de las ciencias fácticas como "sistemas
axiomáticos interpretados" y encuentra elementos comunes en el análisis lógico de las
teorías fácticas y de las teorías formales. Actividades de aprendizaje

ACTIVIDAD 1. CONSTRUCTOS FORMALES Y REALIDAD


La relación entre ciencias formales y realidad reintroduce, en parte, las disputas o
debates en torno a la relación entre lenguaje y realidad, ya presentados en la primera
unidad puesto que las ciencias formales son de muy productiva aplicación en la
realidad (pensemos que no podríamos siquiera pagar el boleto del colectivo si no
tuviésemos un dominio de las operaciones matemáticas elementales; mucho menos
podrían construirse puentes o edificios de gran altura ni tampoco planificar actividades
económicas, para citar algunos ejemplos). Pero estas ciencias justifican el valor de
verdad de sus enunciados por medio de métodos totalmente formales, sin recurrir a la
contrastación empírica ni esperando su validación o justificación más que en
razonamientos deductivos. En el fragmento que se presenta a continuación, se trata de
destacar la diferencia entre un conocimiento formal (tautológico) y un conocimiento
empírico (contingente) y se propone volver al cuadro de clasificación de las ciencias
presentado en el libro de la bibliografía (cap. 1) que, como dijimos, sirve como un
"mapa" para ir completando a lo largo de todo el curso.

Lea el siguiente fragmento:


Definir una estructura es lo mismo que formular su teoría. Hay que especificar
cuáles son los conceptores, qué combinaciones de conceptores son los axiomas y
qué lógica determina la relación de consecuencia entre axiomas y teoremas. [...]
todo esto es independiente de la realidad empírica del mundo, todo esto es mera
matemática. En efecto, la matemática suele definirse como la ciencia de las
estructuras. En este sentido, todas las teorías son matemáticas. [...] Lo que nos
interesa es, en primer lugar, el abigarrado yjugoso mundo perceptual que nos
rodea, y en segundo lugar, el mundo que simbólicamente captamos con nuestro
lenguaje y con nuestros conceptos, en resumen, la historia. La teoría es un mero
instrumento para iluminar la historia. Pero la historia es siempre hipotética e
insegura. Sólo los fríos y vacíos teoremas de la teoría son seguros, pero no dicen
nada acerca del mundo. [...] En definitiva, poseemos un saber perfecto y seguro
sobre lo irreal, vacío y formal (las estructuras, objeto de las teorías), pero sólo un
saber imperfecto e inseguro sobre lo real, lo vivo y lo material (los sistemas objeto
de la historia).
Somos como las arañas, y las teorías son como las redes o telas de araña con que
tratamos de captar o capturar el mundo. No hay que confundir estas redes o telas
de araña con el mundo real pero, sin ellas, ¡cuánto más alejados estaríamos de
poder captarlo y, en último término, gozarlo![14]

B. Ahora le proponemos que conteste las siguientes preguntas:


a. Teniendo en cuenta las características de las ciencias formales, ¿cómo interpreta
la frase "la matemática suele definirse como la ciencia de las estructuras"?
b. Mosterín asocia el conocimiento perfecto y seguro a lo formal y el imperfecto e
inseguro a lo real. Explique esta distinción tomando en cuenta el cuadro de
clasificación de las ciencias de la bibliografía (cap. 1).
c. Este autor realiza una analogía entre los científicos y las arañas. Explique esta
analogía.

ACTIVIDAD 2. SISTEMAS AXIOMÁTICOS. ACERCA DE LA INTENCIONALIDAD


DE TODO ORDENAMIENTO LÓGICO Y DE LA NECESIDAD DE TENER UN
MÉTODO.
Una de las características destacadas del conocimiento científico es la utilización de
métodos o procedimientos que nos permitan llegar a la afirmación de teorías. En esta
unidad se presentan las características del "método axiomático" en tanto es el
adecuado para la construcción de teorías en ciencias formales. Recordemos que la
palabra método, derivada del griego, etimológicamente significa camino. En el pasaje
que se cita a continuación, Lewis Carroll nos indica que es necesario definir adónde
queremos llegar antes de saber qué método o camino seguir ya que, por definición,
todo camino nos conduce a algún lugar.
A. Lea el siguiente diálogo entre Alicia y el gato de Cheshire.
B. Luego, explique por qué Alicia afirma que la proposición: "Si sólo caminas bastante,
puedes estar segura de llegar a algún lado", es irrefutable. "Gatito de Cheshire"
—empezó con un poco de temor, porque no Y V estaba muy segura de que a élle
gustara el nombre. Sin embargo, el Gato sonrió algo más ampliamente. "Vamos, hasta
ahora le gusta", pensó Alicia y continuó:

"¿Querría decirme, por favor, qué camino debo tomar para irme de aquí?"
"Eso depende mucho del lugar adóndequieras llegar"—dijo el Gato.

"Me da lo mismo ellugar[...]"—dijo Alicia.

"Entonces no importa qué camino tomes"—dijo el Gato.

"[...]siempre y cuando llegue a algún lado —agregó Alicia a modo de explicación."

"Oh, puedes estar segura de llegar a algún lado "—dijo el Gato—, "si sólo caminas
bastante."

Alicia comprendió que esto era irrefutable, de modo que probó con otra pregunta [...]
(Carroll, L. 1990)

ACTIVIDAD 3. ACERCA DE LAS PROPIEDADES DE UN SISTEMA AXIOMÁTICO


Dado el siguiente sistema axiomático formal (SAF): Términos primitivos: M, I, U.
Axiomas: MI Reglas:
RI: Si se tiene una cadena cuya última letra sea I, se le puede agregar una U al final.
RII: Supongamos que se tenga Mx. En tal caso, puede agregarse Mxx a la colección.
("x" es cualquier cadena, por ejemplo: I, o IU, etc. Así podemos pasar de MI a MII
siendo I "x" e II "xx", o pasar de MIU a MIUIU siendo IU "x" e IUIU "xx"). Las cadenas
nunca pueden contener "x".
RIII: Si en una de las cadenas de la colección aparece la secuencia I I I, puede
elaborarse una nueva cadena sustituyendo I I I por U.
RIV: Si aparece UU en el interior de una de las cadenas, está permitida su
eliminación.[15] ¿Puede usted producir la palabra MU como teorema? El desafío consiste
en probar o bien que la palabra MU es un teorema en el sistema MIU, o bien que no lo
es. Si MU es un teorema, entonces puede ser generado a partir de su único axioma y
las cuatro reglas de transformación de fórmulas, y en tal caso habrá que mostrar su
derivación. Si MU no es un teorema de MIU, entonces habrá que probar que bajo
ninguna circunstancia puede ser generado por el axioma y las cuatro reglas de MIU.
Esta prueba no es una derivación en MIU, sino que tendrá la forma de una
demostración en el metalenguaje. Ése es el desafío.

MATERIALES DE LECTURA I
ACERCA DE LAS CARACTERÍSTICAS DEL CONOCIMIENTO DE LAS CIENCIAS
FORMALES Y SU DIFERENCIA CON EL CONOCIMIENTO DE LA REALIDAD TAL
COMO LO PRESENTA BERTRAND RUSSELL
En el siguiente texto, de modo algo irónico, Bertrand Russell destaca la diferencia
entre un mundo "ucrónico" (no afectado por el paso del tiempo) y el mundo real donde
todo es perecedero y el tiempo es una magnitud primordial. Con esta nota señala la
diferencia entre el conocimiento formal y el conocimiento fáctico. Aquí Russell alude al
mundo de las ciencias formales como un "mundo ucrónico", intemporal, es decir,
donde el paso del tiempo no cambia nada y donde no hay progreso, en el sentido en
que podríamos decir que no hay progreso entre adoptar como un juego al fútbol o al
tenis, sencillamente son dos juegos distintos. Como veremos en las siguientes
unidades, se debate acerca del progreso de las ciencias en el campo de las ciencias
fácticas. De todos modos, la aplicación de la noción de progreso a las ciencias formales
es motivo de debates epistemológicos, por ejemplo, si representa un progreso
científico la aparición de las geometrías no euclidianas.
Bertrand Arthur William Russell, tercer conde Russell, nació el 18 de mayo de
1872 y murió, a los 97 años, el 2 de febrero de 1970. Durante casi un siglo vivió una
vida asombrosamente variada. Bertrand Russell fue el segundo hijo del Vizconde de
Amberley, un título creado en 1861 para su abuelo, Lord John Russell, el primer
ministro liberal que luchó por la Ley de Reforma de 1831 y fue partidario decidido de
las doctrinas de John Stuart Mill. Su madre murió cuando tenía tres años y fue educado
por su padre en el agnosticismo, pero al año siguiente también él murió. Su abuela le
impartió una educación estrictamente victoriana en franca contravención a la voluntad
de sus padres muertos. Su abuelo, John Russell, había sido primer ministro de la Reina
Victoria de 1846 a 1852 y de nuevo de 1865 a 1866. De adulto, Russell recordaba
cuando se sentaba en las rodillas regias de Victoria durante las visitas de ésta a la
mansión de su abuelo. Según su propio relato, fue un joven solitario y reprimido.
Aludiendo a sí mismo confesó: "Yo no nací feliz[...] Durante mi adolescencia aborrecí la
vida y estuve continuamente al borde del suicidio, del cual me apartaba debido a mi
deseo de aprender más matemáticas". Llegado el momento, marchó al Trinity College,
en Cambridge, la misma institución que acogió al joven Isaac Newton, más de dos
siglos antes y que luego recibiría a otros genios de la talla de John Maynard Keynes y
Ludwig Wittgenstein, con quienes entabló una relación de profunda colaboración
intelectual. Inmediatamente luego de entrar a la vida universitaria, Russell se entregó
a las matemáticas, algo impersonal que, en sus propias palabras, podía amar sin ser
amado en reciprocidad. Para Russell, las matemáticas ofrecían una única vía para la
certeza y perfección. "Me desagradaba el mundo real, y busqué refugio en el mundo
ucrónico, sin cambio ni corrupción ni el fuego fatuo del progreso."

MATERIALES DE LECTURA II
ACERCA DE BORGES Y LOS INFINITOS
El primer párrafo del cuento de Borges "El libro de arena",[16] dice:
La línea consta de un número infinito de puntos; el plano, de un número infinito de
líneas; el volumen, de un número infinito de planos; el hipervolumen, de un número
infinito de volúmenes... No, decididamente no es éste, more geométrico, el mejor
modo de iniciar mi relato. Afirmar que es verídico es ahora una convención de todo
relato fantástico; el mío, sin embargo, es verídico.
En este cuento Borges problematiza la idea de infinito. Según el relato, el
protagonista compra a un vendedor que toca a su puerta un curioso libro, muy pesado.
La particularidad de este libro es que una vez abierta una hoja ya nunca se la volverá a
encontrar. En este curioso libro entre la tapa y la primera página hay infinitas páginas
de modo que se presenta la situación anómala de que nunca podríamos acceder a la
primera página porque siempre aparecerá otra. "Ninguna es la primera, ninguna es la
última", afirma el vendedor. El protagonista lo compra pero luego de un tiempo se
deshace del libro que considera tan monstruoso como él mismo. Así nos lo cuenta
Borges:
Declinaba el verano, y comprendí que el libro era monstruoso. De nada me sirvió
considerar que no menos monstruoso era yo, que lo percibía con ojos y lo palpaba con
diez dedos con uñas. Sentí que era un objeto de pesadilla, una cosa obscena que
infamaba y corrompía la realidad.
Pensé en el fuego, pero temí que la combustión de un libro infinito fuera parejamente
infinita y sofocara de humo al planeta.
Recordé haber leído que el mejor lugar para ocultar una hoja es un bosque. Antes de
jubilarme trabajaba en la Biblioteca Nacional, que guarda novecientos mil libros; sé
que a mano derecha del vestíbulo una escalera curva se hunde en el sótano, donde
están los periódicos y los mapas. Aproveché un descuido de los empleados para perder
el Libro de Arena en uno de los húmedos anaqueles. Traté de no fijarme a qué altura
ni a qué distancia de la puerta. Siento un poco de alivio, pero no quiero ni pasar por la
calle México.
En el siguiente texto, el escritor y matemático Guillermo Martínez profundiza sobre
este tema en su libro Borges y la matemática:[17] Dice Guillermo Martínez (Argentina,
1962): "Hay elementos de matemática muy variados a lo largo de la obra de Borges.
Los cuentos 'El disco', 'El libro de arena', 'La biblioteca de Babel', 'La lotería de
Babilonia', 'Del rigor en la ciencia', 'Examen de la obra de Herbert Quain', Argumentum
ornithologicum'; los ensayos La perpetua carrera de Aquiles y la tortuga' junto con
'Avatares de la tortuga', 'El idioma analítico de John Wilkins', 'La doctrina de los ciclos',
'Pascal' junto con 'La esfera de Pascal', etc. Hay textos que son incluso pequeñas
lecciones de matemática. Uno de los cuentos señalados es 'El libro de arena'. ¿Qué
ocurrirá cuando consideramos los números fraccionarios? Los números fraccionarios
son muy importantes en el pensamiento de Borges. ¿Por qué? Recordemos que los
números fraccionarios, que también se llaman quebrados, o números racionales, son
los que se obtienen al dividir números enteros, los podemos pensar como pares de
enteros: un número entero en el numerador y un número entero (distinto de cero) en
el denominador. 3/5, 5/4, 7/6, 7/16... ¿Cuál es la propiedad que tienen estos
números, la propiedad que usa Borges en sus relatos? Entre dos números fraccionarios
cualesquiera siempre hay uno en el medio. Entre 0 y 1 está 1/2, entre 0 y 1/2 está 1/4,
entre 0 y 1/4 está 1/8, etc. Digamos, siempre se puede dividir por 2.

De modo que cuando yo quiero saltar del 0 al primer número fraccionario,


nunca puedo encontrar ese primer número en el orden usual, porque siempre hay uno
en el medio. Ésta es exactamente la propiedad que toma prestada Borges en 'El libro
de Arena'. Recordarán que hay un momento en este cuento en que al personaje de
Borges lo desafían a abrir por la primera hoja el Libro de Arena. 'Me dijo que su libro
se llamaba el Libro de Arena porque ni el libro ni la arena tienen principio ni fin.
Me pidió que buscara la primera hoja. Apoyé la mano izquierda sobre la portada y abrí
con el dedo pulgar casi pegado al índice. Todo fue inútil: siempre se interponían varias
hojas entre la portada y la mano. Era como si brotaran del libro'."

MATERIALES DE LECTURA III


ACERCA DE KURT GÖDEL Y JOHN NASH. MENTES QUE BRILLAN
Es frecuente y parte del imaginario popular la asociación entre genios matemáticos y
esquizofrenia. La película norteamericana A Beautiful Mind, traducida como una mente
brillante, alude a los trastornos psiquiátricos del matemático norteamericano John
Nash, Premio Nobel de Economía de 1994, por sus aportes a la teoría de juegos y los
procesos de negociación, junto con Reinhard Selten y John Harsanyi. Existe una vasta
literatura tanto científica como no científica que asimila al genio, es decir, a una
persona con talentos intelectuales más desarrollados o un coeficiente intelectual
superior, con un carácter extravagante, insociable o patológico. En esta actividad
presentamos algunas de las circunstancias de la vida de Gódel, no tanto para
detenernos en detalles biográficos, sino para mostrar el ambiente intelectual de la
producción de teorías revolucionarias en ciencias formales en las primeras cinco
décadas del siglo XX, visto desde los avatares personales de algunos de sus principales
protagonistas que padecieron las consecuencias de dos guerras mundiales, la
persecución racial o política, el exilio y, en muchos casos, la incomprensión de sus
contemporáneos.
Kurt Gódel nació el 28 de abril de 1906 en Brünn, Moravia, en la época parte del
Imperio Austro-húngaro. Ingresó en la Universidad de Viena en 1924 planeando
estudiar Física Teórica. Hacia 1926 su atención volvió a las matemáticas y se produjo
su unión a lo que más tarde fue conocido como el Círculo de Viena. Gódel estuvo
asociado con este grupo durante muchos años.
Gódel se fue interesando progresivamente en Teoría de Números y, después, en
Lógica Matemática durante estos años. En 1930, Gódel se doctoró en matemáticas
dirigido por H. Hahn, un notable matemático miembro del Círculo de Viena. A partir de
aquí Gódel comienza a trabajar en sus más importantes teorías sobre la completitud de
sistemas formales. Viajó a los Estados Unidos dando un ciclo de conferencias y se
encontró por primera vez con Albert Einstein en 1933. Gódel pasó el año académico
1933-34 en Princeton, en el recién fundado Instituto de Estudios Avanzados, donde
disertó sobre sus resultados de incompletitud. Fue invitado a volver al año siguiente,
pero al poco de regresar a Viena sufrió una grave crisis mental. Se recuperó a tiempo
para retornar a Princeton en el otoño de 1935; al mes de su llegada sufrió una recaída,
y no volvió a impartir enseñanza hasta la primavera de 1937, en Viena.
Dedicó alguno de los años siguientes al estudio de problemas de Física y de
Psicología. Durante esta época tuvo que ser ingresado varias veces en hospitales por
problemas de salud. Por ser confidencial el historial médico de Gódel, la diagnosis de
su mal sigue siendo desconocida. Sus problemas parecen haber comenzado con
hipocondría: estaba obsesionado por su dieta y por sus hábitos intestinales. Durante
veinte años llevó un registro diario de su temperatura corporal y de su consumo de
leche de magnesia. Temía sufrir un envenenamiento accidental; con los años, le
aterraba ser objeto de una intoxicación deliberada. Esta fobia le llevó a no querer
tomar alimentos, con la consiguiente desnutrición. Lo que no le impedía ingerir píldoras
de diversa condición para un imaginario problema cardíaco. Gódel se casó con Adele
Porkert en 1938 y decidieron trasladarse definitivamente a los Estados Unidos en 1940.
Más de una vez Adele, sirviéndole de catadora de alimentos, contribuyó a paliar los
temores de Gódel, cada vez más fuertes, de que buscaban envenenarlo. Se asentaron
en Princeton, New Jersey, donde residieron hasta el final de sus vidas. Llegó a ser un
gran amigo de Einstein, y trabajaron juntos los aspectos filosóficos y matemáticos de la
Teoría General de la Relatividad. Gódel incluso trabajó con éxito en las ecuaciones del
campo gravitatorio, encontrando soluciones sorprendentes. También dedicó gran parte
de esa época al estudio del concepto de tiempo, publicando varios artículos y dando
varias conferencias sobre el tema. El último de sus artículos publicados en vida
apareció en 1958. Después, se sumió en la introversión, cada vez más demacrado,
paranoide e hipocondríaco. Su última aparición pública aconteció en 1972, al recibir un
doctorado honorífico por la Universidad Rockefeller. Tres años después le fue otorgada
la Medalla Nacional de Ciencias, pero Gódel disculpó su asistencia por razones de
salud. El 1 de julio de 1976, alcanzados los 70 años, edad de jubilación obligatoria,
Gódel se convirtió en profesor emérito. Sus responsabilidades empero no
disminuyeron, porque su esposa, que durante tantos años lo había alimentado y
protegido, había sufrido pocos meses antes un ataque cardíaco que la dejó inválida.
Ahora le correspondía a él cuidarla. Y así lo hizo, con devoción, hasta julio de 1977,
cuando ella hubo de someterse a una operación de urgencia y permaneció
hospitalizada durante casi seis meses. Por aquellas fechas, Morgenstern, el amigo que
había contribuido a cuidar de Gódel tras fallecer Einstein en 1955, murió de cáncer.
Gódel tuvo entonces que luchar por sí solo contra su cada vez más acusada paranoia.
Solo frente a ella, su declive se aceleró. Temeroso de ser envenenado, dejó de comer y
acabó muriendo por desnutrición el 14 de enero de 1978.
MATERIALES DE LECTURA IV
GÖDEL PARA TODOS
En la Feria del Libro de la ciudad de Buenos Aires realizada en abril de 2009 se
presentó el libro de Guillermo Martínez y Gustavo Piñeiro, Gódel para todos.[18] En una
nota de la época, se hace un comentario del libro en el que encontramos una breve
reseña de los principales hitos en la historia de las ciencias formales.

Gödel para todos


En la evolución de cada disciplina existen hitos, momentos de ruptura. En el caso de
la matemática, el Teorema de Incompletitud de Gódel marca un antes y un después. El
método axiomático-deductivo, fuente de toda verdad desde que fuera introducido por
Platón y Aristóteles y plasmado por Euclides, mostrará importantes limitaciones:
existen verdades no demostrables. En Gódel para todos, Guillermo Martínez y Gustavo
Piñeiro se plantean el inmenso desafío de desarrollar la demostración completa del
Teorema de Gódel con rigor absoluto y accesible a todos.
Los egipcios y los babilonios comienzan a construir la matemática en torno de
problemas ligados a la medición de la Tierra. Los griegos toman el relevo y constatan
que, para mejorar su eficacia, la matemática debe revisar sus fundamentos. Así, en los
Segundos Analíticos, Aristóteles señala: "La ciencia demostrativa debe partir de
premisas verdaderas, primeras, inmediatas, más familiares, anteriores, causas de la
conclusión". En este contexto surgen los elementos de Euclides (325-265 a.C.), que
organizan por primera vez los principales resultados de la Geometría a partir de diez
verdades primeras. Euclides distingue entre postulados y nociones comunes o axiomas.
Las nociones comunes son verdaderas en todos los campos del pensamiento e incluyen
afirmaciones tales como: "Las cosas iguales a una misma cosa son iguales entre sí", y
los postulados se aplican a temas específicos de la geometría como: "Dos puntos
determinan una recta". Desde Euclides, para validar sus resultados, la matemática
recurre a un método especial, la demostración deductiva a partir de estas verdades
primeras. En la formulación de los postulados de Euclides hubo cierta incomodidad. Las
sospechas caían sobre el Postulado de las paralelas (quinto postulado): "Dados una
recta y un punto exterior a ella puede trazarse una única recta paralela a la recta dada
que pase por ese punto". ¿Era necesario considerarlo un postulado o se podría deducir
de los otros, en cuyo caso era en realidad un teorema? En 1799, Karl Friederich Gauss
escribe: "He hecho algunos progresos en mi trabajo que no conducen en absoluto a la
meta que buscamos (la deducción del quinto postulado); más bien parecen obligarme
a dudar de la geometría misma". A partir de 1831, Gauss desarrolla una geometría
basada en la afirmación de todos los postulados y la negación del quinto, y concluye
que su trabajo no conlleva contradicción. Muy asombrado afirma: " Los teoremas de
esta geometría parecen paradójicos y, para los no iniciados, absurdos, pero una
reflexión tranquila y sosegada revela que no contiene en absoluto nada imposible". Los
trabajos de Nicolái Lobachevski, János Bolyai y Georg Riemann profundizan las ideas
de Gauss, mostrando que es posible negar el postulado de las paralelas sin generar
contradicciones y desarrollar las consecuencias del nuevo conjunto de axiomas. Dan
fundamento al descubrimiento de nuevas geometrías -no euclidianas- e instalan la idea
de que los axiomas y postulados de Euclides son verdades empíricas más que verdades
evidentes por sí mismas. En particular, deja de tener sentido distinguir entre
postulados (verdades de la matemática) y nociones comunes (verdades fuera de la
matemática). Las verdades primeras pasan a considerarse todas axiomas,
independientes de la intuición y la experiencia. En la segunda mitad del siglo XIX, los
resultados sobre las geometrías no euclidianas hacen que los matemáticos pongan en
duda los fundamentos de otras zonas de la disciplina. Esto lleva a un trabajo de
revisión y a una preocupación por elaborar una fundamentación sólida eliminando los
conceptos vagos y las contradicciones, que resultan en axiomas. Como ejemplo de
conceptos que se busca precisar, podemos citar la ambigüedad de Euclides al hablar
de puntos que están entre otros puntos sin aclarar la noción de "estar entre", y al
definir una recta como "aquella que yace por igual respecto de los puntos que están en
ella", sin aclarar qué entiende por "yacer por igual". David Hilbert, principal impulsor de
la matemática de la segunda mitad del siglo XIX y primera del XX, propone no definir
los conceptos básicos y explícita: "Las nociones de puntos, rectas y planos pueden
reemplazarse por mesas, sillas y jarros de cerveza". Siguiendo a Hilbert, basta que los
axiomas hagan afirmaciones acerca de conceptos no definidos. Si el punto o la recta
son conceptos no definidos, el axioma que dice que dos puntos determinan una sola
recta proporciona una afirmación que puede ser utilizada para obtener posteriores
resultados acerca de puntos y rectas. En términos coloquiales, no importa quién eres,
basta saber con quién andas. La intención de Hilbert era refundar la matemática sobre
bases sólidas a partir de sistemas axiomáticos (o teorías), entendidos como conjuntos
de axiomas con reglas lógicas que permiten desarrollar demostraciones. El desafío era
encontrar un sistema axiomático consistente (libre de contradicciones) y completo, es
decir, que permita obtener a través de demostraciones todos los resultados verdaderos
de la matemática.
En 1929, el jovencísimo Kurt Gódel (1906-1978) presenta su tesis doctoral, en la que
demuestra su Teorema de Completitud, que refiere a la lógica de predicados, esto es,
a las afirmaciones, válidas en todo contexto, que sustentan el razonamiento
matemático. Es posible dar axiomas que permiten demostrar todas las afirmaciones de
esta clase. En 1930, casi todos los matemáticos estaban convencidos de que en todos
los sistemas axiomáticos sería posible encontrar teoremas de completitud similares;
elegidos adecuadamente los axiomas, toda afirmación verdadera en la teoría sería
deducible. Sin embargo, Gódel demuestra que no es así en su famoso primer Teorema
de Incompletitud (1931), en el que prueba que la aritmética elemental es incompleta:
no es posible dar axiomas que permitan demostrar todas las verdades de esta teoría.
[...] Se lee en la Introducción: "A diferencia de la teoría de la relatividad, en que por la
sofisticación de las ecuaciones, los mejores intentos de divulgación parecen
condenados a ejemplos con relojes y personas que no envejecen en viajes por el
espacio, en el caso del Teorema de incompletitud hay una buena noticia, y es que
puede darse una exposición a la vez rigurosa y accesible que no requiere ninguna
formación matemática, más que el recuerdo de la suma y la multiplicación tal como se
enseñan en la escuela primaria".
Extraído de: Gódel para todos. (2009, junio 6). Revista Ñ. [en línea]. [consulta: 26 de
julio de 2010]. http://www.revistaenie.clarin.com/notas/2009/06/06/_-01932842.htm

Actividad de integración de los temas de la Unidad


Determine si las siguientes afirmaciones son verdaderas o falsas (V ó F). Justifique en
cada caso. a. En una regla de inferencia deductiva nunca se da el caso de que de una
falsedad se obtenga una verdad. b. En una regla de inferencia deductiva nunca se da
el caso de que de una verdad se obtenga una falsedad. c. En un sistema axiomático,
todos los enunciados se demuestran. d. En un sistema axiomático, todos los términos
se definen. e. Las geometrías no euclidianas surgieron al cuestionarse la independencia
del V postulado de Euclides. f. Los teoremas son relativos a su sistema de pertenencia;
esto es, en otro sistema pueden funcionar como axiomas. g. Las geometrías no
euclidianas representan un avance científico que resta interés a la geometría de
Euclides. h. La independencia de los axiomas permite que unos se puedan deducir de
otros. i. Si todos los teoremas se demostraran, los sistemas axiomáticos serían viciosos
o infinitos.
Temas de la Unidad
4.1. Estructura de las teorías empíricas
4.2. El problema del método científico
4.3. Estrategias metodológicas básicas de las ciencias fácticas
4.4. Método inductivo: inductivismo "estrecho" e inductivismo "sofisticado"
4.5. Método hipotético-deductivo
Bibliografía obligatoria Unidad 4 en la Guía de Estudio de ipc, producida por UBA
XXI y editada por Eudeba, a partir de la edición de 2010. Asti Vera, C. y Ambrosini, C.
(2009). Capítulo 5. En Argumentos y teorías. Aproximación a la Epistemología. Buenos
Aires: Educando.
BIBLIOGRAFÍA COMPLEMENTARIA
Cohen, I. y Nagel, E. (1980). Capítulos X, XI, XIV y XVI. En Introducción a la lógica y
al método científico (vol. II). Buenos Aires: Amorrortu. Hempel, C. G. (1985). Capítulos
2, 3 y 4. En Filosofía de la ciencia natural. Madrid: Alianza. Klimovsky, G. (1994).
Capítulos 9, 13 y 14. En Las desventuras del conocimiento científico. Buenos Aires: AZ.
Presentación de la Unidad
Quién soy yo? ¡Ah!¡Eso sí que es un misterio!" Y
con esto se puso a en pensar en todas las niñas de su edad que conocía, para ver si se
había transformado en una de ellas. [...] "Voy a ver si al menos sé las cosas que antes
sabía. Veamos: cuatro por cinco son doce, cuatro por seis son trece, y cuatro por
siete... ¡Ay, Dios mío! ¡Así no llegaré nunca a veinte! Bueno, de todas formas la tabla
de multiplicar no me vale; probemos con la Geografía. Londres es la capital de París, y
París es la capital de Roma, y Roma es[...]"

(Carroll, L. Alicia en el país de las maravillas)

La distinción entre ciencias formales y fácticas reconoce diferencias entre ambos tipos
de saberes, lo que no significa necesariamente que no existen condiciones
constructivas comunes. Según algunos autores, una teoría empírica puede
considerarse como un conjunto de hipótesis de partida y sus consecuencias lógicas
(Popper, Bunge, Klimovsky). De allí que esta unidad comienza con la exposición del
lenguaje y la estructura de una teoría fáctica, realizando luego una presentación de la
cuestión del método en las ciencias fácticas, tema que es motivo de importantes
debates en el ámbito de la epistemología contemporánea. En su desarrollo
examinaremos las estrategias básicas de las ciencias empíricas, tanto como las
discusiones más importantes en el contexto de las metodologías rectoras en el siglo
XX. Entre otras cuestiones anticipan las controversias, aún no canceladas, en torno al
monismo metodológico y al pluralismo metodológico que estudiaremos de manera más
específica en las unidades siguientes y que afectan especialmente a las ciencias
sociales, a su constitución, a la legitimación de sus métodos y a su proyección
histórica. En esta unidad veremos distintas versiones acerca del modo de interpretar la
relación teoría-observación. Estas diferencias se presentan a partir de la distinción
propuesta por el filósofo alemán Hans Reichenbach (1891-1953) entre contexto de
descubrimiento y contexto de justificación, a los que el matemático y filósofo argentino
Gregorio Klimovsky (1922-2009) agrega el contexto de aplicación de las ciencias antes
de presentar la posición del filósofo, sociólogo y teórico de la ciencia austríaco Karl
Popper (1902-1942), quien confina la metodología al contexto de justificación. Esta
diferenciación entre contextos y la reclusión de la Epistemología a alguno de estos
contextos con exclusión de otros es motivo de fuertes polémicas, en especial entre las
posiciones de Popper y del epistemólogo estadounidense Thomas Kuhn (1922-1996), y
creemos que es importante advertir la presencia de dichas distinciones en esta unidad
ya que luego esas diferencias serán recuperadas en las unidades siguientes. A
continuación conviene que se detenga a estudiar con especial interés, en el capítulo 5
de la bibliografía, las distintas estrategias metodológicas básicas en las ciencias fácticas
donde encontrará señalada la especificidad de las ciencias sociales en la postulación de
métodos distintos a los de las ciencias naturales. Este punto es conveniente que lo
relacione con el Documento de Cátedra de Gastón Beraldi de la Unidad 5. En la Unidad
2, desde distintos enfoques, hemos visto la inducción como forma de razonamiento y
los esfuerzos de Carnap por legitimar este tipo de inferencia como un procedimiento
formal. En esta unidad, enfocaremos el problema del inductivismo considerado como
una posición metodológica, es decir, la posición epistemológica que considera a la
inducción como método de investigación científica. El llamado por sus críticos
"inductivismo ingenuo" defiende la inducción por enumeración simple, cree que la
ciencia empieza con la observación y sostiene que la observación ofrece una base
segura a partir de la cual se puede derivar el conocimiento. En su versión refinada, el
inductivismo tiene métodos más sofisticados que la enumeración simple, pero sigue
reconociendo a la observación como primera fuente de conocimiento. En referencia al
llamado "inductivismo ingenuo", debe detenerse en la lectura y comprensión de los
llamados "métodos de Mill", que toman el nombre de su creador, el filósofo inglés John
Stuart Mill (1806-1873). Se conocen estos cánones o reglas de investigación como
"método de la concordancia", "método de la diferencia", "método conjunto de la
concordancia y la diferencia", "método de los residuos" y "método de la variación
concomitante". Estos métodos buscan descubrir conexiones causales entre fenómenos.
La insistencia de Mill en la utilidad de estos métodos para descubrir conexiones
causales lo llevó a largas polémicas con sus contemporáneos y a reafirmar sus
convicciones en defensa de la lógica inductiva. Dada la importancia central de esta
posición en la historia de la epistemología de los siglos XIX y XX, en esta unidad
encontrará expuestas distintas versiones del inductivismo, comenzando con la
presentación de los métodos de Mill y las críticas del filósofo alemán Carl Hempel
(1905-1997) cuando lo caratula como inductivismo ingenuo antes de señalar las
versiones más elaboradas de Carnap y de Reichenbach. El estudio de este tramo del
capítulo 5 de la bibliografía es necesario para luego contextualizar las distintas
posiciones epistemológicas presentadas en la Unidad 6. Ya avanzando en la historia del
siglo XX, se presenta en este capítulo al "método hipotético-deductivo" en las versiones
confirmacionistas de Carnap y Hempel, representantes del llamado "inductivismo
sofisticado". Se presenta luego la versión refutacionista de Popper y el llamado
"refutacionismo sofisticado" del matemático y filósofo de la ciencia húngaro Imre
Lakatos (1922-1974). En la presentación de la posición refutacionista, adjudicada a
Popper, conviene que advierta la importancia del Modus Tollens como un argumento
deductivo que permite la refutación de una hipótesis y ligada a la importancia de este
tema, acceder a la comprensión del concepto de "falsabilidad" como criterio de
demarcación para considerar científica a una hipótesis. En el desarrollo de esta parte
de la unidad, conviene que lea atentamente los pasos del método hipotético-deductivo
y los distintos tipos de hipótesis involucradas en la puesta a prueba de las teorías
científicas (hipótesis fundamental, hipótesis auxiliar e hipótesis ad hoc). Estas
distinciones son importantes ya que serán consideradas como temas centrales en la
presentación de la posición de Lakatos en la Unidad 6. Dentro de la metodología de las
ciencias fácticas es importante la utilización de métodos estadísticos en ciencias
naturales y en ciencias sociales, de allí que éste sea el tema que cierra esta unidad. Si
bien la utilización de los métodos estadísticos ha tenido un gran desarrollo en el campo
de las ciencias, su justificación no deja de ser problemática y de dar lugar a la
incursión en diversos tipos de argumentos falaces. Al comienzo de este tema se realiza
una breve reseña histórica de los estudios sobre cálculos de probabilidad aludiendo a
los matemáticos franceses Blas Pascal y Pierre de Fermat (1608-1665). Se presenta el
concepto de "promedio estadístico" visto como elemento importante para luego revisar
las limitaciones de estos métodos según han sido señaladas por los filósofos
estadounidenses Morris R. Cohen (1880-1947) y Ernest Nagel (1901-1985). En este
punto es importante reconocer la conexión de este tema con el uso de procedimientos
inductivos y los problemas que se presentan cuando se trata de justificar este tipo de
inferencias. Actividades de aprendizaje

ACTIVIDAD 1. INDUCTIVISMO
El inductivismo es la posición epistemológica que postula la inducción como único
método de investigación científica. Ya se ha visto que la inducción es una forma de
razonamiento que no tiene validez lógica; sin embargo, para conocer el mundo
empírico y ampliar nuestro conocimiento acerca de él, con todas sus falencias, la
inducción sigue siendo imprescindible. El inductivismo ingenuo defiende la inducción
por enumeración simple, cree que la ciencia empieza con la observación y sostiene que
la observación ofrece una base segura a partir de la cual se puede derivar el
conocimiento. En su versión refinada el inductivismo tiene métodos más sofisticados
que la enumeración simple pero sigue reconociendo a la observación como primera
fuente de conocimiento. Para los inductivistas ingenuos la investigación tiene los
siguientes pasos:
1) Observación neutral de la realidad.
2) Captación de una propiedad en diversos individuos.
3) Generalización de dicha propiedad a todos los individuos.
4) Formulación de una ley o generalización empírica. El supuesto que permite ir del
paso 2 al paso 3 es el llamado Principio de Regularidad de los Fenómenos, principio
por el cual se cree que la naturaleza es constante, de modo que se hace valer para
todos los casos lo observado sólo en algunos. Lea el siguiente texto e identifique los
pasos de la investigación inductiva apuntados más arriba: Beber alarga la vida

Un estudio sobre los hábitos de ingestión de alcohol de los norteamericanos ha


arrojado un resultado sorprendente. Los hombres que toman entre 0,8 litros y 2,4
litros de cerveza a la semana tienen más oportunidades de vivir durante más tiempo.
Según el profesor David Williams, de la Universidad de Gales, en Cardiff (Reino Unido)
existen pruebas fehacientes de que las personas que beben cerveza moderadamente
tienen una expectativa de vida cinco años mayor que los abstemios.

ACTIVIDAD 2. INDICTIVISMO. LOS MÉTODOS DE JOHN STUART MILL


En el texto de la bibliografía, se mencionan los que se han considerado, en su
formulación clásica, "métodos de Mill" de la inferencia inductiva. Mill ha insistido en la
utilidad de estos métodos para descubrir conexiones causales y reafirmado sus
convicciones en defensa de la lógica inductiva: "La misión de la lógica inductiva es
proveer de reglas y modelos (como el silogismo y sus reglas son modelos para el
raciocinio) que, si los razonamientos inductivos se adecuan a ellos, son concluyentes,
pero en caso contrario no lo son." Según Mill, sus métodos permitirían descubrir y
probar conexiones causales. Es indudable que esta pretensión no se ha realizado,
afirma Copi, puesto que la utilización de métodos mecánicos no alcanzó, en muchos
casos, para descubrir la causa de algunos fenómenos ya que no hay ningún recurso
simple o método mecánico que por sí mismo haya permitido conquistar el
conocimiento científico. "Para hacer un análisis correcto en la aplicación de estos
métodos se necesita el conocimiento previo de teorías", concluye Copi. (1984.
452-453)

2.1. El bebedor científico


Irving Copi en su libro introducción a la lógica (1984) cita el caso de un bebedor que
todas las noches se emborracha. Estaba arruinando su carrera y su salud. Uno de los
pocos amigos que le quedaba le aconsejó abandonar la bebida. Resolvió llevar a cabo
un cuidadoso experimento para descubrir la causa exacta de las borracheras al tomar
distintas mezclas de bebidas. Durante cinco noches seguidas tomó gin con soda,
whisky con soda, aguardiente con soda, cognac con soda y ron con soda. Usando el
método de la concordancia de Mill llegó a la conclusión: "Nunca más tomo soda".
Responda: ¿El método de la concordancia de Mill avalaría esta conclusión? ¿Dónde
radica la falacia?

2.2. Pasteur y el experimento con la vacuna contra el carbunco


En la primavera de 1881, Pasteur puso a prueba su hipótesis de que la vacuna de
carbunco produce inmunidad a la enfermedad, hipótesis ridiculizada por los
veterinarios de la época. En una granja de Poully-le-Fort se administró la vacuna a 24
ovejas, una cabra y varias vacas. Se tomó como muestra de control otras veinticuatro
ovejas, una cabra y varias vacas que quedaron sin vacunar. Dos meses después las 48
ovejas, las dos cabras y todas las vacas fueron inoculadas con una dosis letal de
virulentos bacilos de carbunco. Tres días después los animales del primer grupo
brincaban y comían normalmente como si nunca hubieran estado en contacto con el
bacilo, mientras que los animales no vacunados estaban muertos o en una irreversible
agonía.
Así mostrado, este experimento parece responder al método conjunto de la
concordancia y la diferencia. Identifique los pasos de la investigación en el texto. Este
experimento ¿prueba que es verdadero el enunciado "la vacuna inmuniza contra la
enfermedad"? ¿Por qué?

2.3. El piojo y el tifus


Se supuso durante mucho tiempo que el virus del tifus es inoculado por el piojo al
chupar la sangre, pero no es así. La infección no está en la saliva, como en el caso del
mosquito, sino en las heces. Al ser picado, el animal se rasca y la infección entra en
contacto con la sangre de tal modo que "picadura" e "infección" son fenómenos
inseparables. En 1922 dos biólogos pusieron piojos infectados sobre un mono,
tomando la precaución de que las heces no entraran en contacto con la piel del mono.
A pesar de las picaduras, el mono no se infectó.
Responda: ¿Cuál de los métodos de Mill puede advertirse en este experimento? Los
resultados del experimento ¿son concluyentes para afirmar que "las heces del piojo
contagian la enfermedad"?

ACTIVIDAD 3. ACERCA DEL MÉTODO HIPOTÉTICO-DEDUCTIVO


El amor es una droga dura es el título de una novela de la escritora uruguaya Cristina
Peri Rossi,[19] residente en Barcelona. La historia trata sobre un fotógrafo de 50 años,
felizmente casado y asentado en su profesión, que superó las secuelas de una vida de
excesos y adicciones al alcohol y las drogas pesadas pero que sucumbe a la última
tentación: Nora. Del mismo lugar, el diario La vanguardia (Barcelona), el 2 de junio de
2005, publica una nota donde da cuenta de investigaciones científicas acerca del
funcionamiento del cerebro humano, titulada "El amor como adicción". Lea
atentamente el texto y, luego, conteste las preguntas. La Vanguardia
Josep Corbella
Barcelona

EL AMOR COMO ADICCIÓN


El cerebro humano experimenta el enamoramiento igual que una adicción, según
investigadores de Estados Unidos que han observado por resonancia magnética qué
ocurre en el cerebro de personas que han iniciado hace poco una relación de pareja.
La investigación responde a viejos debates sobre la relación entre amor y sexo. Según
resultados publicados el martes en The Journal of Neurophysiology, la atracción física y
el enamoramiento son procesos diferentes que activan regiones distintas del cerebro.
Esto explicaría, según los autores de la investigación, por qué una persona puede
encontrar atractivas a múltiples parejas potenciales, pero difícilmente enamorarse de
más de una a la vez: la región que procesa el enamoramiento tiende a la monogamia,
mientras que la que procesa la atracción física tiende a la poligamia.
Los investigadores -de la Universidad del Estado de Nueva York, la Universidad
Rutgers de Nueva Jersey y la Escuela de Medicina Albert Einstein de Nueva York- han
analizado a 10 mujeres y 7 hombres de 18 a 26 años que declararon llevar entre 1 y
17 meses "intensamente enamorados".
Las resonancias magnéticas indican que las regiones del cerebro que se activan
cambian a medida que una relación de pareja madura. En las parejas que llevan entre
uno y siete meses juntas se observa una hiperactividad en áreas involucradas en las
adicciones, sobre todo el área tegmental ventral y el núcleo caudado. Pero en parejas
que llevan más tiempo juntas empieza a activarse el pálido ventral, que parece vital
para establecer relaciones duraderas. El estudio muestra que, a medida que pasan los
meses y unas áreas del cerebro se activan, otras se desactivan. Esto explicaría por qué
una relación duradera y gratificante no impide que una pareja pueda verse sorprendida
por un enamoramiento imprevisto: mientras el área del cerebro que garantiza la
relación estable permanece activa, la habitación de la pasión, en el área tegmental
ventral, se encuentra disponible.
Pero el resultado que los autores de la investigación consideran más relevante es que
el enamoramiento no es una emoción sino más bien una adicción. "No hemos
encontrado ningún patrón emocional consistente", declara en un comunicado Arthur
Aron, codirector de la investigación de la Universidad del Estado de Nueva York,. Las
emociones del enamoramiento, advierte Aron, pueden oscilar de manera caótica entre
la euforia, la ansiedad, el enfado, la tristeza o la alegría. Por el contrario, "todos
nuestros voluntarios mostraron una actividad intensa en las regiones de motivación y
recompensa del cerebro". Estas regiones son las mismas que se activan en las
adicciones. Igual que en una adicción, los investigadores han observado que el
enamoramiento se asocia a intensas descargas de dopamina en el centro del cerebro.
Y otro punto en común con las adicciones: cuando una persona que se encuentra en
esta fase efervescente es rechazada por su pareja, presenta un patrón de actividad
cerebral similar al de un síndrome de abstinencia, según un nuevo estudio del mismo
equipo de investigación difundido por The New York Times. Este síndrome de
abstinencia explicaría por qué muchas rupturas, más que inhibir el deseo de estar con
la otra persona, lo acrecientan. Ignasi Morgado, catedrático de Psicobiología de la
Universitat Autónoma de Barcelona (UAB), recordó ayer que los estudios de Samir Zeki
y Andreas Bartels en el Colegio Universitario de Londres han mostrado una inhibición
de la actividad del córtex prefrontal -la principal sede de la racionalidad en el cerebro-
en personas que declaran estar muy enamoradas. "Por lo tanto, parece hacer falta una
cierta irracionalidad para el enamoramiento", señaló.
Las principales áreas del cerebro involucradas en el enamoramiento trabajan a nivel
inconsciente y son comunes a todos los mamíferos. Los autores del estudio recuerdan
además que hay una especie de ratones de las praderas que establecen relaciones
monógamas duraderas gracias a la activación del mismo área del cerebro que las
personas.
Todo ello apunta a que "el enamoramiento es posiblemente un tipo de impulso básico
de los mamíferos para optimizar el proceso de apareamiento", afirma Helen Fisher,
codirectora del estudio. Lo cual, sin embargo, no significa que la experiencia del
enamoramiento sea igual en la especie humana que en otros mamíferos, ya que el
cerebro humano tiene una capacidad de ser consciente del proceso, superior al de
cualquier otra especie. Y tampoco significa, advierten los investigadores, que el
enamoramiento pueda reducirse a los procesos fisiológicos que se observan en las
resonancias magnéticas, ya que es un fenómeno complejo condicionado por influencias
culturales.

Josep Corbella. (2005, junio 2).


El amor como adicción. La vanguardia.
A. Según el método hipotético-deductivo en su versión falsacionista:
a. ¿Cuál es el problema?
b. ¿Cuál o cuáles son las hipótesis de partida de los neurólogos?
c. ¿Qué consecuencias observacionales pondrían a prueba estas hipótesis?
d. ¿Considera suficiente la base observacional ofrecida por los neurólogos
mencionados? Justifique su respuesta.
e. ¿Se ha demostrado la verdad de alguna hipótesis, según este informe? Justifique su
respuesta.

B. ¿Encuentra falsables los siguientes enunciados? Justifique sus respuestas en cada


caso.
a. "El enamoramiento es un fenómeno complejo condicionado por influencias
culturales".
b. "El enamoramiento es posiblemente un tipo de impulso básico de los mamíferos
para optimizar el proceso de apareamiento."

ACTIVIDAD 4. MÉTODO HIPOTÉTICO-DEDUCTIVO. ACERCA DE LOS TIPOS


DE HIPÓTESIS
En el libro de la bibliografía encontrará la distinción entre distintos tipos de hipótesis
que pueden participar en la puesta a prueba de una teoría: hipótesis fundamental,
hipótesis auxiliar e hipótesis ad hoc. Cuando hay que derivar consecuencias
observacionales de las hipótesis principales, a menudo hay que hacer suposiciones
adicionales ya que estas hipótesis principales, por sí solas, no permiten derivar
directamente consecuencias observacionales. A estas suposiciones adicionales se las
llama "hipótesis auxiliares". Se supone que estas hipótesis han sido contrastadas con
anterioridad, de forma independiente, con la hipótesis principal y pueden o no formar
parte de la disciplina a la que pertenece la hipótesis principal. Una hipótesis auxiliar se
considera ad hoc cuando no puede ser contrastada de forma independiente de la
principal y es usada para salvar a la principal de ser refutada. Lea el siguiente texto y,
luego, responda las preguntas.

Martes 6 de enero de 2009


LA ESFERIDAD DE LA TIERRA 2: DE LA ANTIGÜEDAD A COLÓN
La idea de una Tierra esférica es muy antigua y se remonta al menos a los tiempos de
la filosofía griega clásica y posiblemente también a la filosofía india antigua.
El concepto de una Tierra esférica desplazó a antiguas creencias sobre una Tierra
plana. En el pensamiento mesopotámico se consideraba que el mundo era un disco
plano flotando en el océano y este retrato de la Tierra fue plasmado en los primeros
mapas de la Tierra realizados por Anaximandro y Hecateo de Mileto. [...] Los primeros
filósofos griegos se refieren a una Tierra esférica aunque un tanto ambiguamente.
Pitágoras (nacido en 570 a.C.) basaba la creencia de que la Tierra y los demás
planetas eran esféricos porque, para él, el sólido geométrico más armónico era la
esfera. [... ] Eratóstenes (276-194 a.C.) no sólo creía que la Tierra era redonda, sino
que además estimó sus dimensiones. No poseía computadoras, ni satélites de
reconocimiento, tan sólo una mente brillante y conocimientos de trigonometría.
Eratóstenes escuchó que al sur en Siena durante el solsticio de verano el Sol
permanecía en el cénit de forma que los objetos no arrojaban sombra alguna. Se dice
que Eratóstenes contrató a otra persona para medir la distancia entre Siena y
Alejandría mediante la cuenta de sus pasos. Al llegar el solsticio de verano se hicieron
dos observaciones simultáneas. Mientras que en Siena una estaca no arrojaba sombra
ninguna a mediodía, en Alejandría existía una pequeña sombra. Eratóstenes midió el
ángulo de la sombra que resultó ser de unos 7,2°, como conocía la distancia a Siena,
por un sencillo cálculo trigonométrico calculó la circunferencia de la Tierra en 250.000
estadios (una medida griega de longitud) traducido a valores actuales nos damos
cuenta de que Eratóstenes se equivocó en apenas un 3% en su estimación, sin duda
un logro impresionante. [...]
[20]
a. ¿Cuál era la hipótesis principal de Eratóstenes? b. ¿Qué consecuencias
observacionales se derivaban de ella? c. ¿Qué pudo probarse, finalmente?
d. ¿Encuentra alguna hipótesis auxiliar implícita en este experimento, entre las que se
mencionan a continuación? Justifique su elección. i) La forma en que caían los rayos
del sol. ii) La distancia de Siena a Alejandría. iii) La existencia de dos obeliscos.
e. Reflexione acerca de la importancia de las hipótesis auxiliares. ¿Puede una hipótesis
auxiliar falsa malograr la aceptación de la hipótesis principal? ¿Por qué?
Temas de la Unidad
5.1. ¿Qué significa "explicar"?
5.2. Concepto de explicación científica
5.3. Modelos de explicación científica
5.4. La especificidad de las ciencias sociales: intencionalidad y explicación por
mecanismos
5.5. Dimensión explicativa y dimensión predictiva
5.6. El enfoque comprensivista en ciencias sociales

Bibliografía obligatoria Unidad 5 en la Guía de Estudio de IPC, producida por UBA


XXI y editada por Eudeba, a partir de la edición de 2010. Asti Vera, C. y Ambrosini, C.
(2009). Capítulo 6. En Argumentos y teorías. Aproximación a la Epistemología. Buenos
Aires: Educando. Beraldi, G. (2010). Documento de cátedra: La tensión entre
explicación y comprensión. El problema de la explicación en las ciencias sociales. En
esta Guía de Estudio de IPC, Buenos Aires: Eudeba; y en el Campus virtual de UBA
XXI.

BIBLIOGRAFÍA COMPLEMENTARIA
Díez, J. A y Moulines, C. U. (1999). Capítulo 7. En Fundamentos de filosofía de la
ciencia. Barcelona: Ariel. Schuster, G. (1986). Capítulos 2, 3, 4 y 5. En Explicación y
predicción. Buenos Aires: Clacso. Wright, G. H. von (1979). Explicación y comprensión.
Madrid: Alianza.
Presentación de la Unidad
Sabes por qué son blancas las pescadillas?"
"Nunca me lo he preguntado", respondió Alicia. "¿Porqué?"
"Pues porque sirven para darles brillo a los zapatos y a las botas", explicó el Grifo con
gran solemnidad, "por lo blancas que son ".
Alicia se quedó de una pieza. "¡Para sacar brillo!", repetía, sin saber cómo
explicárselo.
"¡Pues claro! ¡A ver! ¿Cómo se limpian los zapatos?", le preguntó el Grifo. "Quiero
decir, ¿cómo se les saca brillo?
Alicia se miró los pies y reflexionó un poco antes de dar una contestación: "Con negro
de betún me parece".
"Pues bajo el mar a las botas y a los zapatos se les da con blanco de pescadilla",
interpuso el Grifo con voz pretenciosa: "Ya lo sabes."

(Carroll, L. Alicia en el país de las maravillas)

En el capítulo 6 de la bibliografía, se examinan los modelos de explicación científica


más transitados por la Epistemología contemporánea así como los debates que se
generan en torno a las modalidades explicativas en ciencias sociales. Además de los
modelos de explicación nomológico-deductivo y estadístico-inductivo, en esta parte se
debe prestar atención a las características de la explicación genética y de la explicación
teleológica. En este texto resulta de especial interés la revisión de la particular posición
del filósofo noruego Jon Elster (n. 1940) quien, a partir de sus estudios sobre distintos
modelos de racionalidad, considera que el criterio de clasificación de las ciencias debe
tomar en cuenta las distintas estrategias explicativas. Reconoce tres modalidades
básicas de explicación: causal, funcional e intencional. En este orden, los campos de
investigación a los que se aplica cada una de éstas son: ciencias físicas, ciencias
biológicas y ciencias sociales. La explicación causal, que es el modelo único y canónico
de explicación en el campo de la física, también se emplea frecuentemente en los otros
dos campos de investigación. Elster critica el uso de explicaciones funcionales en
ciencias sociales y reserva para estas ciencias el uso de las explicaciones por
mecanismos, de allí la presentación de este tema que cierra el capítulo dedicado a
dicho tema en la bibliografía. El material de lectura de esta unidad se completa con el
Documento de Cátedra escrito por Gastón Beraldi, La tensión entre explicación y
comprensión. El problema de la explicación en las ciencias sociales, en el que aparece
el cuestionamiento a las versiones explicacionistas en las ciencias sociales y la
búsqueda de métodos propios. En este artículo se presentan a los principales
representantes del "comprensivismo", siguiendo el análisis de los filósofos Wilhelm
Dilthey (1833-1911, alemán) y Georg H. von Wright (1916-2003, finlandés),
identificados en oposición al Positivismo como antipositivistas aunque no representan
un grupo homogéneo ni permiten reducir sus posiciones a unos pocos postulados
comunes. El rasgo común del enfoque positivista es el monismo metodológico y una
posición "explicacionista" para el caso de las ciencias sociales. Aquí es importante
advertir la oposición de estos autores al monismo metodológico y la búsqueda de
estrategias explicativas propias para las ciencias sociales alrededor del concepto de
"comprensión".
Actividades de aprendizaje
MATERIALES DE LECTURA I
LA EXPLICACIÓN CIENTÍFICA
Mientras que describir es responder a la pregunta acerca de cómo es algo, explicar es
responder a la pregunta de por qué ese algo es o sucede. O sea que explicar es dar
cuenta de la producción y/o existencia de algún fenómeno. Toda teoría científica puede
ser vista como una gran explicación acerca de algún fenómeno, evento, suceso o
proceso. Mas no toda explicación científica constituye por sí sola una teoría completa.
Sin embargo, no hay teoría sin explicación, de allí la importancia que reviste este tema.
Empleamos el término "explicar" en distintos sentidos: Explicar qua narrar: "Ana me
explicó su viaje a China". Explicar qua elucidar: "Ana me explica las reglas de la
gramática". Explicar qua respuesta a una pregunta por qué": "Ana me explicó por qué
decidió viajar a China".
Partes. Toda explicación tiene dos partes: el explanandum y el explanans. El
enunciado que contiene el hecho a explicar se denomina explanandum y el conjunto de
enunciados que conforman la explicación propiamente dicha, se denomina explanans.
El explanandum puede describir un hecho puntual, por ejemplo "se inunda el patio de
mi casa", o una regularidad, "el corcho flota en el agua". En ambos casos el
explanandum encierra el enigma de por qué ocurren tales cosas, enigma que el
explanans va a resolver.
Requisitos. Para que una explicación sea científica debe ser atinente y contrastable.
La atinencia o relevancia explicativa es la pertinencia del explanans respecto al
explanandum y la contrastabilidad es la condición por la cual todo los enunciados de la
explicación tienen que poder ser puestos a prueba.
Tipos. Hay varios tipos de explicación: genéticas, teleológicas o funcionales,
intencionales, nomológico-deductivas y las estadístico-inductivas.
Explicaciones nomológico-deductivas y estadístico-inductivas. En ambos tipos
de explicación el explanans está formado por leyes y condiciones iniciales. Las leyes
son los enunciados más generales a los que los enunciados observacionales se
subsumen y las condiciones iniciales son los enunciados que describen las situaciones
de hecho que acompañaron la producción del fenómeno que el explanandum describe.
Explanans Modelo nomológico-deductivo Modelo estadístico-inductivo
L1,L2,L3..................Ln L1,L2,L3............Ln C1,C2,C3................Ln L1,L2,L3............Ln

----------------------------------- Explanandum Explanandum

La diferencia entre los dos tipos de explicación deriva de la diferencia que hay entre las
leyes que están en el explanans de uno u otro modelo. En efecto, las leyes pueden ser
de probabilidad máxima, o bien de alta probabilidad. Las leyes de probabilidad
máxima son aquellas tales como la Ley de Gravedad, la Ley de Boyle y Mariot, el
Principio de Arquímedes, etc., todos ellos enunciados nomológicos que describen
regularidades de la naturaleza. En cambio, las leyes de alta probabilidad son
enunciados estadísticos que describen regularidades probables, con un considerable
margen de excepciones. Se encuentran en este último grupo la mayor parte de las
leyes estadísticas de las ciencias sociales, por ejemplo las que vinculan enfermedades
virósicas y contagio (Salud), o pobreza e índice de mortalidad infantil (Sociología),
ausencia de imagen paterna y anorexia u homosexualidad (Psicología), etc. Esta
diferencia en las leyes es muy importante porque en las explicaciones
nomológico-deductivas, dándose por verdaderas las leyes de probabilidad máxima que
cubren la totalidad de los casos, puede inferirse el explanandum del explanans como
una conclusión deductiva. En cambio, como en el explanans del modelo
estadístico-inductivo, las leyes probabilísticas no cubren la totalidad de los casos -hay
un cono de sombra en donde están los que no siguen la ley-, aun siendo este
explanans verdadero, no hay garantía de verdad para el explanandum. Esta situación
es la que se indica con la doble raya que antecede al explanandum en el esquema de
las explicaciones estadístico-inductivas (EI).
L1,L2,L3..................Ln

ACTIVIDAD 1. ACERCA DE LOS DISTINTOS MODELOS DE EXPLICACIÓN


En los siguientes ejemplos, determine qué modelo de explicación podría aplicarse en
cada caso y justifique su elección:
a. El enfermo se curó porque lo trataron con penicilina.
b. Juan contrajo hepatitis por usar jeringas contaminadas.
c. María compró un auto para dormir dos horas más a la mañana.
d. Ernesto chocó con el auto porque manejaba alcoholizado.
e. Los huesos sirven para que el cuerpo se mantenga erguido.
f. Las golondrinas emigran en otoño porque necesitan vivir en lugares cálidos.
g. El Papa convocó a un Concilio porque quiere reformar el ritual de la misa.
h. La babosa vive en lugares oscuros y tiene hábitos nocturnos porque no tiene
caparazón.
i. El supermercado X vende la gaseosa por debajo del precio de costo porque quiere
eliminar a la competencia.
j. El corazón late porque la médula determina su ritmo y fuerza.
k. Juan se enfermó de los pulmones porque fumaba mucho.
l. Mi vecino perdió su fortuna porque es adicto a las carreras de caballos.

ACTIVIDAD 2. MODELOS DE EXPLICACIÓN CIENTÍFICA. ACERCA DE LA


EXPLICACIÓN NOMOLÓGICO-DEDUCTIVA
Hemos visto que la explicación nomológico-deductiva se ajusta mejor a los casos de la
Física. En el artículo que se encuentra a continuación, se presenta el especial
comportamiento del agua en relación con la Ley general de expansión térmica (la
materia se dilata cuando se calienta y se contrae cuando se enfría) que presentaría un
comportamiento distinto. Luego de leer atentamente el artículo: a. Señale cuál sería el
explanandum y el explanans en el caso presentado del comportamiento del agua a
distintas temperaturas. b. Determine si el explanandum menciona una regularidad o un
caso particular.

¿Por qué el agua no obedece a la Ley general de expansión térmica?


Con pocas excepciones, las formas de la materia -sólidos, líquidos, gases y plasmas-
se dilatan cuando se calientan y se contraen cuando se enfrían. En los sólidos, por lo
general, esos cambios de volumen no son muy notables, pero existen: las líneas
telefónicas, por ejemplo, se estiran y cuelgan más en un día cálido que en uno de
invierno; los líquidos, por su parte, se dilatan en forma aprecia-ble al aumentar su
temperatura; pero el agua tiene comportamientos notables: a temperaturas cercanas a
su punto de congelación, a menos de 0 grados cuando el agua ya es hielo macizo, su
volumen es bastante mayor y su densidad es menor (por esto el hielo flota en el
agua). Al calentarse el agua, hasta que su temperatura llega a 4 grados, continúa
contrayéndose. Sólo a temperaturas más altas se comienza a dilatar y la expansión
continúa hasta llegar al punto de ebullición.

Revista Muy interesante. N° 290 (2009, diciembre).


ACTIVIDAD 3. LA ESPECIFICIDAD DE LAS CIENCIAS SOCIALES. LA
MODALIDAD EXPLICATIVA COMO CRITERIO DE CLASIFICACIÓN DE LAS
CIENCIAS
Elster plantea que hay diferentes modalidades de explicación: la causal, la funcional y
la intencional. En su libro El cambio tecnológico[21] propone el siguiente esquema que
solamente agregamos a los fines de visualizar la distinción entre conducta intencional y
conducta racional.

En el texto, a continuación, Elster dice que puede haber intencionalidad sin


racionalidad.
A. Lea el texto.
B. Explique con sus palabras por qué puede haber intencionalidad sin racionalidad.
C. Elabore tres ejemplos de conducta intencional-racional.

Intencionalidad y racionalidad según Jon Elster


¿Puede haber intencionalidad sin racionalidad? ¿O racionalidad sin intencionalidad?
[...] Evidentemente todo depende de cómo definimos el concepto de racionalidad. Si
solamente queremos decir "adaptación en el sentido de maximización local", hemos
visto que puede haber racionalidad no intencional. Pero cualquiera sea el modo en que
definimos la racionalidad, creo que debería reservarse para los casos en que tiene
poder explicativo. Es decir que nunca habría que caracterizar una creencia, una acción
o un modelo de conducta como racional a no ser que se esté dispuesto a afirmar que
la racionalidad explica que lo que se dice es racional. [...] Para los propósitos presentes
es suficiente observar que la racionalidad mínimamente implica consistencia de metas
y creencias. Para calzar una cuña entre intencionalidad y racionalidad, debemos
demostrar que puede haber deseos inconsistentes y creencias inconsistentes. Con
respecto a las creencias inconsistentes, demostraré su posibilidad mediante una
historia sobre Niels Bohr, que cierta vez tenía una herradura sobre la puerta. Cuando
se le preguntó si la había colocado allí porque creía que le traería suerte, contestó:
"No, pero me dijeron que traen suerte incluso a quienes no creen en ellas". Arreglando
un poco la historia, resulta lo siguiente:
1) Niels Bohr cree "La herradura no me traerá suerte".
2) Niels Bohr cree "Las herraduras le traen suerte a quienes no creen que les traerá
suerte".
Aunque las creencias entre comillas son consistentes entre sí, ambas no pueden ser
ciertas y ser creídas (por Bohr). Pero un sistema de creencias es consistente solamente
si existe un mundo posible en el que son todas ciertas y creídas. Si a favor del
argumento, suponemos que Bohr no estaba haciendo una broma y que en realidad
colocó una herradura sobre la puerta porque quería suerte y creía aunque
inconsistentemente que le traería suerte, tenemos el caso de una acción claramente
irracional y, sin embargo, explicada intencionalmente.
[22]

ACTIVIDAD 4. DIMENSIÓN EXPLICATIVA Y DIMENSIÓN PREDICTIVA. EL


HOMBRE NO LOGRÓ CUMPLIR SU GRAN ILUSIÓN: PREDECIR EL FUTURO
Después de leer el siguiente texto, realice la actividad propuesta a continuación:
Eduardo Flichman (1932-2005), argentino, fue un físico y filósofo de la ciencia,
dedicado a la investigación y docencia en el ámbito de la epistemología y la enseñanza
de las ciencias. En una nota periodística publicada en el diario Clarín, Flichman dice: "la
inquietud por predecir la conducta humana todavía tiene más interrogantes que
certezas". Para él, si pudiésemos predecir el futuro, si pudiésemos saber con antelación
lo que nos va a pasar, dejaría de tener sentido el concepto de libertad y los hombres
estarían condenados a seguir un destino. Y proporciona un ejemplo: "se pueden
estudiar las constantes en las variaciones del tránsito y determinar que a las 7 de la
tarde se va a ir más rápido por una calle lateral que por la autopista porque ésta última
está colapsada". Pero cuando se dan a conocer los resultados de la investigación, ya
ese conocimiento altera los comportamientos de los conductores y, si todos eligen ir
por las calles laterales, entonces resultará falsa la predicción y allí se irá mucho más
lento y más rápido por la autopista. En ciencias sociales, expresa Flichman, "el
conocimiento modifica las conductas".
Dada esta situación: "Frente a la predicción de una epidemia de gripe A durante el
mes de julio de 2009, en Argentina se tomaron medidas preventivas. La epidemia no
se produjo en los niveles previstos y eso llevó a algunos a pensar que se trató de una
falsa alarma destinada a sembrar miedo en la población y a aumentar la venta de
medicamentos". Responda: ¿El hecho de que no se haya cumplido la epidemia en el
grado previsto, indica que los biólogos se equivocaron en la predicción?

ACTIVIDAD 5. ENFOQUE COMPRENSIVISTA DE LAS CIENCIAS SOCIALES


5.1 Teniendo en cuenta lo estudiado en el enfoque comprensivista de las ciencias
sociales, ubique los términos en las columnas según corresponda. Términos Explicación
Comprensión ¿Por qué?

Hempel

Pluralistas

Popper

Dilthey

¿Cómo?

Positivistas

Descripción
Busca explicar

Teleológico

Monistas

Antipositivistas

Aristóteles

Rechazo de las
ciencias naturales
como patrón metódico

Simmel

Intencionalidad

Galileo

Comte

Historia

Platón

Unidad de método

Hermenéutica

Busca comprender

Droysen

Reducción del método


de las ciencias sociales
a las naturales

Causalidad

Geisteswissenschaften

Empatía

Reduccionismo

Sentido

Vertehen

Weber
Tiempo

Situación

Predicción

Acción

Dray

5.2. Responda el siguiente cuestionario:


a. ¿Qué pregunta se hacen los partidarios de la comprensión en las ciencias sociales y
cuál los de la explicación?
b. Realice un cuadro comparativo entre las posiciones de Hempel, Popper, Dilthey,
Droysen y von Wright.
c. ¿Cuál es la diferencia entre monismo metodológico y pluralismo metodológico?
d. Von Wright sostiene que el problema de la explicación y la comprensión en las
ciencias se remonta a dos tradiciones científicas. ¿Cuáles son esas dos tradiciones y a
qué problema se orienta cada una? Indique, asimismo, con qué corrientes
epistemológicas contemporáneas puede trazar una correspondencia.
e. ¿Por qué motivo los partidarios del enfoque comprensivista rechazan el monismo
metodológico? ¿Qué razones brinda Dilthey al respecto?
f. ¿Quién introdujo la dicotomía metodológica entre "explicación" y "comprensión"?
g. Describa brevemente cuál es la tarea de la comprensión (hermenéutica) como
método científico a partir de lo expuesto por Simmel, Dilthey, von Wright, Weber y
Echeverría.
h. ¿Cuál es la posición de von Wright en esta tensión entre explicación y comprensión?
i. Mencione cuál sería el objetivo de la ciencia para cada una de estas posiciones.
5.3. Dadas las siguientes situaciones:
A. Supongamos que un alumno se copia en un examen y es descubierto por el
profesor. De acuerdo con el modelo comprensivista de von Wright, responda:
a. ¿Qué deberíamos preguntarnos?
b. ¿Cómo daríamos cuenta de ese hecho temporal y causalmente?
c. ¿Cómo puede relacionar este hecho con los conceptos de intención, acción, libertad
y responsabilidad?
d. ¿Se podría brindar una explicación que siempre fuese la misma para ese hecho?

B. Supongamos ahora que una piedra cae desde una montaña y que muere una
persona al ser golpeada por esa piedra. Conteste las mismas preguntas anteriores.

5.4. Dada la siguiente situación:


Supongamos el caso de un conocido hecho: el incendio en el centro comercial de
Paraguay. Allí mueren más de cien personas. Se hace la investigación judicial y se
solicita la opinión de distintos expertos para comprender por qué o cómo murieron
esas personas.
A. De acuerdo con el modelo de van Fraassen, qué respondería...
a. ... el perito médico
b. ... el fiscal que investigó el caso
c. ... un ingeniero
d. ... un urbanista
e. ... la policía
f. ... el abogado defensor
g. ... el resto del público asistente al centro comercial, pero que no fue alcanzado por
el incendio
h. ... el dueño del centro comercial

B. Según van Fraassen, ¿todas esas respuestas son legítimas?, ¿de qué depende su
legitimidad?

5.5. Dada la siguiente situación:


Imagine que usted debe realizar una investigación sobre un autor ya fallecido (filósofo,
epistemólogo, sociólogo, jurista, poeta, arquitecto, economista, etc.) y desea
"comprender" su pensamiento y "explicar" el mismo mediante un escrito monográfico
para que el público también lo conozca, o lo conozca mejor. De acuerdo con el modelo
de la comprensión (hermenéutica) brindado por los aportes de Simmel, Dilthey,
Schültz, Gadamer, Habermas, Vattimo, Ricoeur, responda:
a. ¿Qué debería hacer usted?
b. ¿Qué datos debería investigar y de qué manera?
c. ¿Por qué debería indagar ciertos datos que, en apariencia, no serían útiles?
d. ¿Se llegaría al mismo resultado de su investigación si no recabase toda esa
información? Justifique.

Introducción

En Argumentos y teorías. Aproximación a la epistemología de Carlos Asti Vera y


Cristina Ambrosini, encontramos que: "[...] el criterio que distingue entre ciencias
naturales y sociales es menos firme que el que diferencia a las ciencias formales y a las
fácticas, ya que, a menudo no hay acuerdo acerca de dónde ubicar la línea divisoria
[entre un tipo de ciencia y otro], lo que también afecta a las consideraciones sobre los
métodos a emplear." (2009, 189) Esta cuestión de la metodología de las ciencias
fácticas comienza a presentarse como problemática hacia mediados y fines del siglo
XIX y principios del XX con las consideraciones realizadas por Johann Gustav Droysen
(historiador alemán, 1808-1884) y, fundamentalmente, por Wilhelm Dilthey (filósofo
alemán, 1833-1911). Hasta ese entonces se entendía que el método de justificación,
modelo en las ciencias fácticas, era el correspondiente a las ciencias naturales (el
inductivista y posteriormente, ya en el siglo XX, el hipotético-deductivo), y en ese
sentido, las ciencias sociales se subsumían bajo dicho modelo. Esta concepción, en
líneas generales, se la conoce bajo la denominación de monismo metodológico.[23] La
problemática de la distinción (pluralismo metodológico)[24] o no (monismo
metodológico) entre los métodos de la ciencias fácticas trae aparejada la cuestión de si
estas ciencias (tanto naturales como sociales) explican -causalmente-o describen -para
comprender. Y aquí nuevamente nos encontramos con estas dos posiciones
epistemológicas: monismo y pluralismo. Al inicio del capítulo 6 de Asti Vera y Ambrosini
(2009. 214), se indica que en las ciencias fácticas hay una diferencia entre explicar y
describir, dando cuenta de que la primera responde a la pregunta del "porqué" y la
segunda a la pregunta del "cómo". Así, si nos preguntáramos cuál sería el objetivo de
la ciencia: ¿explicar o describir?, la posición mayoritaria dentro de la comunidad
científica se inclinaría por la afirmación que la ciencia debe explicar y no describir,
incluso frente a las diferencias existentes entre ciencias naturales y ciencias sociales o
humanas o del espíritu. Por otra parte, el campo del conocimiento social estuvo
dominado por el empirismo lógico hasta la aparición de un conjunto de teorías
epistemológicas que en los últimos treinta años cambiaron el panorama rechazando la
idea de que pudiera haber observaciones teóricamente neutrales. Este distanciamiento
con el criterio de neutralidad en el campo de la ciencia natural permitió que
emergieran con renovada vitalidad tradiciones ya existentes como la fenomenología y
la hermenéutica; esta última tal como la desarrollaron Hans-Georg Gadamer (filósofo
alemán, 1900-2002) y Paul Ricoeur (filósofo francés, 1913-2005) siguiendo y
recreando a Dilthey. Y por otra parte, adquirió cierta relevancia la teoría crítica
representada por Jürgen Habermas (filósofo alemán, n. 1929). Lo más importante de
la situación de la epistemolgía de los últimos años es la consideración de la ciencia
como una actividad interpretativa y, entonces, los problemas de significado y
comunicación adquieren relevancia en la epistemología. 1. El problema de la
"explicación" en las ciencias fácticas

Particularmente respecto del problema de la explicación en las ciencias sociales, Carl


Hempel (filósofo, epistemólogo y lógico germano-estadounidense, 1905-1997) en La
explicación en la ciencia y en la historia (1981), sostiene que los dos tipos básicos de
explicación científica, el modelo nomológico-deductivo y el modelo
estadístico-inductivo, sirven de soporte al modelo de explicación característicamente
histórico. Y defiende que la explicación histórica aspira, además, a demostrar que los
hechos no ocurren por azar, sino que podía esperarse su ocurrencia en vista de ciertos
antecedentes o condiciones simultáneas (von Wright, 1979. 44) y si no podemos
formular leyes generales en las explicaciones históricas, eso se debe a la excesiva
complejidad de tales leyes y a la insuficiente imprecisión con que las conocemos. De
esta manera, las explicaciones históricas constituyen esbozos de explicación (von
Wright, 1979. 45). Algo similar sostiene Karl Popper (sociólogo y espistemólogo
austro-británico, 1902-1994), representante también del modelo de cobertura legal
(nomológico-deductivo), quien afirma que la razón de que las leyes no sean
formuladas en las explicaciones históricas es que ellas son demasiado triviales para
merecer una mención explícita, y que las damos implícitamente por supuestas (von
Wright, 1979. 45). Sin embargo, la cuestión acerca de si la ciencia, y particularmente
las ciencias sociales, deben explicar o describir no está zanjada. Respecto de esta
cuestión, Georg Henrick von Wright (filósofo finlandés, 1916-2003), en su obra
Explicación y Comprensión (1979), emprende el tratamiento de la explicación en la
historia y en las ciencias sociales, y analiza las diferencias en los métodos explicativos
entre las ciencias humanas y las ciencias naturales. Desde el punto de vista
metodológico, se suelen señalar dos actitudes con relación al problema del método, lo
que se llama monismo metodológico (hay un solo modelo tanto para las ciencias
naturales como para las sociales), y lo que se denomina, sensu contrario, pluralismo
metodológico. En este sentido, en la base de la discusión entre explicación y
comprensión (descripción) está la cuestión de dónde reside el origen de la
controversia. Hempel indica que los factores que han estimulado la investigación
científica son fundamentalmente dos: "[...] Una es el deseo persistente del hombre por
mejorar su posición estratégica en el mundo por medio de métodos confiables para la
predicción y, cuando sea posible, el control de los acontecimientos. [...] Pero además,
una segunda motivación es la insaciable curiosidad intelectual del hombre [...]"
(Hempel, 1981).

1.1. ORIGEN DE LA CONTROVERSIA


Recordemos que la dicotomía existente entre explicación y comprensión hunde sus
raíces en la historia de las ideas y se relaciona con el problema de si la construcción
teórica es intrínsecamente un mismo género de empresa tanto en las ciencias
naturales como en las ciencias sociales, humanas o del espíritu. von Wright sostiene
que el problema parte de dos tradiciones científicas en la historia de las ideas. Una es
la aristotélica y la otra la galileana que se remonta a Platón. La primera se vincula a los
esfuerzos del hombre por "comprender" las cosas "teleológicamente", es decir,
comprender las cosas como tendientes a un fin, a un objetivo; y la segunda por
"explicarlas causalmente" (von Wright, 1979. 18). Y considera que cuando el filósofo se
cuestiona el tipo de conocimiento, se encuentra con estas dos tradiciones que difieren
en el planteamiento de las condiciones a satisfacer por una explicación científica. En el
mismo sentido, en la historia de la ideas, y particularmente de las ciencias, nos
encontramos frente a dos corrientes epistemológicas fuertes, la positivista y la
antipositivista. Y según von Wright, la positivista, cuyos representantes principales son
Auguste Comte (sociólogo francés, 1798-1857) y John Stuart Mill (filósofo y
economista inglés, 1806-1873), queda vinculada a la tradición galileana a través de su
monismo metodológico, y por otro lado, la antipositivista, cuyas figuras más
significativas son Dilthey y Max Weber (filósofo, economista y sociólogo alemán,
1864-1920), entre varios otros, representa una tendencia mucho más diversificada y
heterogénea que el Positivismo, rechazando el monismo metodológico y rehusándose a
establecer como patrón metodológico a las ciencias naturales exactas como ideal
regulador único y supremo de la comprensión racional de la realidad. Para caracterizar
a esta última corriente, von Wright utiliza el nombre de hermenéutica -cuestión que
trataremos en detalle en los próximos parágrafos- y la vincula con la tradición
aristotélica (von Wright, 1979. 23-24).

1.2. EL PROBLEMA DEL MÉTODO Y EL REDUCCIONISMO


Antes de incursionar en las concepciones de la corriente "comprensivista" y en su
enfoque hermenéutico, hay que tener en cuenta que, por un lado, desde el
Positivismo, se sostiene la unidad de método tendiendo a subsumir el método de las
ciencias sociales al de las naturales; estableciéndose como ideal metodológico las
ciencias físico-matemáticas; poniéndose énfasis en la explicación y predicción; dando
cuenta de que la explicación es causal; y rechazando a las explicaciones finalistas o
teleológicas como acientíficas. Y mientras, por otro lado, el antipositivismo, la otra
posición, tiende a sostener un contraste entre, por una parte, las ciencias que al modo
de la física, la química o la fisiología, aspiran a generalizaciones sobre fenómenos
reproducibles, y por otro parte, las ciencias que, como la historia, buscan
"comprender" las peculiaridades individuales y únicas de sus objetos, impugnando el
enfoque positivista de la explicación. Las ciencias sociales o humanas no pueden,
sostiene Dilthey, pretender la comprensión de la vida a través de categorías externas a
ella -como lo pretende el Positivismo-, sino a través de categorías intrínsecas,
derivadas de ella misma, por tal motivo no deben explicar, sino comprender y no lo
pueden hacer mediante los métodos de las ciencias naturales sino mediante los suyos
propios. Es decir, para "explicar" (entre comillas porque Dilthey utiliza "comprender") la
vida, para comprender las estructuras del sentido de la vida, no podemos usar
categorías, conceptos de las ciencias naturales como leyes lógicas o razonamientos
deductivos o inductivos, porque todo esto son construcciones formales realizadas por
el hombre, y no son "la vida", es decir, no pertenecen a la categoría de la vida esas
categorías científicas, ya que son sólo objetivaciones que realiza el hombre para
explicarla, pero no son categorías propias de ella. Las categorías propias de la vida
tienen que ver con lo que Dilthey llama "vivencias" y que son, a grandes rasgos, lo que
nos ocurre, y lo que nos ocurre históricamente. Son los acontecimientos de la vida, el
querer, el sentir y el representar. Y por eso, para Dilthey, la comprensión en las
"ciencias del espíritu" se da porque la vida se exterioriza en textos, que dan cuenta de
las vivencias del que lo escribió y, entonces, nos permiten comprender, además del
texto, a su autor.
2. La "comprensión" en las ciencias sociales
La corriente antipositivista, por su parte y de diversas maneras, sostiene que el
objetivo de las ciencias naturales es "explicar", y el de la historia y las ciencias sociales,
es "comprender" los fenómenos que ocurren en su ámbito; y es el mismo Droysen el
que introdujo tal dicotomía metodológica y acuñó los nombres de explicación y
comprensión. Estas ideas metodológicas fueron luego desarrolladas con mayor
profundidad y sistematizadas por Dilthey, quien rechaza la tendencia de fundar un
conocimiento humano siguiendo los procedimientos de las ciencias naturales. Es el
mismo Dilthey quien para designar todo el dominio de aplicación del método de
comprensión se sirvió del término "ciencias del espíritu" (Geisteswissenschaften) que
conocemos también por ciencias humanas o ciencias sociales, y en el habla inglesa por
Moral science (von Wright, 1979. 24). El uso cotidiano del lenguaje no hace una
distinción aguda entre los términos "explicar" y "comprender", y así se podría decir
que, cualquier tipo de explicación proporciona una comprensión de las cosas. Pero
"comprensión" cuenta además con una connotación psicológica de la que carece el
término "explicación". Se considera que la comprensión, como método característico de
las ciencias del espíritu, es según Simmel (filósofo y sociólogo alemán, 1858-1918) una
forma de empatía o recreación en la mente del estudioso de la atmósfera espiritual,
pensamientos, sentimientos y motivos, de sus objetos de estudio (von Wright, 1979.
24). Sin embargo, no es esta la única característica que distingue a la comprensión de
la explicación. Von Wright sostiene que la comprensión se encuentra ligada además
con la "intencionalidad" de una manera en que la explicación no lo está. Se
comprenden los objetivos y propósitos de un agente, el significado de un signo o de un
símbolo, el sentido de una institución social, de un rito religioso. Esta dimensión
intencional, sostiene el mismo von Wright, ha llegado a jugar un papel relevante en la
discusión metodológica más reciente a través de la hermenéutica (von Wright, 1979.
24-28). El gran objetivo de Dilthey consistió en desarrollar una metodología apropiada
para el "entendimiento" (Verstehen) de las obras humanas, que eluda el reduccionismo
y mecanicismo de las ciencias naturales, ya que la vida debe ser entendida a partir de
la propia experiencia de vida. En tal sentido es central para Dilthey esta distinción
entre ciencias naturales y ciencias del espíritu. Las primeras descansan en el concepto
de "fuerza" propuesto por la física y en las matemáticas; las ciencias humanas, en
cambio, se apoyan en el concepto de "sentido" y en la historia. Y así, el concepto clave
de las ciencias del espíritu es el de "entendimiento" o "comprensión" (Verstehen). Si en
sociología (ciencia social), Comte era el principal representante del positivismo, Weber
se inclinaba hacia el lado opuesto y de una manera más sólida que Comte. Uno de los
elementos que aparecen en la concepción weberiana del método científico es el de
"comprensión" (Verstehen). Esta comprensión no significaba una identificación
completa con el sujeto investigado, sino que, para comprender, por ejemplo las obras
de Shakespeare, yo no necesito ser Shakespeare, pero sí tengo que poder de alguna
manera concebir qué puede estar pensando Shakespeare en una determinada
situación. Así, comprender es la capacidad que tiene cualquier ser humano de entender
a otro ser humano en una situación, aunque él no haya vivido exactamente esa
situación. Es, aunque no en el más completo sentido, lo que comúnmente conocemos
por "ponerse en el lugar del otro" para comprender una actitud, una acción, etc. El
sentido propio de la comprensión (o entendimiento) siempre se halla en un contexto
de horizonte que se extiende hacia el pasado y el futuro. La historicidad y la
temporalidad son dimensiones inherentes e inevitables de toda comprensión. Por
ejemplo, si quisiéramos comprender a Sócrates y su pensamiento, lo que deberíamos
hacer es tender un puente con el pasado y con el presente, es decir, no podríamos
comprenderlo plenamente si sólo lo analizáramos desde la actualidad, pero tampoco lo
podríamos comprender plenamente si sólo lo analizáramos en su contexto, con lo cual
hay que tender un puente entre pasado y presente, un puente que nos permita abrir
las posibilidades del pensamiento de Sócrates. De esta manera, el todo recibe su
sentido de las partes y las partes sólo pueden comprenderse en relación con el todo,
conformando como diría Gadamer un círculo hermenéutico. Como sucede con una obra
literaria, no podemos comprenderla leyendo sólo un capítulo (una parte), pero
tampoco podemos comprenderla completa si cada capítulo no le da un sentido
unificador a la obra. El sentido es histórico, dinámico y contextual, es siempre parte de
una determinada situación. La interpretación siempre remite a la situación en la cual se
halla el intérprete, pero esto no significa que el sentido sea subjetivo y nos lleve al
relativismo; el sentido es una percepción de una relación real dentro de un nexo
anterior a la separación sujeto-objeto en el pensamiento (Echeverría, 1993. 203-205).
La tradición antipositivista procura sustituir las nociones científicas de explicación,
predicción y control por las interpretativas de comprensión, significado y acción. Por su
parte, von Wright sostiene que la metodología de las ciencias humanas comenzó
gradualmente a atraer la atención de los filósofos analíticos[25] (positivistas y
neopositivistas vinculados por la pretensión de excluir por completo a la metafísica de
las ciencias), y que la filosofía analítica de la ciencia se vio inmersa en el campo de
batalla de la metodología positivista y antipositivista (von Wright, 1979. 29). Es así que
hasta aproximadamente los años 60 reinó una coincidencia general en cuanto a que el
modelo explicativo causal suministraba el marco de referencia idóneo para el estudio
de los fenómenos sociales, pero luego, y hasta cerca de 1980, la alternativa
interpretativa, el modelo de la comprensión, comenzó a ganar adeptos en los países de
habla inglesa. Los trabajos de varios filósofos de raigambre analítica, positivista y
neopositivista, reflejan el creciente interés, en el seno de la filosofía analítica, por el
concepto de acción que está vinculado con la intencionalidad. Tal es así que, William
Dray (canadiense, filósofo de la historia, n. 1921), en Leyes y Explicación en Historia,
expresa que las explicaciones de la historia son científicas y racionales, a diferencia de
lo que sostenía Hempel, aunque no son ciencia de leyes generales. Dray sostiene que
la historia explica y lo hace en el sentido de mostrar que una acción fue el proceder
adecuado teniendo en cuenta los antecedentes. Así aborda la pregunta de ¿cómo fue
posible? y no la pregunta de ¿por qué? Von Wright indicará, entonces, que este giro
del "porqué" al "cómo" lo sitúa cerca de la tradición hermenéutica (von Wright, 1979.
45-52). De esta manera, en virtud del interés de algunos epistemólogos positivistas por
la explicación en las ciencias sociales, von Wright trata de tender un puente entre
"explicación" y "comprensión" demostrando que, tanto como sostenían Hempel, por un
lado, o Dilthey, por el otro, no son incompatibles, sino complementarias. Influenciado
por las consideraciones filosóficas del segundo Ludwig Wittgenstein (filósofo y lingüista
austro-británico, 1889-1951) y la filosofía neowittgensteniana, y también por la
hermenéutica, von Wright postula que la filosofía analítica es afín a la hermenéutica a
partir de los conceptos de significado, intencionalidad, interpretación y comprensión, y
por compartir, además, la preocupación por la metodología y la filosofía de la ciencia.
Sostiene en este sentido que la comprensión en el lenguaje de la filosofía
hermenéutica debería diferenciarse de la empatía, afirmando que se trata de una
categoría antes semántica que psicológica (von Wright, 1979. 53). Así von Wright
indica que las explicaciones causales son típicamente cuasi-teleológicas: explicaciones
susceptibles de formulación en términos teleológicos pero cuya validez depende de la
efectividad de conexiones nómicas (leyes). Las explicaciones de este género suelen
responder a las preguntas sobre ¿cómo es? o ¿cómo llega a ser posible algo? Y estas
"explicaciones" tienen un lugar predominante en las ciencias humanas, permitiendo
"comprender" lo que algo es o cuál es la razón de que algo ocurra (von Wright,
1979.108-109). Señalando de este modo que los acontecimientos que resultan de
acciones básicas tienen lugar cuando los investimos de intencionalidad. 2.1. EL
MODELO COMPRENSIVISTA DE VON WRIGHT
De acuerdo con las diversas posiciones epistemológicas y metodológicas que antes
tratamos, podemos decir que la noción de causalidad vale con más facilidad para el
ámbito de los fenómenos naturales que en el de los humanos. Sin embargo, von
Wright sostiene que si rastreamos el concepto de causalidad, veremos que se origina
en nuestro conocimiento inmediato, íntimo, de la acción humana. Es decir, este
concepto proviene de una proyección de nuestras propias acciones sobre el mundo
natural. Nosotros, cultural o históricamente, experimentamos la acción, y la noción de
causa surge por una especie de proyección de nuestras propias acciones al mundo
natural. Así, si nosotros no actuáramos, no tendríamos el concepto de causa. De esta
manera, en lugar de pensar que yo soy la causa por la cual, supongamos, "x" murió
por mi acción o mi acción fue la causa de la muerte de "x"; la causalidad en la
naturaleza deriva del "saber que yo puedo" matar a una persona. O sea, invierte el
orden con respecto a la idea que previamente uno podía tener. Todos los comentarios
de von Wright están dirigidos al problema de mostrar que la conducta humana es algo
completamente distinto de los hechos que nos encontramos en el mundo natural, y
hasta formula hipótesis de cómo en el ámbito humano puede haber procesos causales
de tipo retroactivo, en los que el efecto es anterior a la causa. Por ejemplo, se piensa
que un cierto proceso cerebral tiene que ser de alguna manera anterior a un
movimiento muscular, porque se supone que el cerebro ordena al brazo, por ejemplo,
a extenderse, pero von Wright piensa que es al revés. Es el levantar el brazo, el
extenderlo, lo que produce el fenómeno cerebral, pero lo produce antes. Es decir, no
es que la causa de que apreté el dedo haya sido el fenómeno cerebral, sino que el
fenómeno cerebral se produce porque "voy a" apretar el dedo, o "a" extender el brazo.
Y sostiene que el brazo se extiende, o el dedo se aprieta porque "yo decidí hacerlo". Yo
lo que decido es extender el brazo, es una decisión que tomo, decido extender el brazo
y lo extiendo. Con lo cual, en el momento que decidí hacerlo, y lo hago efectivamente,
ese hecho, para von Wright, es primario causalmente, no temporalmente, y no lo es
temporalmente porque él reconoce que no hubiera extendido el brazo si no hubiese
producido un cierto estado cerebral antes. Así, causalmente, primero está la decisión,
segundo, el proceso cerebral y tercero, la acción. Pero temporalmente, primero está el
proceso cerebral y segundo la acción. Esta cuestión se emparenta directamente con el
tema de la "intencionalidad" y la "responsabilidad". Un ser humano directamente
levanta el brazo, aprieta el botón, etc., y al hacer esto, él es responsable de su acción,
y sólo podrá ser responsable si la decisión la toma él. Y esto es lo que le lleva a decir
que el fenómeno A (proceso cerebral) se da antes que B (apretar el dedo), pero B
(apretar el dedo) es la causa de A (proceso cerebral). Un concepto fundamental en von
Wright es el concepto de acción. La caracterización de este concepto consiste en la
producción de cambios que no hubieran tenido lugar de no haber mediado nuestra
intervención. Por ejemplo, en el caso de un crimen, acusamos a una persona de
haberlo cometido porque no habría tenido lugar la muerte de la otra persona de no
haber mediado la acción de quien, supongamos, accionó el gatillo. Pero una acción es
tanto la acción propiamente dicha como la omisión, que es una acción negativa. Así, la
"explicación" en el caso de las acciones humanas es completamente distinta de la
explicación de los fenómenos de tipo natural, porque en el caso de las acciones no voy
a buscar leyes, y, en el caso de los fenómenos naturales, no puedo adjudicarles ni
intención, ni decisión, ni responsabilidad. von Wright sostiene entonces que lo más
adecuado para "comprender" las acciones humanas no es el esquema de Hempel
(leyes), sino el de Aristóteles (interpretación), que es el esquema que se conoce como
silogismo práctico, esquema sobre las explicaciones intencionales que ya se ha visto en
la bibliografía.[26] De esta manera, es posible afirmar que, en el modelo de "explicación
comprensivista" de von Wright decir que el concepto de causalidad supone el de
libertad es correcto en el sentido de que únicamente a través de la idea de hacer cosas
se llega a "comprender" las ideas de causa y efecto (von Wright, 1979. 106).
2.2. EL MODELO PRAGMÁTICO DE VAN FRAASSEN
Ya se había anticipado que los modelos de explicación en ciencias sociales se
vinculaban fundamentalmente con el concepto de comprensión de origen
hermenéutico, y habíamos indicado que en la comprensión eran inescindibles nociones
tales como intencionalidad y contexto, como también las de temporalidad, significado e
interpretación. Precedentemente se ha tratado la cuestión del "modelo de
comprensión" vinculado a la intencionalidad, lo que se denominó el modelo
comprensivista de von Wright. Se iniciará ahora el recorrido al modelo pragmático de
Bas van Fraassen (filósofo holandés, n. 1941), vinculado fundamentalmente con el
contexto, y aunque este epistemólogo no se vincule directamente con el
comprensivismo, podemos observar ciertas semejanzas con esa posición metodológica.
Van Fraassen considera los problemas epistemológicos más allá, no exclusivamente
desde el punto de vista del análisis de las relaciones lógicas y relaciones conceptuales
(dimensión sintáctica y semántica), sino tomando en cuenta lo que se denomina la
dimensión pragmática. Recordemos que la dimensión pragmática toma en cuenta los
usuarios o las circunstancias en las cuales se usan las expresiones. Con Dilthey
decíamos que el sentido era contextual, y la interpretación se remite siempre a la
situación en la cual se halla el intérprete. La circularidad del entendimiento (círculo
hermenéutico) tiene otra importante consecuencia, no existe realmente un punto de
partida verdadero para el entendimiento. Ello significa que no es posible concebir un
entendimiento carente de presupuestos. Todo acto de entendimiento, de comprensión,
tiene lugar en el interior de un determinado contexto u horizonte. Ello es igualmente
válido para las explicaciones científicas. Estas siempre requieren de un marco de
referencia. Así, comprendemos sólo por referencia a nuestra experiencia. Van Fraassen
piensa que es inútil hacer una caracterización de las explicaciones que no tenga en
cuenta las situaciones en las cuales las explicaciones se producen, es decir, su
contexto. Así, si yo pregunto ¿por qué (algo que Hempel diría que genera una
explicación) Adán comió la manzana?, por lo pronto, el tipo de respuesta que voy a
admitir como adecuada tiene que ver con determinado contexto. Así, una explicación
que podría ser perfectamente válida para un contexto, podría estar fuera de lugar en
otro. Lo que sostiene van Fraassen es que una pregunta, en primer lugar, siempre se
refiere a un tópico, y el tópico será el explanandum. Y así, la propia suposición de que
el explanandum es verdadero depende de una cuestión contextual. Por otro lado, van
Fraassen sostiene que puede haber respuestas distintas a la misma pregunta de "¿por
qué?" e incluso puede haber respuestas completamente distintas, pero todas válidas,
cada una en relación con un contexto. Por ejemplo, en un accidente automovilístico
murió Juan X. Se hace una investigación judicial y se solicita la opinión de distintos
expertos para "comprender" cómo o por qué murió Juan X. Las respuestas variarán
dependiendo de quien las responda, así el médico podrá responder "por un paro
cardíaco", el mecánico "porque los frenos no estaban en buenas condiciones y
fallaron", y un urbanista responderá "porque los árboles tapaban el semáforo". Todas
estas respuestas son legítimas según van Fraassen, pero eso no quiere decir que
cualquier respuesta en cualquier contexto sea legítima. Con lo cual, esto significa sólo
que puede haber explicaciones válidas alternativas. Van Fraassen indica que puedo
explicar un hecho A a partir de un hecho B, o puedo hacer al revés, explicar B a partir
de A. Y en esto parece contradecir lo que sostenía Hempel respecto a que la
explicación era unidireccional, es decir, que se va de las leyes a las consecuencias y no
al revés. Pero, en este caso, si uno acepta un determinado contexto, es perfectamente
legítima una explicación y no lo es la otra. Una de las consecuencias que extrae van
Fraassen es que es completamente equivocado pensar que las explicaciones
constituyen uno de los objetivos esenciales de las ciencias, ya que una explicación no
puede ser juzgada como correcta o no sobre la base de las características que Hempel
había tomado en cuenta, relativizando de esta manera el concepto de explicación. A
modo de conclusión

Finalmente podemos decir que el punto crítico se encuentra entonces en los


conceptos de método y objetividad propios de la ciencia moderna. Actualmente, desde
la epistemología comprensivista podemos destacar los aportes de: a) Alfred Schütz
(sociólogo y filósofo austríaco, 1899-1959), para quien la comprensión es la
reconstrucción de las perspectivas que los individuos tienen de la sociedad. La acción
social es entendida como una vivencia guiada por un plan o proyecto. La interpretación
surge entonces de los motivos del actor (Schütz, 1932), pero el límite es la
imposibilidad de generalizar y predecir. b) Hans-Georg Gadamer aplica el método
al análisis de textos históricos, y sostiene la naturaleza histórica de la comprensión
misma. Sostiene que hay una tensión entre la objetividad del texto y la subjetividad del
intérprete, y la forma de resolver esta tensión es mediante la fusión de horizontes,
[27]
es decir, lograr un acuerdo de ajuste de nuestros prejuicios (Gadamer, 1981).
Según Gadamer, la etimología del término "hermenéutica" significa explicación, y se
trata de explicar unos enunciados analizándolos mediante otros enunciados. Es, pues,
el arte o la teoría de la interpretación (Gadamer, 1998). Para el filósofo alemán, "[...]
la hermenéutica no constituye un método determinado que pudiera caracterizar por
ejemplo a un grupo de disciplinas científicas frente a las ciencias naturales. La
hermenéutica se refiere más bien a todo el ámbito de comunicación infrahumana [...]"
(Gadamer, 1998. 85).
c) Jürgen Habermas, por su parte, destaca la capacidad de persuasión como
estrategia para la dominación a partir de la comprensión de las significaciones dadas
en la acción comunicativa. La comprensión es entendida como experiencia
comunicativa que implica una actitud reflexiva y la posibilidad no sólo de intercambios
de mensajes, sino también para cambiar y formar actitudes. Así, la hermenéutica se
vincula con la retórica, con la argumentación de tipo informal. En el mismo sentido,
Gadamer sostiene que "el gran legado de la retórica sigue influyendo en puntos
decisivos en relación con la nueva labor de interpretación de los textos". De manera
que retórica y hermenéutica son profundamente afines, y particularmente, por
ejemplo, las clases de retórica de Melanchthon
[28]
(teólogo alemán, 14971560) ejercieron una influencia determinante en la
configuración del sistema escolar (Gadamer, 1998. 271-272).
d) Paul Ricoeur, en Del texto a la acción (1969), también problematizará el intento de
resolver la aporía central de la hermenéutica, es decir, la alternativa entre explicación y
comprensión, aporía que a su juicio es infundada. Él buscará la complementariedad de
las dos actitudes, dado que la objetivación del discurso en la obra y el carácter
estructural de la composición, a lo cual se agregará el distanciamiento mediante la
escritura, lo obliga a poner en tela de juicio la oposición recibida de Dilthey entre
comprender y explicar. Ahora bien, sería ciertamente ilusorio, sostiene von Wright,
creer que la verdad reside inequívocamente en una de estas dos posiciones opuestas
(explicacionista-comprensivista), sino que existe una confrontación de base, y ésta se
funda en la elección de conceptos primitivos básicos para la argumentación en
conjunto. Podría calificarse esta elección de "existencial" consistiendo en la opción por
un punto de vista no susceptible de ulterior fundamento (von Wright, 1979. 57). Sin
embargo, se puede considerar desde Dilthey, que las ciencias del espíritu y, por ende,
el método de comprensión, son epistemológicamente anteriores a las de la naturaleza,
a las que por lo demás abarcan, pues toda ciencia natural es también un producto
histórico. Pero como ya hemos dicho anteriormente, la cuestión aún no está zanjada, y
sigue en pie precisamente por ser un problema filosófico.
Temas de la Unidad
6.1. La Epistemología del siglo XX: problemas fundamentales
6.2. La concepción epistemológica del Positivismo lógico
6.3. Las perspectivas falsacionistas: Popper y Lakatos
6.4. La ruptura: del racionalismo de justificación al análisis de la historia de la ciencia
6.5. El pensamiento de Kuhn en perspectiva

Bibliografía obligatoria Unidad 6 en la Guía de Estudio de IPC, producida por UBA


XXI y editada por Eudeba, a partir de la edición de 2010. Asti Vera, C. y Ambrosini, C.
(2009). Capítulo 7. En Argumentos y teorías. Aproximación a la Epistemología. Buenos
Aires: Educando.
BIBLIOGRAFÍA COMPLEMENTARIA
Echeverría, J. (1999). Capítulos 1, 2, 3, 4, 5 y 6. En Introducción a la metodología de
la ciencia. La filosofía de la Ciencia en el siglo XX. Madrid: Cátedra. Klimovsky, G.
(1994). Capítulos 21, 22 y 23. En Las desventuras del conocimiento científico. Buenos
Aires: AZ.
Presentación de la Unidad
Creo que podrían ustedes hacer algo más útil para matar el tiempo que malgastarlo
con adivinanzas que no tienen solución".

"¡Ay! ¡Si conocieras al Tiempo tan bien como lo conozco yo", exclamó el Sombrerero,
no hablarías de malgastarlo y menos de matarlo! Se trata de un tipo de mucho cuidado
y no de una cosa cualquiera."

"Me parece que sigo sin comprenderle", dijo Alicia.

"¡Naturalmente que no me comprendes!", dijo el Sombrerero elevando


orgullosamente la nariz. "Con toda seguridad; ni siquiera habrás hablado con el
Tiempo!"

"Puede que no", contestó Alicia con cautela. "Pero sí sé", añadió esperanzada, "que
en las lecciones de música marco el tiempo a palmadas".

"¡Ah!¡Ah!¡Eso lo explica todo!", afirmó el Sombrerero. "El Tiempo no tolera que le den
de palmadas [...]"
(Carroll, L. Alicia en el país de las maravillas)

Esta Unidad 6 corresponde a la lectura del capítulo 7 de la bibliografía y supone un


enfoque metateórico ya que aquí se señalan algunos de los aspectos centrales de la
Epistemología contemporánea, y reitera algunos de los problemas abordados en las
unidades anteriores desde un ángulo diferente. La expresión "la concepción heredada",
mencionada en este capítulo, para referirse básicamente a la epistemología del Círculo
de Viena, se adjudica al filósofo estadounidense Hilary Putnam (n. 1926) y se identifica
con las críticas a esta epistemología caratulada también como "cientificista".
Encontrará en el capítulo, una mención a los aportes del psicólogo suizo Jean Piaget
(1896-1980) identificando su posición con la llamada Epistemología genética y algunos
representantes de la filosofía francesa que acentúan el importante papel de la historia
de la ciencia en el análisis epistemológico. Para hacer explícitos los postulados
filosóficos del llamado Positivismo lógico, se centra la atención en los postulados del
Círculo de Viena y la extendida influencia que ejercieron sus miembros sobre otros
grupos de científicos y epistemólogos de la época. Un punto central dentro de los
temas tratados, y sobre el que ya se presentaron varios aspectos, es la posición
falsacionista a través de las figuras de Popper y de Imre Lakatos, tomando en cuenta
sus ideas acerca de la producción de teorías científicas aludiendo a la posición del
primero como "realismo crítico" y, al segundo, a partir de su propuesta de los
"programas de investigación". Deberá detenerse especialmente en estos párrafos, en
la presentación de las ideas fundamentales de la epistemología de Thomas Kuhn que
pueden ser vistas como una ruptura respecto a las posiciones anteriores, enfocadas en
la búsqueda del método científico (monismo metodológico) y en el análisis de la
estructura interna de las teorías científicas para interesarse por la historia de la ciencia
y el proceso de génesis de las teorías científicas. Las nociones centrales de
"paradigma", "ciencia normal", "revolución científica", propias de esta posición,
muestran un quiebre y una transformación de la concepción de la epistemología
centrada en el análisis lógico y en las estructuras formales de las teorías. Luego de la
presentación de la epistemología de Kuhn y el señalamiento de algunas de las críticas
de las que ha sido objeto, se señala un panorama del complejo entramado de teorías
en las que se mencionan distintas estrategias para el análisis de las teorías científicas.
Debemos advertir que estos temas no están incluidos como temas del programa y no
son de lectura obligatoria. A pesar de ello, como en otros casos, recomendamos su
lectura para tener una comprensión cabal de las posiciones presentadas.

Actividades de aprendizaje
ACTIVIDAD 1. LA CONCEPCIÓN HEREDADA DE LA CIENCIA
Uno de los puntos más polémicos de la posición de Popper, en especial respecto al
monismo metodológico y a la propuesta de considerar "científico" un mismo método
para todo tipo de ciencias, aun en las ciencias sociales, es el estricto criterio de
demarcación que propone entre "ciencia" y "pseudociencia" a partir de considerar el
criterio falsacionista como el único válido para legitimar el conocimiento científico. En
sus análisis de las teorías científicas, haciendo lugar con esto a un debate todavía
vigente, Popper llegó a la conclusión de que dos teorías consagradas en el campo de
las ciencias sociales (el Marxismo y el Psicoanálisis freudiano) eran suficientemente
flexibles como para acomodarse a cualquier situación histórica o de comportamiento
humano, dando la falsa sensación de que explicaban los hechos. Popper advierte que
una teoría que parece explicarlo todo en realidad no explica nada. Según Popper, una
teoría con una capacidad explicativa genuina hace predicciones "arriesgadas", que
excluyen la mayor parte de posibles resultados. El éxito en la predicción es impactante
sólo hasta donde el fracaso sea una verdadera posibilidad. Popper compara esta
situación con la forma, completamente diferente, en que la comunidad científica
planteó la comprobación de la Teoría General de la Relatividad de Einstein. La teoría
predecía que la luz, al propagarse, debería sentir la acción de la gravedad como
cualquier forma de energía y, en consecuencia, su trayectoria debería curvarse al pasar
cerca de un cuerpo de masa elevada, como es el caso del Sol. En consecuencia, un
observador que apuntase su telescopio a una estrella, la vería en posiciones distintas si
lo rayos de luz pasaban cerca del sol que si éste no se encontraba en su camino. En
1919, el astrónomo británico Arthur Stanley Eddington (1882-1944) llevó a cabo este
experimento durante un eclipse y comprobó que los resultados estaban de acuerdo con
las predicciones de Einstein y, en consecuencia, la Teoría General de la Relatividad fue
aceptada. Lo verdaderamente importante, en términos de Popper, no era el hecho de
que la observación de Eddington implicara la validez de la Teoría General de la
Relatividad, sino que la Teoría General de la Relatividad era susceptible de ser
comprobada. Existía un procedimiento bien establecido, de acuerdo con el cual podía
demostrarse que una teoría dada era falsa. Popper empleó esta posibilidad de
"falibilidad", como característica esencial que servía para separar las creencias,
explicaciones y teorías en dos categorías: científicas y pseudocientíficas. Siguiendo
estas ideas y profundizando en la polémica, el físico y filósofo argentino Mario Bunge
(n. 1919) extrema la tesis central del monismo metodológico en el siguiente artículo y,
en este caso, critica especialmente al Psicoanálisis al ponerlo a la altura de la magia o
la superstición.

A. Lea el artículo de Bunge, a continuación:


Una pseudociencia es un montón de macanas que se vende como ciencia. Ejemplos:
alquimia, astrología, caracterología, comunismo científico, creacionismo científico,
grafología, ovnilogía, parapsicología y psicoanálisis. Una pseudociencia se reconoce por
poseer al menos un par de las características siguientes:
- Invoca entes inmateriales o sobrenaturales inaccesibles al examen empírico, tales
como fuerza vital, alma, superego, creación divina, destino, memoria colectiva y
necesidad histórica.
- Es crédula: no somete sus especulaciones a prueba alguna. Por ejemplo, no hay
laboratorios homeopáticos ni psicoanalíticos. Corrección: en la Universidad Duke existió
en un tiempo el laboratorio parapsicológico de J. B. Rhine; y en la de París existió el
laboratorio homeopático del doctor Benveniste. Pero ambos fueron clausurados cuando
se descubrió que habían cometido fraudes.
- Es dogmática: no cambia sus principios cuando fallan ni como resultado de nuevos
hallazgos. No busca novedades, sino que queda atada a un cuerpo de creencias.
Cuando cambia lo hace sólo en detalles y como resultado de disensiones dentro de la
grey.
-Rechaza la crítica, matayuyos normal en la actividad científica, alegando que está
motivada por dogmatismo o por resistencia psicológica. Recurre pues al argumento ad
hominem en lugar del argumento honesto.
- No encuentra ni utiliza leyes generales. Los científicos, en cambio, buscan o usan
leyes generales.
- Sus principios son incompatibles con algunos de los principios más seguros de la
ciencia. Por ejemplo, la telequinesis contradice el principio de conservación de la
energía. Y el concepto de memoria colectiva contradice la perogrullada de que sólo un
cerebro individual puede recordar.
- No interactúa con ninguna ciencia propiamente dicha. En particular, ni psicoanalistas
ni parapsicólogos tienen tratos con la psicología experimental o con la neurociencia. A
primera vista, la astrología es la excepción, ya que emplea datos astronómicos para
confeccionar horóscopos. Pero toma sin dar nada a cambio. Las ciencias propiamente
dichas forman un sistema de componentes interdependientes.
- Es fácil: no requiere un largo aprendizaje. El motivo es que no se funda sobre un
cuerpo de conocimientos auténticos. Por ejemplo, quien pretenda investigar los
mecanismos neurales del olvido o del placer tendrá que empezar por estudiar
neurobiología y psicología, dedicando varios años a trabajos de laboratorio. En cambio,
cualquiera puede recitar el dogma de que el olvido es efecto de la represión, o de que
la búsqueda del placer obedece al «principio del placer». Buscar conocimiento nuevo
no es lo mismo que repetir o siquiera inventar fórmulas huecas.
- Sólo le interesa lo que pueda tener uso práctico: no busca la verdad desinteresada.
Ni admite ignorar algo: tiene explicaciones para todo. Pero sus procedimientos y
recetas son ineficaces por no fundarse sobre conocimientos auténticos. Al igual que la
magia, tiene aspiraciones técnicas infundadas.
- Se mantiene al margen de la comunidad científica. Es decir, sus cultores no publican
en revistas científicas ni participan de seminarios ni de congresos abiertos a la
comunidad científica. Los científicos, en cambio, someten sus ideas a la crítica de sus
pares: someten sus artículos a publicaciones científicas y presentan sus resultados en
seminarios, conferencias y congresos.
Veamos en un ejemplo cómo obran los científicos cuando abordan problemas que
también interesan a los pseudocientíficos. En 1998 los psicobiólogos J. S. Morris, A.
Ohman y R. J. Dolan publicaron en la célebre revista Nature un trabajo sobre
aprendizaje emocional consciente e inconsciente en la amígdala humana. Ya que este
artículo trata de emociones conscientes e inconscientes, parecería que debiera
interesar a los psicoanalistas. Pero no les interesa porque los autores estudiaron el
cerebro, mientras que los analistas se ocupan del alma: no sabrían qué hacer con
cerebros, ajenos o propios, en un laboratorio de psicobiología. Pues bien, la amígdala
cerebral es un órgano diminuto pero evolutivamente muy antiguo, que siente
emociones básicas tales como el miedo y la furia. Dada la importancia de estas
emociones en la vida social, es fácil imaginar los trastornos de conducta que sufre una
persona con una amígdala anormal, ya sea atrofiada o hipertrófica. Si lo primero, no
reconocerá signos peligrosos. Si lo segundo, será propensa a la violencia. La actividad
de la amígdala cerebral puede registrarse mediante un escáner PET. Este aparato
permite detectar objetivamente las emociones de un sujeto en cada lado de su
amígdala. Sin embargo, tal actividad emocional puede no aflorar a la conciencia. O sea,
una persona puede estar asustada o enojada sin advertirlo. ¿Cómo se sabe?
Agregando un test psicológico a la observación neurobiológica. Por ejemplo, si a un
sujeto normal se le muestra brevemente una cara enojada e inmediatamente después
una cara sin expresión, informará que vio la segunda pero no la primera. ¿Represión?
Los científicos citados no se contentaron con bautizar el fenómeno. Repitieron el
experimento, pero ahora asociaron la cara enojada con un estímulo negativo: un
intenso y molesto ruido «blanco», es decir, no significativo. En este caso, la amígdala
fue activada por la imagen visual, aun cuando el sujeto no recordara haberla visto. O
sea que la amígdala cerebral «sabe» algo que ignora el órgano de la conciencia
(cualquiera que éste sea).
En principio, con el método que acabo de describir escuetamente se podría medir la
intensidad de una emoción. Por ejemplo, se podría medir la intensidad del odio que,
según Freud, un varón siente por su padre. Sin embargo, antes de proceder a tal
medición habría que establecer la existencia del complejo de Edipo. Pero éste no
existe, como lo mostraron las extensas investigaciones de campo del profesor Arthur P.
Wolf condensadas en su grueso tomo Sexual Attraction and Childhood Association
(Stanford University Press, 1995).
Las pseudociencias son como las pesadillas: se desvanecen cuando se las examina a
la luz de la ciencia. Pero mientras tanto infectan la cultura y algunas de ellas son de
gran provecho pecuniario para sus cultores. Por ejemplo, un psicoanalista
latinoamericano puede ganar en un día lo que su compatriota científico gana en un
mes. Lo que refuta el refrán: "no es oro todo lo que reluce".
[29]

B. En base a lo estudiado sobre el tema:


a. Determine cuáles son los objetivos del Positivismo lógico, y por qué se denomina de
esta manera a esta corriente de pensamiento.
b. Mencione qué dos tipos de objetos pueden ser únicamente objeto de investigación y
de conocimiento científico para el Positivismo lógico, y señale cuáles no serían objetos
de conocimiento para ellos.
c. Responda:
¿Cómo es posible demarcar el campo entre la ciencia y la pseudociencia según
Popper? ¿Qué método utilizó él para separarlas?
¿Cuáles son las características que tendría que cumplir una teoría para que sea
científica, según el mismo autor?
d. Explique brevemente cómo se decide cuándo una teoría es aceptable (evitamos el
término "verdadera" porque justamente es lo que se discute en distintas posiciones)
para los positivistas lógicos, para Popper y para Kuhn.
e. Responda:
¿Qué supone que podría oponer a las afirmaciones de Bunge un partidario del
pluralismo metodológico?
f. En los últimos párrafos del artículo de Bunge se afirma: "Las pseudociencias son
como las pesadillas: se desvanecen cuando se las examina a la luz de la ciencia. Pero
mientras tanto infectan la cultura".
¿Considera pertinente la extrapolación de un concepto de la biología, "infección", al
análisis de las sociedades? ¿Por qué?

ACTIVIDAD 2. CONCEPCIÓN EPISTEMOLÓGICA DEL POSITIVISMO LÓGICO.


KUHN, TRES PENSADORES EN UNO

A. Lea el siguiente artículo de Bunge:


Todos los universitarios han oído hablar de Thomas S. Kuhn (1922-1996). Parecería
que no se puede pasar por culto sin citarlo. De hecho, Kuhn es el más citado, aunque
no necesariamente el más leído, de todos los autores no literarios. Hasta la fecha su
libro más conocido ha vendido más de un millón de ejemplares en veinte lenguas. Sin
embargo, pocos saben que Kuhn no fue uno sino trino, como diría un teólogo cristiano.
Y lo peor es que el más influyente de los tres no es el que el propio Thomas hubiera
querido ser, o sea, un historiador de la ciencia venerado por sus pares como lo fue, por
ejemplo, George Sarton en su tiempo. En efecto, el Kuhn popular es el de los
paradigmas y desplazamientos de tales, o revoluciones científicas. Éstas eran las ideas
centrales (aunque oscuras) de su libro La estructura de las revoluciones científicas, que
en 1962 le otorgó fama de la noche al día. Un año después lo vi ocupar el centro de la
primera reunión de historiadores de la ciencia, en Filadelfia. En 1965, en Londres,
volvió a atraer la atención en el simposio dedicado a Popper, y ello por dos motivos.
Uno de éstos fue que Margaret Masterman, una filósofa desconocida, expuso una
ponencia clara y combativa en la que mostraba que Kuhn había metido por lo menos
dos docenas de conceptos distintos en la bolsa "paradigma". Entre ellos figuraban los
de cosmovisión, modelo a imitar y programa de investigación. Kuhn aprendió esta
lección. Unos años después, cuando vino a hablar a mi universidad sobre los orígenes
de la teoría cuántica, un asistente le preguntó algo sobre los paradigmas, y él lo paró
en seco: "Estoy harto de eso. Ahora estoy en otra cosa".
Controversia con Karl Popper: Al terminar su conferencia le pregunté cuál sería su
próximo proyecto y me contestó que pensaba estudiar la tesis de Mary Hesse, de que
las teorías científicas son modelos visualizables, como el modelo atómico de
Rutherford-Bohr. Tom no tenía idea de que las teorías son sistemas de hipótesis, ni de
que la teoría cuántica moderna no alienta los modelos visuales, porque se ocupa de
cosas que carecen de forma propia. El otro motivo por el cual Kuhn descolló en aquel
memorable simposio de 1965 fue la resonante controversia que sostuvo con Karl
Popper. El contraste entre ambos era físico, psicológico y filosófico. Kuhn era un
gigantón, hablaba fuerte y fumaba un enorme habano. En cambio, Popper era
menudo, hablaba bajito y odiaba el tabaco. La incompatibilidad filosófica entre ambos
no era menos obvia, pese a que Karl intentó minimizarla. Mientras Popper era
racionalista, Kuhn sostenía la tesis irracionalista de que los cambios de teoría son tan
irracionales como las conversiones religiosas. Sin embargo, paradójicamente, ambos
concordaban en que no hace falta justificar la adopción de una teoría; en particular, los
datos favorables no serían importantes. Pero volvamos a mi tesis. Mi tesis es que hubo
tres Thomas S. Kuhn en una misma persona: el historiador, el filósofo y el sociólogo de
la ciencia. El primero fue ignorado o fuertemente criticado por sus colegas y no formó
escuela. El segundo logró la popularidad que sabemos. Y el tercero, aunque
igualmente popular, sólo existió en la imaginación de ciertos sociólogos de la ciencia:
que lo consideran, junto con su amigo Paul K. Feyerabend, como el cofundador o al
menos padrino de la nueva escuela en ese campo. Esta escuela niega la existencia de
verdades objetivas y afirma que las ideas, e incluso los hechos, son construcciones o
convenciones de grupos o comunidades de investigadores. Se llaman a sí mismos
constructivistas (por oposición a realistas) y relativistas (por negar la existencia de
verdades universales, independientes de las circunstancias sociales). Lo curioso es que,
aunque Kuhn sostuviera que la sociedad cambia de teorías científicas como de modas
sartoriales, sus trabajos históricos son tan internalistas como los tradicionales. O sea,
no practicó como profesional lo que predicó en su libro más popular. No menos curioso
es que este libro fuera publicado originariamente como el último fascículo de la
Encyclopedia of Unified Science, de orientación positivista. Esto es curioso porque Kuhn
era netamente antipositivista. En efecto, no concedía mayor valor a los datos empíricos
y creía más en la analogía que en la inducción (generalización a partir de datos
empíricos). Pero volvamos al constructivismo-relativismo. Contradicciones reveladoras:
Hace unos años, un periodista de Scientific American entrevistó a Kuhn y le preguntó si
creía que, cada vez que cambia la cosmovisión dominante, también cambia el propio
mundo. "¡Por supuesto!", contestó Tom con su vozarrón. Segunda pregunta: "¿Cree
que el mundo que lo rodea existe independientemente de usted?" Respuesta: "¡Por
supuesto!". Esta contradicción muestra a las claras la ingenuidad filosófica de Tom.
Hacia el final de su vida, particularmente en una conferencia que pronunció en Harvard
en 1991, Kuhn se distanció explícitamente de los constructivistas, que niegan la
existencia autónoma del mundo. Aunque siguió admitiendo (como toda persona
razonable) que la política desempeña un rol en la vida científica, negó que éste fuese
el principal. Desgraciadamente, Tom no dijo cuáles son las motivaciones de los
investigadores básicos. El gran Robert K. Merton lo dijo y con razón: son la curiosidad
y el deseo de ganar prestigio. Quienes buscan poder se dedican a los negocios o a la
política. Consejo a los admiradores del triple Kuhn: Decídanse a cuál de ellos venerar,
porque no sólo son diferentes, sino que no armonizan entre sí. A menos, claro está,
que estén dispuestos a reconocer que tampoco esta trinidad es inteligible.
Mario Bunge (2000, diciembre 4). La Nación.

B. Según lo estudiado sobre el tema:


a. Caracterice esquemáticamente el concepto de "progreso" en la ciencia según el
Positivismo lógico, Popper, Kuhn y Lakatos.
b. Indique, de acuerdo con la lectura de los dos artículos de Bunge, ¿a qué corriente
epistemológica cree usted que pertenece él? Justifique.
c. Determine a qué epistemólogo realiza críticas, y cuáles son.
d. En relación con la siguiente expresión de dicho artículo, responda las preguntas a
continuación:
[... ] pocos saben que Kuhn no fue uno sino trino, como diría un teólogo
cristiano. Y lo peor es que el más influyente de los tres no es el que el propio Thomas
hubiera querido ser, o sea, un historiador de la ciencia venerado por sus pares [...].
En efecto, el Kuhn popular es el de los paradigmas y desplazamientos de tales,
o revoluciones científicas. Estas eran las ideas centrales (aunque oscuras) de su libro
La estructura de las revoluciones científicas, que en 1962 le otorgó fama de la noche al
día [...]

Explique ¿por qué afirma Bunge que Kuhn no fue uno sino trino?
d.1. ¿Por qué usa la frase de un teólogo?
d.2. ¿Cuál sería la relación que quiere mostrar Bunge entre Kuhn y la religión? ¿Con
qué objetivo intenta mostrar esa relación? Según lo estudiado en la unidad, elabore
una crítica breve a lo planteado por Bunge.

C. La falacia del espantapájaros o del hombre de paja


Esta falacia está citada en el libro de la bibliografía, se refiere al recurso que consiste
en "inventar" un oponente más fácil de destruir que el oponente real. El nombre viene
del uso de muñecos de paja para entrenarse los soldados antes de un combate. Es
bastante común encontrar este tipo de recursos en las polémicas entre partidarios de
posiciones rivales. ¿Considera que respecto a Freud, a Marx ó a Kuhn, Bunge está
construyendo un espantapájaros? Explique su posición.

Actividad 3. Thomas Kuhn y la visión del ojo del conejo


A. Lea el siguiente párrafo de Thomas Kuhn en La estructura de las revoluciones
científicas:[30]

Desde la atalaya de la historiografía contemporánea, el


historiador de la ciencia puede sentirse tentado a proclamar que cuando cambian los
paradigmas, el mundo mismo cambia con ellos. Guiados por un nuevo paradigma, los
científicos adoptan nuevos instrumentos y buscan en lugares nuevos. Lo que es
todavía más importante, los científicos ven cosas nuevas y diferentes al mirar con
instrumentos conocidos y en lugares en los que ya habían buscado antes [...] los
cambios de paradigmas hacen que los científicos vean el mundo de investigación que
les es propio, de manera diferente [...] Lo que antes de la revolución eran patos en el
mundo del científico, se convierten en conejos después.

En este párrafo Kuhn alude a la figura pato-conejo para aludir a la visión


paradigmática. Lo que desde un paradigma se ve como "pato" en el otro puede verse
como "conejo". Dicho de otro modo, viendo lo mismo, ven cosas distintas. Podemos
decir, entonces, que respecto a teorías rivales como pueden haber sido el
geocentrismo (Tolomeo) o el heliocentrismo (Copérnico), algunos vieron "patos" (la
Tierra está en el centro del Universo) donde otros vieron "conejos" después (el Sol
está en el centro de nuestro sistema planetario). Hasta el siglo XVII, el paradigma o
marco científico era el de Aristóteles, completado por la Astronomía de Tolomeo, que
presentaba el Universo dividido en dos partes de naturaleza diferente, con la Tierra en
el centro: la imperfecta, corruptible y cambiante que llegaba hasta la esfera de la
Luna; y la perfecta, inmutable e incorruptible, sin movimiento alguno, que se extendía
más allá de la esfera lunar. Pero fueron las interpretaciones que Galileo hizo,
elaboradas por Kepler después, y de las suyas propias empleando el telescopio, las que
destruyeron ese paradigma imponiendo la nueva visión que se ha llamado
"copernicana". Un caso parecido de cambio revolucionario es el introducido por la
Teoría de la evolución de Charles Darwin respecto a las teorías creacionistas
anteriores, en el campo de la Biología.

B. En base a lo estudiado sobre la posición de Kuhn:


a. Explique cuál es el papel que cumple la historia de la ciencia en la epistemología de
este pensador, y en qué se diferencia de las epistemologías anteriores.
b. Identifique en las dos modalidades de producción científica de Kuhn (modo normal y
modo extraordinario, y en cada uno de sus elementos), el cambio de la concepción
"creacionista" a la "evolucionista", y/o de la "geocéntrica" a la "heliocéntrica".
c. Intente definir mediante sinónimos el concepto de "paradigma" según este autor.
d. Explique brevemente por qué podría decirse que Kuhn afirma que dos observadores
que contemplan los mismos hechos desde supuestos diferentes ven cosas diferentes,
ven "mundos" diferentes.
e. En virtud de la consigna anterior, explique qué significa que las teorías son
"inconmensurables".

MATERIALES DE LECTURA I
PERSPECTIVA FALSACIONISTA. LA SUPERVIVENCIA DEL MÁS APTO. VIDA Y
MUERTE DE UNA HIPÓTESIS
Según el epistemólogo Gregorio Klimovsky, la concepción hipotética de la ciencia
supone admitir lisa y llanamente que los enunciados científicos son aceptados a título
de hipótesis, de conjeturas, y no de enunciados justificados ya que no ofrecen
garantías para su veificación. Esta conclusión puede parecer pesimista pero también es
posible concebirla como una honesta y modesta posición al admitirse que en las
ciencias fácticas algunas de las teorías más consagradas han terminado siendo
reemplazadas por otras consideradas más eficaces o abarcativas. Supongamos que
frente a un problema tenemos una hipótesis que trata de resolverlo pero allí no
podemos dar por terminada la investigación puesto que el conocimiento científico no
concluye en la mera formulación de hipótesis. Proponer hipótesis no es sinónimo de
obtener conocimiento. ¿Cómo sigue entonces la investigación? Parece razonable
preguntarnos ¿qué se deduce de esta hipótesis? Esto nos lleva a la formulación de
otras hipótesis y a enunciados observacionales que nos permitan controlar la hipótesis
por medio de la experiencia y puede ocurrir que la hipótesis salga exitosa de la
contrastación empírica. Según Popper, y es difícil no coincidir con él (afirma
Klimovsky), la operación de contrastación exitosa no aumenta en ningún sentido (ni
absoluto ni probabilístico) nuestro conocimiento acerca de la verdad de la hipótesis. Si
la hipótesis ha resistido todos los intentos de falsación, se puede admitir que es
"fuerte" en el sentido de que ha mostrado su capacidad de supervivencia. ¿Qué pasa
cuando hay otras hipótesis rivales que compiten como teorías rivales? En general, no
hay más remedio que investigar cada hipótesis por separado y confiar en que alguna
de las hipótesis resulte refutada y otra ha mostrado ser lo suficientemente "fuerte"
para sobrevivir. Esto lleva a algunos autores a señalar que el proceso
hipotético-deductivo opera metafóricamente de un modo similar a la supervivencia
darwiniana en la que se admite la supervivencia del más apto. Dice Klimovsky: "En
esta concepción del método, la vida de una hipótesis (o de una teoría) es dura, trágica,
y para cada una de ellas podría escribirse acerca de su nacimiento, pasión y muerte.
Ante el problema que intenta resolver, la hipótesis nace, pero luego empieza el terrible
proceso de contrastación por medio del cual se la intenta aniquilar. Ella resiste, pero
constantemente es agraviada por nuevos episodios de contrastación hasta que,
finalmente, la refutación termina con ella en un dramático episodio de muerte
epistemológica." (Klimovsky, 2005. 140)

MATERIALES DE LECTURA II
PERSPECTIVA FALSACIONISTA
George Soros, millonario falsacionista
El financiero y billonario George Soros, húngaro de nacimiento, antiguo alumno de
Popper, se sintió tan inspirado por su maestro que bautizó su Fundación Sociedad
Abierta en su honor. Soros ganó millones en la bolsa invirtiendo según las líneas
establecidas por Popper. Popper pensaba que debía considerarse que las teorías
científicas más sólidas eran aquellas que estaban dispuestas a someterse a las pruebas
más severas, sobreviviendo a ellas. Soros aplicó este principio a Wall Street. De este
modo hizo una fortuna al invertir en bonos de seguros hipotecarios de una compañía
de California que había sido sometida a severas críticas por una crisis inmobiliaria;
pensó que el hecho de haber sobrevivido era una prueba decisiva de que la compañía
era, en el fondo, solvente. La Fundación Sociedad Abierta era la teoría política de
Popper puesta en práctica, poniendo a prueba el poder transformador de la apertura.
[31]
El éxito económico de George Soros podría ser visto como un resultado exitoso de
la adopción de las ideas de Popper en el campo de los negocios. Como el mismo
Popper admite, la contrastación exitosa no agrega conocimiento acerca de la verdad de
la teoría, el éxito de Soros en los negocios podría deberse a otros factores o variables.
ACTIVIDAD 4. LAS PERSPECTIVAS FALSACIONISTAS. LAKATOS
Imre Lakatos critica el tipo de falsacionismo que él denomina "ingenuo", sin embargo,
su propósito no es atacarlo sino recoger y profundizar la heurística positiva de aquél,
generando una nueva versión del falsacionismo que denomina "sofisticado",
haciéndose cargo -en su reconstrucción- de las "refutaciones historiográficas" que
Kuhn opuso al programa de investigación falsacionista. En el texto de la bibliografía se
presenta un caso hipotético de investigación científica donde se verían cuestiones
centrales de un programa de investigación como el que propone Lakatos.
En vista a este ejemplo:
a. Determine qué constituye el núcleo duro de ese programa.
b. Identifique algunas de las hipótesis auxiliares que constituyen el cinturón protector.
c. Explique por qué el científico incorpora tales hipótesis a medida que se presentan los
fracasos experimentales.

ACTIVIDAD 5. ACERCA DE DISTINTAS POSICIONES EPISTEMOLÓGICAS


Determine si los siguientes enunciados, son verdaderos o falsos.
Justifique en cada caso.
a. "Según Thomas Kuhn, la verdad constituye una meta inalcanzable a la que nos
acercamos en un proceso de creación de teorías tentativas y eliminación de errores."
b. "Imre Lakatos es un epistemólogo identificado con el Positivismo lógico."
c. "Para Karl Popper, en el desarrollo de una ciencia hay que distinguir entre dos
períodos: uno de desarrollo normal y otro de ruptura no acumulativa."
d. "Un rasgo común entre los inductivistas y los falsacionistas es el esfuerzo por
encontrar un criterio universal y ahistórico para juzgar los méritos entre teorías
rivales."
e. "El epistemólogo Rudolf Carnap se identifica con el falsacionismo sofisticado."
f. "Para Thomas Kuhn, el cambio de paradigma implica progreso en el sentido de
mejoramiento de la teoría."
g. "Según Karl Popper, la observación precede a la teoría y ofrece una base
concluyente para obtener conocimiento científico."
h. "En la epistemología de Thomas Kuhn, los científicos normales se ocupan de falsar
la teoría principal o paradigma."
i. "El epistemólogo Thomas Kuhn fue miembro del Círculo de Viena".

Actividad de integración de las Unidades 4, 5 y 6


En esta actividad presentamos un caso muy analizado en el campo de la Epistemología
y revisaremos contenidos de las Unidades 4 y 5.
Lea el siguiente artículo y, luego, responda las preguntas:
Ciencias Fácticas.
Invención y contrastación de teorías en la investigación científica y modelos
de explicación

Un caso de investigación y elaboración de teorías científicas clásico de la


espistemología de las ciencias médicas es el de Semmelweis en relación con la fiebre
puerperal. Presentemos el caso:
Entre 1844 y 1848 en el Hospital General de Viena un médico húngaro, Ignaz
Semmelweis, realizó una serie de trabajos vinculados a las mujeres en estado de
postparto. En ese Hospital había dos Divisiones de Maternidad. Semmelweis era
médico de la Primera División de Maternidad, y en ella un hecho estaba causando
conmoción: una gran proporción de mujeres que habían dado a luz, contraían una
seria enfermedad, la cual con frecuencia era mortal. Esta enfermedad era conocida con
el nombre de Fiebre Puerperal o Fiebre Postparto.
En 1844, en la División Primera de Maternidad del Hospital General de Viena, sobre
un total de 3.157 mujeres en estado de postparto, murieron de esa enfermedad 260,
un 8,2%. En 1845 el índice de muertes en esa División y por esa enfermedad
descendió al 6,8%, pero en 1846 ascendió al 11,4%. La preocupación mayor estaba
dada porque en la División Segunda de ese Hospital, en los mismos años los índices
fueron del 2,3; 2,0 y 2,7 respectivamente. Es decir, mientras que en la Primera
División de Maternidad los números de muertes por esta enfermedad eran
significativamente altos, en la División Segunda, en cambio, eran significativamente
bajos en comparación.
Semmelweis, posteriormente a esta experiencia, da cuenta en un libro de las causas
de la fiebre puerperal y las prevenciones que deben tomarse para evitarla, relatando
sus esfuerzos para resolver este rompecabezas. Hasta ese momento, cuando se
pensaba en las posibles causas de la fiebre puerperal, una opinión ampliamente
aceptada sostenía que se debía a "influencias epidémicas" las cuales se describían
como "cambios atmosférico-cósmico-telúricos" que se expandían por ciudades y
pueblos enteros produciendo la fiebre puerperal en mujeres en estado de posparto.
Respecto de esta posible causa, Semmelweis sin embargo se preguntaba: ¿cómo era
posible que una epidemia semejante pueda haber infestado durante años la División
Primera y haber eludido a la División Segunda? Y ¿cómo era posible que mientras esta
epidemia acosara al Hospital, casi no se registrasen otros casos en la ciudad de Viena y
sus alrededores? Porque una epidemia de verdad, como el cólera por ejemplo, no sería
tan selectiva.
Semmelweis en principio indica que una causa posible podría darse porque algunas de
las mujeres internadas en la División Primera que vivían lejos del hospital, sufrían
repentinamente de dolores de parto y daban a luz en la calle. Sin embargo, el
porcentaje por muerte de fiebre puerperal en "partos callejeros" era menor que en la
División Primera.
Otra opinión atribuía la causa de mortalidad en la División Primera al hacinamiento;
sin embargo, Semmelweis indicaba que el hacinamiento era mucho mayor en la
División Segunda. Luego Semmelweis descartó otras conjeturas similares. En 1846 se
designó una comisión especial para investigar este problema. Esta comisión atribuyó la
frecuencia de esta enfermedad en la División Primera a las lesiones que se producían
en las pacientes por revisiones poco cuidadosas efectuadas por los estudiantes de
medicina, todos los cuales realizaban sus prácticas de obstetricia en esta División. Con
el objetivo de refutar esta última opinión, Semmelweis señala que: a) las lesiones que
se producen naturalmente en un parto son mucho mayores que las que podría haber
producido un examen poco cuidadoso de la paciente; b) las parteras que recibían
enseñanzas en la División Segunda hacían los reconocimientos de sus pacientes de una
manera muy similar a los estudiantes de obstetricia, sin embargo, no se producían los
mismos efectos; c) cuando, a partir de un informe elevado por esta comisión, se
redujo a la mitad el número de estudiantes y se limitó los reconocimientos hechos a
estas pacientes por parte de ellos, la mortalidad, luego de un breve descenso, llegó a
sus índices más altos.
También se acudió a varias explicaciones psicológicas. Una de éstas indicaba que la
División Primera estaba organizada de tal manera que, cuando el sacerdote debía dar
la extremaunción a una moribunda, antes de llegar a la enfermería debía pasar por
cinco salas. Se sostenía que la presencia del sacerdote quien pasaba precedido por un
acólito que hacía sonar una campanilla, causaba un terror tal en las mujeres de las
salas por las que pasaba que las debilitaba y las hacía más propicias a contraer la
enfermedad. En cambio, en la División Segunda esto no sucedía porque el sacerdote
tenía entrada directa a la enfermería. Semmelweis decidió probar esta suposición. Le
solicitó al sacerdote que suprimiera el toque de la campanilla y evitara pasar por las
salas precedentes a la enfermería para que llegara a la habitación de la enferma
terminal en silencio y sin ser visto. A pesar de todo esto, la mortalidad en la División
Primera no descendió. A Semmelweis se le ocurrió una nueva idea: en la División
Primera, las mujeres estaban acostadas de espaldas, mientras que en la Segunda de
costado. Decidió probar si la diferencia de posición resultaba significativa. Sin embargo,
la mortalidad continuó.
En 1847 finalmente por casualidad Semmelweis dio con la clave para la solución del
problema. Un colega suyo, Kolletschka, sufrió una herida cortante en el dedo con un
escalpelo que estaba usando un estudiante suyo en una autopsia. Luego de ese hecho,
Kolletschka murió presentando los mismos síntomas que las víctimas de fiebre
puerperal. Por ese entonces aún no se había descubierto el papel que cumplían los
microorganismos en ese tipo de infecciones. Semmelweis por su parte comprendió que
la materia cadavérica que el escalpelo del estudiante había introducido en la corriente
sanguínea de Kolletschka era la causa de la enfermedad y muerte de su colega. Las
semejanzas entre las dolencias de Kolletschka y las mujeres en estado de postparto le
permitieron a Semmelweis llegar a la conclusión de que sus pacientes habían muerto
por un envenenamiento de la sangre del mismo tipo. Él, sus colegas y los estudiantes
de medicina habían sido los portadores de la materia infecciosa, ya que todos ellos
solían llegar a las salas de parto para realizar el reconocimiento de las parturientas
inmediatamente después de realizar disecciones de cadáveres en la sala de autopsias y
sólo habiéndose lavado las manos de un modo muy superficial, de tal manera que
incluso éstas conservaban a menudo un característico olor a suciedad.
Nuevamente, Semmelweis puso a prueba esta posibilidad. Si la suposición era
correcta, entonces se podría prevenir la fiebre puerperal destruyendo con medios
químicos el material infeccioso que portaban los médicos y estudiantes en sus manos
por el contacto con la materia cadavérica. Para probar esto dictó una orden que exigía
a todos los estudiantes de medicina que se lavaran las manos con una solución de cal
clorurada antes de realizar cualquier reconocimiento a una enferma. Luego de esto, la
mortalidad por fiebre puerperal comenzó a decrecer, y en el año 1848 descendió hasta
un 1,27% en la División Primera frente a un 1,33% en la División Segunda.
Esta hipótesis es apoyada además por el hecho de que en la División Segunda la
mortalidad fuera más baja, ya que allí las parturientas eran atendidas por parteras
quienes no tenían entre sus prácticas realizar disecciones de cadáveres. Asimismo esta
hipótesis explicaba también el caso de que en los "partos callejeros" la mortalidad
fuera también menor respecto a la División Primera, ya que al ingresar en el hospital
habiendo ya dado a luz, casi nunca se hacía un reconocimiento de la paciente evitando
de este modo la posibilidad de infectarse. También esta hipótesis daba cuenta del
hecho de que todos los recién nacidos que contrajeron la fiebre puerperal lo habían
hecho porque sus madres habían contraído la enfermedad durante el parto; porque en
ese caso la infección se le podía transmitir al niño antes de su nacimiento a través de
la corriente sanguínea común de madre a hijo, mientras que resultaba imposible esto
cuando la madre estaba sana.
Posteriormente, a partir de una serie de experiencias realizadas, Semmelweis pudo
incluso ampliar su hipótesis: en una oportunidad, él y sus colaboradores, después de
haberse desinfectado cuidadosamente las manos, examinaron primero a una mujer en
estado de parto que sufría de cáncer cervical ulcerado y, a continuación, y sólo
habiéndose lavado superficialmente las manos, examinaron a otras doce mujeres de la
misma sala. Once de las doce pacientes murieron de fiebre puerperal. De esta manera
Semmelweis llegó a la conclusión de que la fiebre puerperal podía ser producida no
sólo por materia cadavérica, sino también por materia en estado de putrefacción
proveniente de organismos vivos.
[32]

a. ¿Cuál es el problema?
b. ¿Cuáles son las seis hipótesis que han sido descartadas?
c. Reconstruya cuáles son los argumentos y contraargumentos de esas hipótesis.
d. ¿Cuáles son sus consecuencias observacionales?
e. ¿Cuál es la primera hipótesis confirmada y qué argumentos la apoyaron?
f. ¿Es concluyente la verdad de esa hipótesis? Justifique.
g. ¿Cuál es la segunda hipótesis confirmada que extrae Semmelweis? ¿Qué
argumentos la apoyaron?
h. ¿Utilizó algún método para la elaboración/descubrimiento de las hipótesis?
i. ¿Qué método utilizó para la justificación de las hipótesis tanto de las refutadas como
de la aceptada?
j. ¿Cómo se valora el caso confirmatorio de una hipótesis en una posición
refutacionista como la de Popper o en una posición confirmacionista como la de Carnap
y la de Hempel?
k. ¿Qué es posible explicar mediante la primera hipótesis confirmada?, ¿y mediante la
segunda hipótesis confirmada? Construya el esquema de explicación
explanans/explanandum determinando si se trata de una explicación
nomológico-deductiva o estadístico-inductiva.
l. ¿Podría predecir otros hechos con esta teoría? ¿Por qué?
Temas de la Unidad
7.1. Política científica
7.1.1 Características generales
7.1.2. La planificación de la política científica. Debates contemporáneos
7.1.3. La formación del investigador
7.1.4. Etapas de la política científica argentina
7.1.4. Institutos de investigación y Universidad
7.2. Ciencia, tecnología y sociedad
7.2.1. Ciencia, tecnología y ética: la responsabilidad social del científico y el
tecnólogo
7.2.2. Ciencia y ética: debates en torno a la neutralidad valorativa de la ciencia
7.2.3. La ética de la investigación científica
7.2.4. Ciencia básica, ciencia aplicada y tecnología. Cientificismo y
anticientificismo

Bibliografía obligatoria
Unidad 7 en la Guía de Estudio de IPC, producida por UBA XXI y editada por Eudeba, a
partir de la edición de 2010. Contratti, M. B. (2010). Documento de Cátedra: Política
científica: problemas y perspectivas. En esta Guía de Estudio de IPC, Buenos Aires:
Eudeba; y en el Campus virtual de UBA XXI. Contratti, M. B. (2010). Documento de
Cátedra: Ética y ciencia. En esta Guía de Estudio de ipc, Buenos Aires: Eudeba; y en el
Campus virtual de UBA XXI.
Presentación de la Unidad
En esta Unidad 7 enfocaremos el problema de la ciencia desde un punto de vista
distinto al adoptado en las unidades anteriores, en las que se consideraba a la ciencia
exclusivamente como una forma de conocimiento, la más relevante, por cierto, de
nuestro mundo occidental. En esta unidad analizaremos a la ciencia como un
fenómeno social. Es preciso tener en cuenta que la consideración de la ciencia como
un modo de conocimiento y la ciencia como fenómeno social señalan aspectos distintos
pero interdependientes, de modo que en el desarrollo de los temas se mostrarán
vínculos entre uno y otro modo de enfocar el estudio de la ciencia. En tanto institución
pública, la ciencia guarda varios tipos de relaciones con el resto de la sociedad:
política, económica, cultural, etc. En esta parte del programa veremos a la ciencia
desde dos perspectivas: una es la perspectiva política desde la cual el fenómeno
tecno-científico se muestra como un asunto de interés público, por lo tanto, un objeto
más de las políticas públicas que un gobierno debe implementar en todas las áreas de
su incumbencia. El segundo enfoque sobre la ciencia que desarrollaremos hace
hincapié en el aspecto ético que presenta la ciencia, que es un aspecto ineludible de
todo fenómeno social. En relación con el primer punto se trabaja el texto política
científica: problemas y perspectivas, Documento de Cátedra de María Beatriz Contratti.
Como se dijo más arriba, la ciencia y la tecnología constituyen un área de interés de
los Estados, por eso los gobiernos respectivos deben diseñar políticas destinadas a
organizar las actividades que giran alrededor de esos dos procesos. No pueden llevar a
cabo esta tarea unilateralmente pues su complejidad e importancia para la
organización y crecimiento de un país excede en mucho los propósitos de un grupo
político. Por lo general, la planificación de una política científica y tecnológica, que es el
documento más general y exhaustivo de los principios, propósitos y actividades que se
llevarán a cabo en todo lo que concierna al establecimiento, ejecución y promoción de
dicha política, se realiza en forma conjunta con la comunidad educativa, los distintos
sectores económicos y las restantes fuerzas políticas, aunque sea de modo indirecto,
además de atender a las líneas que en la materia se ofrecen en otros países con los
cuales mantiene relaciones de cooperación. Es por esto que cada una de las cuestiones
involucradas en la planificación genera debates teóricos y posiciones encontradas. En
el texto mencionado se intenta destacar este hecho, pues así se pone de relieve que el
diseño y ejecución de una política científica no es producto de decisiones erráticas o
azarosas; por el contrario, requiere un fundamento teórico afianzado y un arduo
trabajo de coordinación y centralización entre actores y sectores. De acuerdo con este
espíritu, las cuestiones que se trabajarán en los apartados del Documento son las
siguientes: - Introducción y conceptos fundamentales. Aquí se desarrollan las nociones
básicas y generales de lo que es una política científica. Se plantea el carácter histórico
que ésta tiene y la opinión de Mario Albornoz, profesor de filosofía, investigador y
encargado de la gestión en organismos públicos y privados en el país y el extranjero,
sobre los diferentes criterios para establecer el momento en que dicha política surge.
- La planificación de la política científica. En este punto se analizan los factores que
concurren en el planeamiento de la política de la ciencia y la tecnología y las
discusiones centrales en torno a la naturaleza, alcance e importancia de cada uno. La
distinción (o no distinción) de la ciencia básica y la tecnología y sus implicaciones en la
asignación de recursos destaca una diferencia entre las posiciones del físico y filósofo
argentino Mario Bunge y el investigador español en agroquímica Eduardo Primo Yúfera.
Asimismo, se mencionan los planteos en torno a la polémica cuestión de la importación
de patentes del doctor en economía, Daniel Chudnosvsky, y del economista Andrés
López. - La formación del investigador. El tema de este apartado recae sobre la
formación de los recursos humanos, el presupuesto que debe asignarse a este fin, la
dotación de becarios, etc., cuestión central en la planificación de una política científica.
Se considerarán las posturas de Héctor Ciapuscio, doctor en filosofía, y de Carlos R.
Abeledo, doctor en química y ex presidente del CONICET, sobre la evaluación que
merece el sistema científico argentino en relación con el punto en análisis.
- Características de la política científica argentina. Siguiendo el pensamiento de
Albornoz, se mencionan las distintas etapas que se han seguido en la Argentina en
materia de política científica, las características de cada una y principales
representantes. Se examina si en nuestro país se ha alcanzado un nivel acorde con los
estándares internacionales actuales en relación con el papel desempeñado por las
nuevas tecnologías en la implementación de políticas. Se considera la información
aportada al tema por el ya nombrado Abeledo y del economista, ex secretario de
Ciencia y Técnica y actual rector de la Universidad de Río Negro, Juan Carlos Del Bello.
- La investigación científico-tecnológica en la Universidad. En este punto, el análisis se
centra en la cuestión del papel que debe cumplir la universidad dentro del sistema de
investigación. Aunque en las últimas décadas se han formulado proyectos de avanzada
en el campo de la investigación en los que se contempla, entre otras cosas, la
articulación entre investigación y empresa, lo cual involucra una reformulación
completa de bases y objetivos en las políticas de la ciencia y la tecnología; la
universidad no ha acompañado estos cambios en forma regular y orgánica. Las
razones sobre éste y otros problemas de la universidad son examinados en este
apartado por Ciapuscio, Abeledo y Del Bello. - Conclusiones. Se sintetiza la
problemática que se ha ido desmenuzando a lo largo del artículo y se cierra el mismo
con una reflexión de Albornoz sobre la situación argentina actual en materia de política
científica y tecnológica. El segundo aspecto que se considerará sobre la ciencia como
fenómeno social se refiere a la relación entre ciencia y ética. El texto que trabaja el
tema es Ética y Ciencia. La responsabilidad social del científico. Crítica de la teoría
sobre la neutralidad valorativa de la ciencia y de la actividad científica. Polémica entre
cientificismo y anticientificismo, Documento de Cátedra de Contratti. La relación entre
ciencia y ética tiene una historia y esta historia se ha ido articulando a través de los
cambios sociales, las distintas concepciones filosóficas de la ciencia y los proyectos
políticos, entre otras cosas. Es por esto que se han generado entre los estudiosos del
tema posiciones encontradas sobre la naturaleza y alcances de la relación; incluso
dentro de cada una de las posturas se puede encontrar una gran diversidad de
matices, lo cual hace dificultosa una presentación unitaria de la cuestión. Pero quizá el
factor fundamental en la diáspora de perspectivas sobre la relación entre ciencia y
ética, sean las situaciones paradójicas que crea la cultura científica y tecnológica a la
sociedad actual. El rostro dual de la ciencia y la tecnología siempre ha sido reconocido:
por un lado, éstas traen progreso y bienestar en casi todos los órdenes de la vida, pero
su marcha, desde su surgimiento en la modernidad, es tan arrolladora que impide
reestructurar a tiempo la integridad ética necesaria para acompañar esos cambios. Es
así que se ven cuestionadas desde la perspectiva ética, en general, por los daños que
ocasionan a su paso, o en particular cuando se produce un descubrimiento o una
innovación por la inestabilidad y conflictos que generan en la vida social. A partir de
este planteo general, el Documento se organiza del siguiente modo: - Introducción. Se
anticipan los temas y el enfoque de los mismos que se van a desarrollar en el texto.
- Caracteres de la sociedad contemporánea. Se realiza una semblanza de la sociedad
actual cuyos rasgos más sobresalientes llevan la impronta de la ciencia y la tecnología.
Esta caracterización muestra la conflictividad generada por estos factores en la vida
social que hacen imperativa la intervención de la reflexión ética para en algunos casos
detener, y en otros contener, el poder adquirido por la ciencia y la tecnología sobre el
resto de los sectores sociales. - La ética y sus relaciones con la ciencia y la tecnología.
Se ofrecen algunas nociones básicas sobre las que se basa la ética y se examina el
sentido preciso del concepto de responsabilidad en el marco de la relación entre ética y
ciencia. También se desarrolla la noción de ética aplicada y se hace referencia a sus
diversas ramas. Se plantea la discusión sobre si se puede adjudicar al conocimiento
mismo involucrado en las teorías científicas y tecnológicas la responsabilidad por los
efectos nocivos ocasionados por la ejecución de los proyectos científicos y
tecnológicos. Otra cuestión relacionada al enunciado del apartado es la que se refiere a
las evaluaciones de riesgo; se pone en duda el valor que tienen las evaluaciones
obtenidas por procedimientos exclusivamente estadísticos al tiempo que se proponen
otros modos más democráticos y, por lo tanto, más cercanos a una perspectiva ética.
- La ética de la investigación científica. Aquí se trabaja la cuestión de la conducta del
científico en el proceso de investigación. Este proceso se lleva adelante mediante la
aplicación del prestigiado método científico. El planteo central aquí es: ¿el escrupuloso
cumplimiento de los pasos del método basta para que la investigación resulte
éticamente correcta? Algunos dan una respuesta afirmativa a esta cuestión, otros una
respuesta negativa. Estas respuestas están condicionadas por la distinción entre
valores cognitivos y valores morales, si sólo los primeros cuentan en el accionar del
científico o también juegan los segundos en el curso de su trabajo. La autora del texto
toma partido por la segunda opción, y con el fin de especificar los problemas éticos
que pueden surgir en el transcurso de la investigación se analiza un documento de la
National Academy of Science de Estados Unidos: On Being a Scientist: Responsible
Conduct in Research. - El cientificismo, el anticientificismo y la ética. Se ha
denominado cientificismo y anticientificismo a dos posturas opuestas respecto de la
naturaleza de la ciencia, su valor epistémico y su rol social. Para el cientificista, la
ciencia es la única empresa intelectual que proporciona conocimiento universal y
objetivo; al satisfacer los estándares cognitivos más altos tiene un rol preponderante
sobre las demás instituciones sociales. El anticientificista, por el contrario, piensa que
la ciencia es una empresa humana entre otras, y por lo tanto, está sujeta a los mismos
vaivenes políticos y sociales a que están sometidas el resto de las instituciones. Mario
Bunge es considerado uno de los más notables representantes de la postura
cientificista. Pero el cientificismo estaría de algún modo respaldado si la posición
epistemológica a la que se considera asociado, el Positivismo lógico, está a su vez
totalmente justificada. A través del texto de Marta I. González García, José A. López
Cerezo y Luján López, Ciencia, tecnología y sociedad (1996), se pone en duda esa
posibilidad. Estos autores enumeran una serie de objeciones epistémicas al Positivismo
que parecen desautorizar su pretendida preeminencia sobre todo otro conocimiento.
- Ciencia básica, ciencia aplicada y tecnología. Otro de los pilares sobre los que se
asienta la posición cientificista es la consideración de la ciencia básica como un proceso
separado y diferente en sus objetivos y resultados de la tecnología: aquélla sólo se
ocupa de conocer el mundo, ésta de transformarlo. La distinción lleva a considerar a la
ciencia éticamente neutral y a la tecnología como éticamente responsable por las
consecuencias de las aplicaciones de las teorías científicas. Nuevamente, Bunge se
presenta como uno de los defensores de esta idea junto al filósofo alemán Nicholas
Rescher. Evandro Agazzi (filósofo italiano), por su parte, ofrece una concepción más
matizada del punto en cuestión: ve una vinculación tan estrecha entre ciencia y
tecnología que las considera como un fenómeno unitario: la tecnociencia, y esta forma
de entender la relación da cabida al enfoque ético. Por último, se expone el
pensamiento del doctor en filosofía e investigador mexicano León Olivé, quien
mantiene una postura sobre la responsabilidad ética de la ciencia que es afín con la
sustentada por la autora del Documento. - Reflexiones generales sobre el tema. Aquí
se hace hincapié en una concepción de la ciencia como una institución integrada al
cuerpo social y solidaria con él. Se remarcan los efectos negativos de la ciencia que
generan incompatibilidad con la posición cientificista, al tiempo que se muestran las
dificultades conceptuales que tiene que enfrentar una ética de la ciencia por la
diversidad de aspectos que el tema conlleva. Actividades de aprendizaje
ACTIVIDAD 1. LA PLANIFICACIÓN DE LA POLÍTICA CIENTÍFICA
1.1. En el Documento Política científica: problemas y perspectivas se analizan algunas
cuestiones que surgen en el proceso de armado de la planificación de la política
científica.
a. Lea nuevamente los puntos del texto que tratan sobre las siguientes cuestiones:
- La distribución del presupuesto en las áreas consideradas prioritarias:
investigación básica, investigación aplicada y desarrollo tecnológico.
- La política científica orientada al desarrollo, en el que se seleccionan las
actividades que permiten que el país alcance niveles de competitividad.
b. ¿Cuál es el problema en discusión correspondiente a cada uno de los aspectos
señalados en a.?
c. ¿Cuáles son las posiciones respectivas? (Si se mencionan autores que las sostengan,
nómbrelos; si no, exprese sólo la idea)
d. Esquematice los argumentos que respaldan a cada una.

1.2. ¿Qué problemática trae aparejada la transferencia de los resultados de la


investigación a los sectores productivos?

ACTIVIDAD 2. LA FORMACIÓN DEL INVESTIGADOR


a. ¿A qué aspecto de la planificación de la I+D alude el título del apartado?
b. Relacione las nociones de conocimiento e innovación.
c. Explique en qué consisten el modelo lineal de innovación y las críticas que él mismo
ha recibido.
d. ¿Cuáles son las ventajas del modelo interactivo?

ACTIVIDAD 3. CARACTERÍSTICAS DE LA POLÍTICA CIENTÍFICA ARGENTINA


a. Complete el siguiente cuadro sobre las etapas cumplidas por la política científica
argentina de acuerdo con los conceptos vertidos por Albornoz: Etapas Caracteríssticas
principales Problemas

b. ¿Por qué, según Albornoz, la Argentina se estanca en la etapa de institucionalización


de la ciencia en lugar de progresar en línea con los países desarrollados?
c. ¿Cuál es la situación actual del problema mencionado en b.?

ACTIVIDAD 4. INVESTIGACIÓN E INNOVACIÓN EN LA ARGENTINA


Lea el caso sobre la producción de alimentos y luego responda las preguntas que están
a continuación.
Lunes 3 de febrero de 2003
ALTA TECNOLOGÍA: "MADE IN ARGENTINA" ÁRA EL SECADO DE ALIMENTOS

EN CHUBUT, UNA PLANTA ÚNICA EN EL MUNDO


Permite tratar frutas y hortalizas, y conservarlas indefinidamente sin necesidad de la
cadena de frío

- La liofilización posibilita aumentar 12 veces el valor de los alimentos


- La técnica fue inventada por incas y vikingos
- Invap desarrolló una ingeniería original

En el gris de la estepa patagónica, el valle inferior del río Chubut y su corredor de


chacras y cipresales esconden un as de espadas para el campo y la industria criollos.
La novedad es una planta de liofilización (secado por sublimación) para alimentos.
Pertenece a la empresa Nutripac SA, y su ingeniería, por ahora única en el planeta, fue
desarrollada por la firma nuclear y espacial Invap SE de Río Negro gracias a créditos
del Fondo Nacional de Tecnología (Fontar) y avales de la provincia de Chubut.
La liofilización, poco conocida en la mesa local, no altera la estructura físico-química
del material original crudo, pero permite su conservación indefinida sin cadena de frío.
A diferencia de lo que pasa en el secado por calor, con la liofilización el aspecto, la
textura, el sabor y el aroma del alimento crudo no se pierden. Por el contrario, se
intensifican. El proyecto tuvo prioridad nacional y provincial porque supone dos
apuestas simultáneas: el país puede mejorar mucho su performance de exportador
frutihortícola, pero además podría vender fábricas de liofilizados "llave en mano"
aprovechando que tiene la planta más avanzada del mundo, una que vale la mitad,
consume la mitad de energía y prácticamente no se rompe. Si para un tecnólogo en
alimentos liofilizar es extraer más del 95 por ciento de agua, para un transportista
significa llevar diez veces más mercancía, pero sin unidad frigorífica a cuestas; para un
supermercadista, stocks de frutas y hortalizas sin gastos de conservación, y para un
economista, valor agregado local para nuestras exportaciones. Pero mucho valor
agregado: el proceso permite transformar un kilogramo de frutillas frescas, que vale 20
centavos de dólar, en 70 gramos. Sólo que cuando se junta un kilogramo de liofilizado
(a partir de 13 kilogramos de material crudo), el valor del producto terminado no es,
como indican las razones y proporciones, de 2,6 dólares. Es de 30 dólares. Casi 12
veces más.
Aunque la liofilización es compleja, su física es simple: previamente congelado y
trozado, el alimento va a una cámara hermética donde soporta un alto vacío como sólo
lo conocen los astronautas: de los 1025 milibares típicos a nivel del mar, la presión de
aire cae debajo de 1 milibar, lo que supone una zambullida igualmente brutal de la
temperatura. El agua está petrificada como hielo, que en alto vacío "sublima", es decir,
pasa directamente de sólido a gas, evaporándose despacio de los tejidos, que siguen
intactos. Por eso, una frutilla liofilizada parece seriamente una frutilla y no una pasa,
aunque pesa menos que el telgopor. El gusto, sin embargo, no es de telgopor. El
tecnólogo logra que la sublimación no arrastre los aceites aromáticos livianos del
alimento. Por ende, sabor y olor no sólo permanecen intactos, sino que se concentran
en la superficie del material. El resultado es que una frutilla liofilizada tiene, aunque
parezca raro, más gusto a frutilla que una fresca. Y es crocante. Y lo mismo vale para
la carne, la papa, la pera, la zanahoria... "Para poner en cifras el potencial del producto
-explica el doctor Jorge Yanovsky, de Nutripac, rarísima mezcla de biólogo molecular y
empresario-, digamos que si la Argentina dedicara no las actuales 800 mil hectáreas de
tierra a frutas y verduras, sino dos millones, y además exportara todo ese producto
liofilizado, estaría generando un millón de puestos nuevos de trabajo. Y nuevos
ingresos por 30 mil millones de dólares." La especulación de Yanovsky impresiona
cuando se piensa que todo lo que genera en la Argentina el poroto de soja anda por
los seis mil millones de dólares por año. Con frutas y verduras liofilizadas se lograría el
mismo resultado económico... pero en sólo 400.000 hectáreas y con 400.000 nuevos
puestos de trabajo, mitad en las chacras y mitad en el segmento industrial. Son
números que apabullan.
El liofilizado surgió de la necesidad, inventado por incas y vikingos que necesitaban
comida hipercalórica, ultraliviana e imputrescible para sus raids militares. Los incas
aprovechaban el altiplano, con sus noches glaciales y su insolación diurna, para
transformar la papa en chuño y la carne de llama en charqui, los primeros liofilizados
de la historia. Los vikingos, con montañas más bajas y sol más oblicuo, liofilizaron el
arenque con menos perfección. Pero aunque hoy los sobres con sopas, guisos y otros
liofilizados son más cosa de astronautas, montañistas y comandos militares que del
comensal común, hay otro mercado más interesante: son las firmas alimentarias que
descubrieron los liofilizados por su sabor intenso, su consistencia crocante y su
carácter novedoso. Una constante en la antropología alimentaria muestra que lo que
empieza como tecnología de conservación la cultura lo transforma en capricho del
paladar. Nadie piensa hoy en pimientas, ahumados o fermentación como modos de
evitar la pérdida de carnes o lácteos, pero hace milenios su origen fue ése. "Más que
en el terreno de la necesidad, en el del gusto hay un mercado de límites desconocidos,
todo a crear y conquistar", resume Yanovsky.

Invap: del uranio a las frutillas

La tecnología de liofilización deriva de otra francamente vetusta: la de refrigeración.


Al hacer vacío en una masa de gas, una bomba genera frío. En el caso de una
heladera, el frío es lo importante, y en el caso de un liofilizador, un subproducto, pero
la idea de base es la misma desde más o menos 1870. En Gaiman hay otra idea mejor:
en lugar de bombear trabajosamente el aire hacia afuera de la cámara de vacío se lo
puede aspirar con un chorro hipersónico de vapor. Éste pasa rugiendo a cinco veces la
velocidad del sonido por una tobera anexa conectada con la cámara mediante un
empalme en T. El teorema de Bernouilli garantiza que en el vapor hipersónico habrá
zonas de presión bajísima que "chuparán" el aire de la cámara hasta expulsarlo casi
enteramente. Sólo que sin enormes, costosas y frágiles bombas llenas de piezas
móviles y sellos, y con la mitad del gasto de energía. Sencillo, cuesta la mitad, a la
medida de un país rico en gas (sirve para generar vapor), y funciona.
La diferencia conceptual entre el liofilizador de Invap y los que hoy funcionan en otros
países es que el aparato nacional no usa bombas de vacío (carísimas y frágiles). Para
crear el vacío y las temperaturas subcongelantes necesarias, el sistema de Invap usa
chorros de vapor hipersónicos, una idea que los ingleses probaron sin mayor éxito en
la década del 60, considerada desde entonces una vía muerta. ¿Y por qué funcionó
aquí y no allá? Contesta el ingeniero Horacio Bóccoli, de Invap: "En Pilcaniyeu
aprendimos mucho de ambientes de alto vacío, sustancias que subliman sin atravesar
una fase líquida y difusión de gases a través de superficies porosas. Y lo que
aprendimos funciona tanto para el uranio como para las frutillas. La historia mundial de
la tecnología abunda en estos cambios de escenario. Lo bueno es que sucedan
también aquí".
Opina al respecto el ingeniero Horacio Bóccoli: "Si el Invap no hubiera hecho en 1983
la instalación de enriquecimiento de uranio de Pilcaniyeu, en Río Negro, lo de Gaiman
habría sido difícil. Pero aunque Nutripac y Pilcaniyeu son plantas con finalidades muy
distintas, tienen tres asuntos en común: ambientes de alto vacío, sustancias que
subliman sin atravesar una fase líquida y difusión de gases a través de superficies
porosas. Lo que Invap aprendió con el uranio tal vez ahora le permita ganar plata al
país con tomates, frutillas, manzanas... Las vueltas de la tecnología son notables.
Tenemos la mejor herramienta. La ingeniería de Gaiman resulta tan barata, económica
y durable, comparada con todo lo actual, que en medio año de funcionamiento esta
planta ya generó más de 30 pedidos de informes en tres continentes". Muchos vienen
de gigantes mundiales de la alimentación.

Daniel E. Arias (2003, febrero 3).


En Chubut, una planta única en el mundo. La Nación.
Lunes 4 de agosto de 2003

Materia gris de exportación: se venden dos plantas de secado de alimentos


MÉXICO COMPRA ALTA TECNOLOGÍA ARGENTINA

- Se utiliza para extraer hasta el 98% del agua de productos frescos sin alterar su
estructura.
- La primera de las unidades vendidas se está levantando a 350 Km de México DF.
- Dentro de poco se exportará otra más. Ambas para liofilizar frutillas.

Entre 2000 y 2001, la firma nuclear y espacial argentina Invap construyó en Gaiman,
Chubut, una planta para secar alimentos por sublimación, o liofilizarlos, que gasta la
mitad de energía y es más robusta que cualquier otra en el mundo. Ahora, México
acaba de comprar dos de esas unidades, pero además aparecen clientes locales. El
país está por comenzar dos nuevos negocios que reúnen casi emblemáticamente sus
capitales de siempre, tierra negra y materia gris... y por bastantes millones de dólares.
La primera de las unidades exportadas se está levantando ya en Querétaro, México,
zona industrial situada en el valle Central, unos 350 kilómetros al norte del Distrito
Federal. En poco tiempo más, el mismo cliente (Exportadora de Hortalizas SD, gigante
mexicano de la alimentación) pedirá una segunda planta, y dedicará ambas a liofilizar
frutillas para la empresa Kellogg's de Estados Unidos. Ambas plantas son de
considerable tamaño, capaces de rendir hasta 80 toneladas anuales de producto seco.
Por otra parte, la empresa rionegrina está a punto de firmar la construcción de una
cuarta unidad, esta vez para una cooperativa frutihortícola neuquina. Tras décadas de
vender sólo productos frescos, o a lo sumo secados al calor, ahora algunos
productores del Alto valle se aprestan a imitar a los mexicanos y poner un pie en el
hemisferio norte con un producto bastante más exclusivo.

Física, química y matemática

Más allá de la física y la química, liofilizar es multiplicar dólares. Al eliminar el 98 por


ciento o más del agua del producto fresco sin alterar su estructura, sabor o calidad
nutricional, la liofilización permite transformar un kilo de frutillas frescas (que vale 20
centavos de dólar) en sólo 70 gramos de lo mismo. Sólo que cuando se junta un kilo
de frutillas liofilizadas (a partir de 13 kilos de material crudo) el valor del producto
terminado no es, como indican las razones y proporciones, de 2,6 dólares, sino de 30.
Esto explica que, hasta que se conoció la planta de Gaiman, en 2002, hubiera
"frutilleros" tucumanos dispuestos a liofilizar su producto en... China. ¡Y para entrar al
mercado inglés!

Semejantes triangulaciones se originan, ante todo, en que hoy la demanda mundial


de liofilizado supera largamente la oferta. Mientras esto siga así, en el rubro frutillas
quien hoy factura un millón de dólares vendiendo producto fresco podría facturar 6,5
millones liofilizando sólo la mitad de su producción, y 12 millones procesándola toda.
Tanta multiplicación de valor surge de las propiedades del producto terminado: su
sabor natural se potencia, se puede almacenar indefinidamente sin cadena de frío,
suele pesar entre el 5 y el 10 por ciento de lo que pesa la materia prima fresca, pasa
fácilmente las barreras sanitarias y ofrece una gama poco conocida de consistencias y
sabores que está originando muchos productos emergentes de consumo masivo,
asuntos casi de "cocina ficción". Por ejemplo, esa barrita de carne tipo snack que uno
puede comprar en un quiosco neoyorquino para comer en la calle y que hace "crunch"
en la boca como un turrón. o papas crocantes idénticas a las papas fritas... pero
crudas, nutritivamente superiores y totalmente libres de algunos inevitables
subproductos cancerígenos que originan las frituras. El dueño actual de la planta de
Gaiman, Jorge Yanovsky, de Nutripac SA, cuyo prototipo ya ha funcionado tres años
sin problemas, hace el siguiente cálculo: "Si la Argentina dedicara, no las actuales 800
mil hectáreas a frutas y verduras, sino dos millones, y además exportara todo ese
producto liofilizado, estaría generando un millón de puestos nuevos de trabajo, mitad
en las chacras y mitad en el segmento industrial. Y, de yapa, nuevos ingresos por 30
mil millones de dólares". Todo lo que genera en la Argentina el poroto de soja anda
por los 6 mil millones de dólares por año. Con frutas y verduras liofilizadas se lograría
el mismo resultado económico... pero en apenas 400.000 hectáreas y con 400.000
nuevos puestos de trabajo. Son números que apabullan: permiten imaginar un futuro
agroindustrial más integrador que el actual, basado casi únicamente en las
oleaginosas.

Daniel E. Arias (2003, agosto 4).


México compra alta tecnología argentina. La Nación.
a. Caracterice los conceptos de I+D, innovación y competitividad (utilice a este fin el
texto completo de Política científica: problemas y perspectivas, especialmente el
apartado: "La investigación científico-tecnológica en la Universidad"). b. ¿Cómo cree
que operan los factores mencionados en 1.1.a. en el caso de la liofilización de los
alimentos? (Aclare si encuentra un nuevo concepto, además de los mencionados en el
Documento) c. ¿Por qué lo narrado en el caso de la liofilización de los alimentos difiere
de las conclusiones presentadas en el texto sobre política científica? ACTIVIDAD 5.
ÉTICA Y CIENCIA En el Documento Ética y ciencia... de Contratti, se han considerado
centralmente dos aspectos de la relación entre ética y ciencia: uno está referido a la
actuación del científico en tanto investigador y a ciertas normas de conducta que el
mismo tiene que satisfacer desde el punto de vista deontológico de su profesión. El
segundo aspecto apunta a la responsabilidad del científico como productor de
conocimientos en la medida en que tales conocimientos son susceptibles de portar los
valores que el científico o la comunidad de científicos, en tanto seres humanos que
viven en una sociedad determinada y en una época histórica dada, introducen en ellos.
La presente actividad requiere que aplique los conceptos desarrollados sobre estas dos
cuestiones al relato sobre los experimentos nucleares franceses en Argelia. Lea el
artículo periodístico y luego resuelva los puntos que están a continuación.
infobae.com 16/2/10

FRANCIA USÓ CIENTOS DE SOLDADOS EN EXPERIMENTOS NUCLEARES


El ejército francés expuso deliberadamente a sus soldados a radiaciones durante una
serie de ensayos nucleares atmosféricos realizados en los años 60 en Argelia, según un
informe militar.
Al situar soldados cerca del punto de explosión, el ejército buscaba "estudiar los
efectos psicológicos producidos por el arma atómica sobre el hombre", según indica el
primer tomo del "Informe sobre los ensayos nucleares franceses de 1960-1965",
clasificado como secreto.
El Ministro de la Defensa Hervé Morin aseguró que se informaría "con total
transparencia" sobre el nivel de exposición a las radiaciones después de cada ensayo.
El informe, escrito por oficiales franceses seguramente en 1998, es decir después de
que Francia dejara de realizar pruebas nucleares (en 1996), hace referencia
especialmente al último ensayo atmosférico realizado en el Sahara argelino, que se
llevó a cabo el 25 de abril de 1961, cuando Argelia aún era una colonia francesa. Poco
después del lanzamiento, se organizaron maniobras para enviar una treintena de
soldados al área contaminada. Algunos soldados se protegieron en trincheras
individuales cavadas a unos 800 metros del lugar de impacto mientras que otros se
acercaban a dicho punto en camiones 4x4. Estaban equipados con botas, guantes y
máscaras de combate. El objetivo era "estudiar los efectos fisiológicos y psicológicos
producidos sobre el hombre por el arma atómica, con el fin de obtener los elementos
necesarios para la preparación física y la formación moral del combatiente moderno",
explica el informe. Las maniobras debían permitir también "realizar un programa de
instrucción sobre las medidas prácticas que deben tomar los combatientes para
ponerse en guardia, protegerse y descontaminarse", añade el texto. Después de la
explosión, los resultados constatados por el informe señalan "la ausencia de
quemaduras visibles, efectos mecánicos prácticamente nulos, nivel de radiactividad
elevado en maniquíes". "Parecía, según estos resultados, que a 800 metros del punto
cero y afuera de la zona de caída de polvo radioactivo, los combatientes estarían
físicamente aptos para continuar en combate", añade el documento. El texto reconoce
sin embargo que "estando a la ofensiva, si la infantería fuese llamada a combatir en
una zona contaminada (... ) la vestimenta especial no les proporcionaría sino una
protección relativa y habría que reducir el tiempo de permanencia". Morin afirmó que
"una síntesis de ese informe fue elaborada para el Ministerio de la Defensa en enero
de 2007", relatando "las experimentaciones tácticas durante" dos pruebas nucleares. El
ministro recordó además que una ley de indemnización de las víctimas de los ensayos
fue adoptada en 2009, indicando que había exigido "un estudio complementario sobre
cada tiro y el nivel de exposición correspondiente".
"Las conclusiones serán públicas", anunció Morin. Francia ha realizado 210 ensayos
nucleares desde el primero en el Sahara en 1960 hasta el último de 1996 en Polinesia
francesa. Miles de veteranos de los ensayos nucleares, convencidos de haber sido
contaminados por la radiactividad, luchan porque se les reconozcan los perjuicios
ocasionados. En otras publicaciones se dan otras precisiones sobre el operativo
francés:

"Participaron 300 personas, en su mayoría reclutas que 20 minutos después de la


explosión salieron de los refugios y a los 35 minutos empezaron la aproximación a pie
al punto cero y llegaron a 700 metros de distancia. Se hizo llegar una patrulla de
vehículos todoterreno a 275 metros. Los autores del informe de 1998 reconocen que
las autoridades militares de los años 60 hicieron manipular sustancias a los miembros
de la tropa, pese a conocer los peligros que eso podía entrañar. Así, por ejemplo
autorizaron a los trabajadores a no llevar máscara, lo que significaba que podían
inhalar en un día el polvo radiactivo equivalente al "normalmente autorizado en tres
meses". En otro artículo se informa sobre la reacción de los afectados por la
contaminación radiactiva y la respuesta de los Estados responsables. "Existen
imágenes del acontecimiento, aunque el montaje y el relato propagandístico dé una
versión de los hechos completamente diferente. Pierre Messmer llegó vivo a los 91
años, pero el ministro Gastón Palewski murió en 1984 como consecuencia de una
leucemia, supuestamente provocada por el incidente. Se desconoce el número exacto
de afectados por este y otros incidentes, y por los ensayos de Reggane, entre soldados
y habitantes de la región, incluyendo tuaregs que no pasaron por un proceso de
descontaminación (unas duchas no muy fiables). Los primeros se organizaron hace
unos años para reclamar una investigación de las consecuencias de los accidentes, así
como indemnizaciones. Las asociaciones de los afectados por los ensayos nucleares,
Aven y Moruroa e Tatou (esta última reúne a las víctimas de los 193 ensayos que se
realizaron en Polinesia tras la finalización de los experimentos argelinos en 1967),
llevan tiempo reclamando, además de indemnizaciones, cosas tan elementales como
un censo de potenciales afectados, la creación de un comité de seguimiento de las
consecuencias sanitarias y medioambientales de los ensayos nucleares. El pasado mes
de julio se anunció la creación de un comité "verdad y justicia" de apoyo a dichas
asociaciones. Pero ni el Estado francés ni el argelino, cómplice de la antigua potencia
colonial, están dispuestos a dar pasos que impliquen asumir algún tipo de
responsabilidad jurídica".
Francia usó cientos de soldados en experimentos nucleares. (2010, febrero 16).
Disponible en
http://www.infobae.com/mundo/500903-0-0-francia-uso-cientos-soldados-experimento
s-nucleares

a. ¿Cuáles son los objetivos del experimento descrito en el caso?


b. Mencione tres objeciones al procedimiento experimental narrado en el caso, desde
el punto de vista del concepto de responsabilidad analizado en el apartado "La ética y
sus relaciones con la ciencia y la tecnología", del Documento de Cátedra.
c. ¿Qué norma o normas de la investigación se ha incumplido específicamente en este
caso?
d.
i) Determine cuál o cuáles de los siguientes agentes deberían haber evaluado la
implementación de experimentos como el descrito antes de su ejecución: - El gobierno.
- La comunidad científica. - El o los científicos que diseñaron e intervinieron en el
experimento exclusivamente. - La sociedad en cuyo país se realizó la experiencia -
Todos los nombrados.
ii) Explique la razón de su elección.
e. identifique en el caso el aspecto científico y el tecnológico. Suponiendo cuál de los
aspectos mencionados se encontrarían incorporados valores políticos, económicos y
sociales para el cientificista y para el anticientificista? Fundamente su respuesta.
f. ¡Qué opinarían sobre lo ocurrido en Argelia, en el supuesto de que lo hicieran, los
siguientes autores: Mario Bunge, Nicolás Rescher, Evandro Agazzi y León Olivé?
g. ¿Cuáles son los mecanismos de reparación con los soldados "cobayos" utilizados por
el ejército y gobierno francés? ¿Cree usted que son suficientes y adecuados? ¿Por qué?

Introducción y conceptos fundamentales


Se denomina política científica al conjunto de disposiciones gubernamentales
destinadas a organizar el potencial investigador de un país y orientarlo en su
crecimiento y su aplicación en otras áreas donde los resultados de la investigación son
necesarios para su desarrollo. Mario Albornoz (investigador argentino) caracteriza este
proceso como "un ámbito de políticas públicas referido a la producción de
conocimiento, su comercialización, adaptación e incorporación al aparato productivo
[...] configura un vasto espacio en el que confluyen aspectos de diversas políticas,
como la educativa, la económica y la industrial. La política científica y tecnológica está
entramada con todas ellas, lo cual le confiere centralidad, en el marco de las políticas
públicas, y refuerza su naturaleza política" (Albornoz, 1990. 171). A través de las
distintas etapas históricas la política científica se fue complejizando más y más.
Mientras a fines del siglo pasado apenas se podía hablar de administración de las
actividades científicas, hoy tenemos un gran número de categorías distintas
interviniendo en la conceptualización del proceso. Por eso haremos continuas
referencias a fenómenos que se han generado en su carácter peculiar especialmente a
partir de la segunda mitad del siglo XX, como el "proceso de innovación", el "cambio
tecnológico" y su impacto en la economía y la vida social, a las actividades de
investigación y desarrollo (representadas habitualmente por la sigla I+D) que son
objeto de la política científica y a la competitividad, uno de los fines fundamentales de
esta política dada la existencia de un mundo globalizado. La vigencia institucional de la
política de la ciencia tiene una historia muy breve; la mayoría de los autores que se
ocupan del tema acuerdan en que la Segunda Guerra Mundial marca la consolidación
de tal institución. Además de las razones de índole bélica que impulsaron el avance de
la investigación en ese momento (entre los productos del interés militar se encuentra
el proyecto Manhattan de la bomba atómica, el radar en la defensa aérea de
Inglaterra, los cohetes balísticos alemanes, etc.), existieron otras razones procedentes
de las características adquiridas en su expansión por la ciencia y la tecnología: el papel
que juegan en el desarrollo económico de un país, la creciente tendencia a la
aplicación de las teorías científicas, los efectos beneficiosos o nocivos de la tecnología
que provocaron el consiguiente interés social por ella y por la ciencia a la cual está
asociada y la aparición del científico como un profesional independiente. Albornoz no
acuerda con esta consideración histórica, al menos en el caso latinoamericano,
especialmente el argentino. Según este autor es "[...] hasta cierto punto, un lugar
común en la bibliografía sobre el tema dar por sentado que la política científica y
tecnológica constituye un fenómeno que emerge después de la Segunda Guerra
Mundial. Pero esta suposición no toma en cuenta importantes decisiones adoptadas en
épocas muy anteriores, ni la centralidad que llegó a tener el problema científico en el
discurso político, en determinados períodos de la historia argentina [... ] una
perspectiva histórica que no se restrinja a las décadas recientes, permitirá ver que la
ciencia y la tecnología se han acercado al centro de la escena política, o se han alejado
a zonas periféricas, por razones que han obedecido tanto a los proyectos políticos
hegemónicos como a la percepción de los científicos respecto de sus propios intereses"
(Albornoz, 1990. 172). La planificación de la política científica La articulación de todas
las disposiciones decididas por los gobiernos en el área científica y tecnológica se
plasma en planificaciones de largo alcance. Éstas deben ser el resultado del consenso
de todas las fuerzas políticas de un país, único modo de alcanzar la estabilidad
necesaria para el logro de los objetivos propuestos. En el diseño de los planes de
política científica se toman en cuenta distintas cuestiones que van de lo general a lo
específico. Dada la diferente generalidad de los aspectos considerados en los planes,
los actores intervinientes en su confección son también distintos, operan en diferentes
niveles. En este trabajo comentaremos algunas de esas cuestiones, aquellas que a
nuestro juicio muestran en toda su riqueza el debate teórico sobre la cuestión de la
política de la ciencia y que son también, frecuentemente, objeto de debate público y
de interés inmediato para el especialista en este tema, y para el ciudadano. Algunos de
los factores a tomar en cuenta en los planes de políticas científicas son,
esquemáticamente, las siguientes: a) el monto total de inversión en I+D; b) la
selección y valoración de los objetivos mayores: ciencia básica, desarrollo industrial,
servicios, etc.; c) la selección de áreas prioritarias científicas y tecnológicas, donde no
sólo se debe tener en cuenta el nivel de desarrollo a alcanzar, sino también la
formulación de prospectivas, es decir la previsión de los avances científicos y
tecnológicos y los cambios sociales; d) la organización del sistema de investigación y la
coordinación interinstitucional e interdisciplinar; e) el control y la evaluación de los
planes implementados en relación con los resultados de la producción científica y
tecnológica; f) la aplicación de los resultados de la investigación, es decir, la
transferencia de información hacia los sectores productivos (innovación); g) la
promoción y organización de las relaciones científicas internacionales (Yúfera,
1994).[33] Como hemos señalado más arriba, no vamos a analizar puntualmente todas
estas cuestiones si vamos a considerar globalmente ciertos problemas que se
presentan al momento de hacer explícitos los planes de política científica. De todos
modos, los aspectos enumerados no sólo no funcionan en forma puramente
secuencial, sino que además interaccionan creando vías de ida y vuelta y generando
mecanismos de condicionamiento mutuo. Veamos la primera cuestión referida a la
inversión en Investigación y Desarrollo. Es una tendencia general que todos los gastos
de investigación estén unificados en el rubro I+D. El peso otorgado a la I+D dentro del
presupuesto total de un país corresponde a una decisión política cuyo criterio de
decisión se basa en las necesidades relativas de todas las áreas que debe cubrir el
presupuesto y a la función que cumplen la ciencia y la tecnología como variables del
desarrollo económico-social para un Estado determinado. Pero el criterio mencionado
no se aplica de modo uniforme en todos los países. En algunos se suele fijar el monto
en I+D adecuándolo al 3% del PBI. Este criterio es menos racional que el mencionado
en el párrafo anterior, porque al considerar sólo la cantidad de recursos que se van a
volcar en investigación científica y tecnológica se corre el riesgo de que dichos recursos
sean malgastados, al no estar guiados por una planificación ordenada y compatible con
otros criterios de crecimiento a nivel nacional. Los comentarios que haremos a
continuación estarán, por lo tanto, orientados por el primero de los criterios
mencionados. Con el fin de realizar la distribución global del monto asignado en ciencia
básica o aplicada y tecnología, el estudio del presupuesto debe contemplar, en primer
lugar, los gastos de estructura y funcionamiento. Esta estructura está constituida por
los centros de investigación y las universidades. En el presupuesto debe tenerse en
cuenta el mantenimiento de los institutos de investigación, el crecimiento del potencial
investigador en la creación de nuevas entidades y aumento del personal, la dotación
de la investigación universitaria y la renovación del instrumental de alto costo. Es
criterio corriente que la cantidad asignada para mantenimiento del personal no debe
superar el 70% del presupuesto total. En el caso de nuestro país, resulta ilustrativo el
documento del médico argentino Bernardo Houssay de 1921, en el cual este
investigador se quejaba de que aquí se gastaba un 90% del presupuesto en sueldos y
un 10% en laboratorio, mientras en Berlín sucedía casi lo contrario: 61% en los
institutos y 2l% en sueldos. Con el correr de los años, la situación no ha variado
sustancialmente. En segundo lugar, otro aspecto que debe caer bajo la consideración
presupuestaria está ligado a los objetivos considerados prioritarios. Es decir, las
cantidades necesarias para la investigación básica, la investigación aplicada y el
desarrollo tecnológico y los ámbitos específicos dentro de cada una de éstas, que se
deben calcular en función de la importancia que cada una de estas áreas representa en
el desarrollo del país. Consideremos con un poco más de detalle los problemas que
surgen en este punto. En la selección de prioridades entran en juego diversas
cuestiones. Según lo define Eduardo Primo Yúfera (químico español, 1918-2007): "La
política de I+D debe decidirse en función de la política socioeconómica general y nunca
desconectada de ésta, lo que supone que la mayor parte de la comunidad científica
deberá adaptar sus objetivos a los grandes problemas tecnológicos, económicos y
sociales de su pueblo" (Yúfera, 1994. 333). Este autor compara el modo en que se da
este proceso de selección en los países desarrollados y en los de menor desarrollo a la
luz de lo que se considera en los países avanzados el principio fundamental: el objetivo
prioritario de la política científica de un país debe subordinarse a sus necesidades
sociales y económicas y no perseguir el incremento de investigación científica como un
fin en sí mismo. Es posible ver la diferencia de enfoque entre países en desarrollo y
desarrollados a través de la declaración de principios de las políticas científicas de
Francia y Alemania. Las declaraciones dicen, en el caso alemán, lo siguiente: "El
gobierno federal alemán ha tomado conciencia, y así lo expresa públicamente, de que
el fomento y perfeccionamiento de la capacidad de rendimiento científico no apunta
tan sólo a los fines de la política científica, sino que es premisa fundamental para las
realización de todos los demás fines nacionales [...] El gobierno federal actuará de
modo que la política de investiga ción no incida sólo sobre el desarrollo de la ciencia,
sino que tendrá en cuenta las necesidades nacionales relacionadas con ciertas parcelas
de la ciencia y la tecnología y sus consecuencias en la sociedad" (Yúfera, 1994. 334).
Por su parte, en el llamado plan Giscard, que preside las investigaciones de Francia
desde los años setenta, de acuerdo con Yúfera, se afirma lo siguiente: "Serán
prioritarias las investigaciones de fuerte impacto colectivo y las que, en el sector
industrial y agronómico, favorecen el crecimiento de las exportaciones y la
independencia nacional en el aprovisionamiento de energía y materias primas" (Yúfera,
1994. 335). En cambio, en los países sin potencial económico y político independiente,
según Yúfera, "las prioridades sectoriales son vagas, difuminadas y poco
fundamentadas en estudios socioeconómicos serios, y la trascendencia de la actividad
de I+D no se valora a la hora de seleccionarlas" (Yúfera, 1994. 334). Y lo que es quizá
peor, el resultado de las investigaciones en ciencia básica practicadas en estos lugares
se publica en revistas científicas internacionales con la triste consecuencia de que es
aprovechado por otros países poseedores de un desarrollo económico afianzado. En
realidad, hace ya bastante tiempo se ha instalado en los medios conectados con las
políticas de la ciencia una discusión sobre el monto que se debe destinar a la
investigación básica en relación con otras áreas. Reproducimos la versión de Yúfera de
esa polémica según la cual hay dos posiciones extremas: Una de esas posiciones
expresa lo siguiente: "Hay que dar preferencia a la ciencia básica libre porque lo
demás viene por su cauce natural; la investigación básica es la causa primaria de la
riqueza". La otra, en cambio, dice: "La ciencia básica necesaria para el desarrollo
tecnológico está en las revistas a disposición de todos; los países menos ricos deben
aprovecharla para crear riqueza; los países ricos son los que pueden estar en cabeza
de la investigación básica" (Yúfera, 1994. 339). Los ejemplos de Estados Unidos y
Japón apoyarían la última posición. El primero se sirvió de la ciencia básica europea
para lograr su potencial industrial y más tarde logró su liderazgo en materia científica.
Japón tomó tanto la ciencia básica como importó la tecnología que necesitaba, y ello le
sirvió de plataforma de lanzamiento, en primer lugar, de su desarrollo tecnológico y
luego, en los últimos años, de su nivel científico. El físico y filósofo argentino Mario
Bunge (1997), en cambio, denomina practicismo o anticientificismo a la posición según
la cual los intereses nacionales, considerados de naturaleza económica y social pero no
cultural y política, deben orientar la política científica de un país. Rechaza esta
perspectiva porque desde ella se abona el desconocimiento sobre la función cultural
que tiene la ciencia básica, esto es la de satisfacer la necesidad de saber y la de
promover la independencia intelectual de la población, subordinando la ciencia a los
imperativos del desarrollo económico y la producción. Además, con ese desprecio a la
ciencia básica -afirma el autor- se olvida al mismo tiempo que la ciencia es
precondición del desarrollo técnico, "La ciencia útil favorecida en nuestros países en
desmedro de la investigación básica, ha resultado inútil" (Bunge, 1997. 106). El autor
proporciona numerosos ejemplos que muestran el valor de la ciencia para el desarrollo
tecnológico: "A quienes dudan de los beneficios prácticos a largo alcance de la
investigación básica, habría que recomendarles la lectura de Pasteur, o de una buena
historia de la ciencia o de la técnica y recordarles los siguientes ejemplos. La industria
mecánica no puede prescindir de la mecánica teórica, que a su vez nació de la
astronomía, ciencia pura si la hay, aunque necesaria para la navegación de altura y la
confección de calendarios [...]" (Bunge, 1997. 106). En consecuencia, en los países
subdesarrollados siempre es ventajoso practicar la investigación básica, aunque deben
darse ciertas condiciones de orden cultural para que ello sea posible. Tales
condiciones, sin embargo, no constituyen una barrera insalvable. Menciona a modo de
ilustración, entre otros, el caso de Florentino Ameghino,[34] quien desarrolló una rama
de ciencia básica en un contexto cultural que no era el más favorable para ese tipo de
actividades (Bunge, 1997. 114). No sólo se puede hacer buena ciencia básica en el
subdesarrollo, sino que es más fácil hacerla que hacer buena técnica. Ésta exige
inversiones en producción que un país pobre no tiene. Por otro lado, la innovación
técnica depende de la demanda, y "¿A quién se le puede ocurrir diseñar un nuevo
proceso metalúrgico en un país sin industria metalúrgica, o una nueva máquina de
escribir en una nación de analfabetos?" (Bunge, 1997. 117). En cambio, lo único que
se necesita para hacer investigación teórica, del tipo de la matemática pura, física y
química teóricas y ciencias sociales teóricas, es "papel, lápiz y bibliografía". Incluso
pueden realizarse estudios experimentales siempre y cuando no signifique la utilización
de equipos costosos. Para Bunge entonces, la libertad de investigación debe estar
necesariamente contemplada en la planificación de las políticas científicas. Aunque la
mayoría de los especialistas están de acuerdo con el postulado de la libre
investigación, el peso que le otorga Bunge y los argumentos que ofrece sobre su
primacía no son compartidos, en general, en la actualidad. Yúfera entiende que en el
presupuesto de I+D debe destinarse una parte a la actividad libre de los científicos y
otra al desarrollo tecnológico. Cuál sea la proporción que le corresponde a la primera
depende de criterios valorativos y pragmáticos: la creatividad científica es
intrínsecamente libre; pero hay también involucrada una cuestión de decisión política:
dicha actividad responde a la necesidad de integrar grupos internacionales de prestigio
en la materia. En tal caso -señala este autor- el nivel profesional alcanzado por los
científicos debe ser el adecuado al compromiso emprendido, cuestión que conviene
cuidar especialmente en los países en desarrollo donde muchas veces este requisito no
se cumple. Veamos ahora otro punto de vista necesario a tener en cuenta en las
planificaciones de I+D. Es el caso de la política científica orientada al desarrollo, en el
que se prioriza el fomento de las actividades que permitan al país alcanzar o sostener
los niveles de competitividad necesarios para su sustentabilidad y también, como
condición necesaria del desarrollo interno, su participación en el juego competitivo
internacional. La cuestión central que aquí se plantea es la relación que debe
establecerse entre la investigación tecnológica y las líneas de desarrollo económico y
comercial del país. La atención en esta etapa se dirige fundamentalmente al potencial
investigador de las empresas, a quienes el Estado debe apoyar y complementar, por
ejemplo, facilitándoles estudios prospectivos cuando sea necesario. En los países
desarrollados, más del 60% de la actividad de I+D se realiza en las empresas. En los
países menos competitivos, en cambio, el Estado sigue concentrando las políticas de
desarrollo tecnológico. En estos lugares, el problema radica en la forma en que se
obtiene la tecnología producida en los centros desarrollados, esto es, por medio de la
compra de patentes o bien por acuerdos de cooperación, aunque estos últimos en
general son de naturaleza científica y no tecnológica. La utilidad que reviste la compra
de patentes para lograr un nivel de desarrollo adecuado es otro asunto siempre
discutido. Como dice Yúfera, en sí mismo no es bueno ni malo que haya un déficit en
la balanza de pagos por la compra de licencias de tecnología, pues la calificación de la
compra depende de si está integrada o no a planes de I+D. El problema de la
importación de tecnología es analizado por el doctor en economía Daniel Chudnovsky y
el economista Andrés López (1996) en el artículo "Política tecnológica en la Argentina:
¿hay algo más que laissez faire?" La expresión laissez faire hace referencia en general
a la política del "dejar hacer" en un asunto; en el caso del desarrollo tecnológico, se
refiere a la falta de regulación por el Estado de la adquisición de patentes y su
consiguiente ausencia de coordinación con los planes generales de desarrollo. Para los
autores mencionados, la presencia del laissez faire en nuestro país en materia
tecnológica se ha dado en casi todas las etapas histórico-políticas y está
estrechamente vinculado a la política económica vigente y a la desatención crónica de
los marcos regulatorios en muchos asuntos nacionales. En la década de los 90, no
obstante, se adoptó la modalidad del laissez faire de una manera más deliberada en
relación con la política de ortodoxia liberal de la economía que, en la mayor parte de
los casos, considera a la tecnología como una variable exógena (es decir, que no juega
en forma directa en el proceso de equilibrio económico) y con otros supuestos
vinculados a procesos de modernización de la economía. Desde esa perspectiva política
se pensaba que: La apertura de la economía a las importaciones estimularía una
mejora en la eficiencia del sector productivo a través de la mayor competencia en el
mercado local; al mismo tiempo facilitaría el acceso a maquinaria y equipo de última
generación. A su vez, la liberalización del régimen de IED (inversión extranjera directa)
promovería la incorporación y difusión de los conocimientos técnicos y gerenciales de
los inversores extranjeros. Así mismo, la desregulación de los acuerdos de
transferencia de tecnología sería otro estímulo al proceso de modernización.
(Chudnovsky y López, 1996. 34) Estos presupuestos por sí solos no garantizan el
cumplimiento de las metas esperadas. Es preciso poner en marcha políticas de
innovación complejas y continuadas en respuesta a la demanda empresarial de
modernización tecnológica. La mayoría de estas condiciones de una política de
desarrollo tecnológico efectivo a partir de la transferencia de tecnología no se han
cumplido por cuanto "las iniciativas implementadas están desarticuladas, carecen de
rumbo definido y no generan las sinergias que deberían constituir el propósito central
de la política tecnológica" (Chudnovsky y López, 1996. 36). A consecuencia de ello, a
pesar de este intento más coherente y fundamentado, el laissez faire tecnológico no se
diferenció de las políticas erráticas del pasado. En un artículo posterior de uno de los
autores citados, Daniel Chudnovsky (1999), se da cuenta de la introducción de cambios
en esta política. Esos cambios están contemplados en el Plan Plurianual de Ciencia y
Tecnología 1998-2000. Allí ya comienza a instrumentarse un concepto, el llamado
Sistema Nacional de Innovación (SNI) que tendrá mucho peso en lo sucesivo en las
políticas científico-tecnológicas de la Argentina (y en otros países también). Haremos
una referencia a las características específicas del SNI más adelante. Por ahora, basta
decir que es un concepto que involucra a una gran cantidad de actores de diversa
índole (Estado, empresas, mercado, sistema de investigación, universidades, ámbitos
sociales y otros). El plan mencionado inspirado en políticas afines con el SNI deja atrás
el fracasado laissez faire. Lo importante a rescatar en la presentación de este plan es
que, a pesar de que se trata sólo de la formulación del plan y considera las condiciones
concretas de la ejecución del mismo, pone en evidencia la necesidad de desarrollar
internamente la investigación científico-tecnológica (no depender solamente de la
inversión extranjera) y su vinculación con el sector productivo, para lo cual la
articulación por parte del Estado de los diferentes sectores con políticas adecuadas es
un factor imprescindible. Esto, por sí mismo, representó en su momento un avance
importante. Otro objetivo que debe ser considerado en una planificación de I+D es la
transferencia de los resultados de estos planes a los sectores productivos. Dado que el
nivel económico de las naciones está basado, de una forma cada vez más decisiva, en
el desarrollo de las explotaciones agrícolas y la industria, es obligación del Estado
aumentar el nivel tecnológico de dichas áreas. Esta cuestión se articula, no sólo en
torno a la transferencias de conocimientos del Estado al sector productivo privado, sino
también a la distribución del financiamiento total de I+D, o sea a la parte que le
corresponde realizar al Estado y a la parte que le corresponde a la empresa privada. En
general, en los países desarrollados el Estado participa en la financiación de la I+D
realizado en las industrias. Algunos datos ilustran bien la situación: en EE.UU. el 70%
de la investigación se realiza en las empresas, pero del gasto global de I+D, el Estado
aporta el 50%; en Japón el 75-80% de los gastos totales de investigación lo invierte el
sector privado y en Suiza casi toda la investigación la hace la empresa. En los países
de menor desarrollo corresponde al Estado liderar los emprendimientos de I+D. Puede
formalizar programas conjuntos entre empresas y universidades o centros de
investigación, o proporcionar incentivos fiscales a la investigación, subsidios, etc. Un
problema aquí es que las empresas transnacionales no realizan labor de I+D en los
países poco avanzados. Por consiguiente, el país debe importar todas las tecnologías
necesarias cuyas consecuencias son, entre otras, el atraso en materia de innovación y
el desequilibrio de la balanza de pagos. El último aspecto del proceso de planificación
de I+D, estrechamente ligado al anterior, que vamos a considerar aquí, es la
producción comercial de los resultados de la investigación, es decir la transferencia de
la labor de los institutos de investigación aplicada a la industria. En este nivel se
consolida el punto final del avance tecnológico: la "innovación" tecnológica. Aquí se
pone en juego otro aspecto de la financiación que representa grandes inversiones de
capital sin garantía de rentabilidad, pues la innovación producida está sujeta a muchos
factores de incertidumbre, como cambios de ciclos económicos, desequilibrios
financieros, variaciones de la demanda del mercado internacional, etc. El "capital de
riesgo" que se requiere, en general, procede de inversores específicos, es decir, de
entidades financieras ad hoc. Un caso es la Agence National de Valorisation de la
Recherche de Francia, creada para promover el aprovechamiento de la tecnología
producida en los organismos públicos de investigación. La formación del investigador
Un aspecto crítico del proceso de I+D es la cantidad y la formación de científicos que
lo lleva adelante. Lo ideal es que el número de investigadores no implique un gasto
mayor del 65% del gasto global, porque de lo contrario no queda nada para invertir en
mantenimiento y materiales. Algunas cifras, nuevamente, ilustran la diferencia entre
los distintos países en esta cuestión: en EE.UU. la cantidad de personal entre
científicos e ingenieros asciende a 750.000; en Japón, 370.000; en Francia, 135.000 y
en Gran Bretaña, 92.000. Con referencia a la cantidad de habitantes: en España hay
2,21 investigadores por cada 1.000 habitantes activos, en Francia, 5/1.000 y en
Alemania, 5,9/1.000. Hay que tener en cuenta, como observa Yúfera y es importante
para interpretar las cifras, que la escasez de investigadores es a la vez causa y efecto
del bajo desarrollo. Al mismo tiempo, el número del personal condiciona la política
científica y ésta a su vez determina la cantidad de científicos que se requieren de
acuerdo con los planes de I+D, condicionados, por su parte, a la situación económica
general. Como vemos, se producen espirales de las cuales sólo es posible salir
mediante decisiones de alta política cuyos principios y planeamientos trascienden los
intereses sectoriales hacia el crecimiento de la nación. Una perspectiva sobre el modo
en que es posible encontrar un punto de partida para tomar decisiones es la formulada
por Héctor Ciapuscio. Este autor afirma: "Dado que la revolución científico-tecnológica
actual tiene al 'conocimiento' como su factor principal, la formación intensiva de los
recursos humanos es una estrategia hábil para mejorar la situación relativa de los
países menos desarrollados" (Ciapuscio, 1990. 257). Ciapuscio apoya su tesis en
afirmaciones como la siguiente: "La materia prima estratégica de la industria no es
más el carbón, el petróleo o algún metal raro, es la materia gris. Ésta, si se la sabe
movilizar, transforma todo; no hay más sector de industria o de punta del pasado, sino
empresas que giran sobre la inteligencia y otras que giran sobre los resortes menos
poderosos, financieros y materiales" (Ciapuscio, 1994. 257). Una consecuencia de la
adopción de políticas basadas en la premisa de que el saber es el factor fundamental
del desarrollo de un país es la necesidad de estudiar el problema de los recursos
humanos (formación de profesionales, investigadores y becarios) relacionados con las
políticas de la ciencia y la tecnología y los mecanismos que permitan implementar
eficazmente los resultados de dichos estudios. En este aspecto, es importante la fecha
en que Ciapuscio escribe, pues los datos que ofrece al respecto hoy han perdido
vigencia, aunque no podamos decir todavía que se hayan revertido totalmente las
condiciones de atraso. Por ejemplo, el autor se refiere al sistema de becarios, y pone a
modo de ilustración del rezago argentino en este campo: en 1989 Brasil, de acuerdo
con su Programa de becarios, esperaba contar con 10.000 becarios de este tipo; en
tanto la Argentina tenía, en ese mismo año, nada más que 100 becarios externos. Ésta
y otras cuestiones que hacen al tema de los recursos humanos han cambiado en la
actualidad. Existen ejecución de políticas y proyectos pergeñados durante las casi dos
décadas posteriores al texto de Ciapuscio que permiten visualizar mejoras importantes:
en 2006 había ya 3700 becarios sólo del CONICET (Abeledo, 2006b). Ciapuscio analiza
la opinión según la cual la escasez de investigadores en formación se justifica por
nuestra deficiente situación económica y la estrecha estructura del sector productivo y
de servicios. Bajo estas condiciones, mayor cantidad de becarios significaría un gasto
inútil y gente muy preparada sin lugares de inserción para su realización profesional. El
autor califica esta perspectiva de "malthusiana"[35], Venezuela, que contaba en 1974
con poderosos recursos derivados del petróleo, implementó un ambicioso programa de
becas en el exterior. Cuando los beneficios del petróleo se acabaron, el país quedó con
una buena infraestructura industrial, tecnológica y universitaria. Nosotros, en cambio,
afirma, quedamos en total condición de inferioridad frente a un vecino como Brasil, por
ejemplo, que cuenta con una gran población capacitada científica y técnicamente. El
problema es que, pese a los datos en contrario, "en nuestro país se tiene la arraigada
ilusión de que disponemos de recursos humanos de alta calidad y relativamente
abundantes" (Ciapuscio, 1990. 258). Esto se debe a "la supervivencia del mito de la
riqueza nacional y la autocomplacencia de algunos de nuestros dirigentes cuando se
refieren al hombre argentino." En realidad, según el autor, esta idea del hombre
argentino proviene de ciertas condiciones del pasado (segunda mitad del siglo XIX y
principios del XX) en las que se destacan la conducta de ciertos sectores sociales que
contribuyeron en su momento al crecimiento general en materia política, social y
económica del país, a saber: "el legado de la inmigración, la vieja escuela pública y el
período 'cientificista' de la Universidad" (Ciapuscio, 1990. 259). El autor sostiene
también que, comparados con otros países donde se ha producido una verdadera
revolución académica, consistente en incluir la investigación científica en el nivel de
grado universitario, nosotros estamos en el nivel de pregrado, ciclo de enseñanza
caracterizado por la enseñanza repetitiva, el libro de texto y la clase "magistral". El
nuevo modelo, en cambio, pone el acento en la actitud crítica e independiente del
aprendiente, favoreciendo su capacidad de creatividad y cuestionamiento.
Especificando y dando forma concreta a la noción de conocimiento, mencionada por
Ciapuscio, como motor de las decisiones de política tecnológica, Carlos Abeledo (doctor
en química) parte de la noción de un sistema nacional de innovación, surgida de los
encargados de formular políticas científicas en los países industrializados. Dicha noción,
afirma, "proporciona un marco útil para la formulación de políticas tecnológicas[36]
puesto que hace explícitas las muchas y variadas formas de aportes necesarios para
producir una economía innovadora y por lo tanto competitiva en los mercados
actuales, cada vez más globales" (Abeledo, 2006b). Antes de continuar ahondando en
las características de este sistema nacional de innovación, es preciso hacer algunas
aclaraciones respecto del término "innovación". En otro texto del autor que estamos
viendo (Abeledo, 2004a), éste dice que el término tiene dos significados: uno responde
al modelo lineal, según el cual la relación entre investigación y desarrollo tecnológico
sigue la secuencia: Investigación básica → investigación aplicada → desarrollo
tecnológico → introducción de nuevos productos al mercado (Abeledo, 2004a) El autor
sostiene que este modelo lineal ya no responde a la realidad. Originalmente surgió a
partir de innovaciones excepcionales que se gestaron a raíz de descubrimientos
científicos de gran importancia y notoriedad, como por ejemplo los casos de la energía
nuclear, el caucho y los textiles sintéticos, el radar, la computación digital, los
transistores y circuitos integrados, el láser, los antibióticos y la ingeniería genética.
Pero la mayoría de las innovaciones no obedece a la secuencia establecida por el
modelo. La inadecuación del mismo se acentúa cuando se intenta aplicar a los
procesos de investigación, innovación y desarrollo tecnológico al ámbito
latinoamericano. Como explica Abeledo: "La mayoría de las innovaciones que impulsan
el desarrollo tecnológico no son necesariamente invenciones de gran alcance. Algunas
innovaciones son 'radicales' y conducen a la creación de productos o procesos
totalmente nuevos, en tanto que la mayoría son 'incrementales' y representan una
serie de pequeñas mejoras en la forma de hacer las cosas. Las empresas introducen
constantemente innovaciones incrementales en sus productos o procesos para poder
mantener su competitividad" (Abeledo, 2004a). A raíz de estas comprobaciones, y
dada la diversificación de las economías actuales, se ha formulado otro concepto de
innovación denominado modelo interactivo, no causal, que responde a las variadas
relaciones que se establecen entre los principales actores que intervienen en el proceso
de innovación. En este modelo se muestran las interacciones entre los ámbitos de
innovación de las empresas y a su vez éstas con las fuentes del conocimiento, es decir,
las universidades y los organismos de investigación, que en el caso de nuestro país,
son: CONICET (Consejo Nacional de Investigaciones Científicas y Técnicas), INTA
(Instituto Nacional de Tecnología Agropecuaria), INTI (Instituto Nacional de Tecnología
Industrial), CNEA (Comisión Nacional de Energía Atómica), etc. Siendo un modelo no
causal las interacciones pueden darse en paralelo, no linealmente. Es preciso notar
también que el modelo descrito pone de relieve el papel de las empresas y las
demandas del mercado en el proceso de innovación. En el concepto de "sistema
nacional de innovación" se piensa la innovación en el segundo de los sentidos, tal
como lo muestra la definición que da Abeledo: "un sistema de interacción de empresas
(pequeñas o grandes) del sector público y privado, de universidades y organismos
estatales orientado a la producción de ciencia y tecnología dentro de fronteras
nacionales. La interacción entre estas unidades puede ser técnica, comercial, jurídica,
social o financiera siempre y cuando el objetivo de la interacción sea el desarrollo, la
protección, el financiamiento o la reglamentación de ciencia y tecnología nuevas"
(Abeledo, 2006b). Este ambicioso programa de innovación, plasmado en proyectos y
políticas de las instituciones correspondientes, exigió prestar mucha atención al tema
de los becarios y personal de investigación. Desde 1990, fecha del artículo de
Ciapuscio, hasta la actualidad, en todos los ámbitos conectados con la implementación
de las políticas de I+D e innovación se ha tomado conciencia de los cambios que era
necesario introducir, tanto en la formulación de políticas como en el nivel operativo,
para superar el atraso en materia de recursos humanos denunciado por Ciapuscio.
Como vimos, el dato del número de becarios proporcionado por Abeledo en 2006,
contrasta fuertemente con el ofrecido por Ciapuscio. Veremos más adelante si la
enseñanza universitaria también se ha puesto a tono frente a los desafíos planteados a
la educación por la ciencia y la tecnología mundiales. Características de la política
científica argentina Albornoz sostiene respecto de las características de la política
científica en la Argentina "la hipótesis de que, a lo largo del proceso de organización
nacional y durante el presente siglo, hasta nuestros días, la política científica se fue
desplazando desde el centro hacia la periferia de la escena política pero, inversamente,
se especificó 'hacia adentro' del sistema, lo cual alentó tendencias autonomizantes y
corporativas." (Albornoz, 1990a. 173). Entonces, según esta afirmación, la política
científica fue perdiendo esa centralidad no por integrarse a la acción de otros factores
importantes para el desarrollo nacional, sino más bien por encerrarse en sí misma y
separarse de esos otros ámbitos del progreso económico y social. No obstante, desde
su ciuda-dela continuó ejerciendo su papel rector en los procesos políticos de la ciencia
y la tecnología. La distinción de las diferentes etapas de la historia de la política
científica argentina que realiza Albornoz explica claramente la afirmación antedicha, la
ciencia tuvo un lugar central al comienzo del proceso de institucionalización de la
política científica. En la segunda mitad del siglo XIX, en el período de organización
nacional, se desplegó un gran interés por la ciencia expresado en el marco de un
proyecto político articulador del sistema económico, industrial, educativo y de la
consolidación de la institución científica y su inserción en la escena internacional. La
figura que se destaca en este emprendimiento es la de Domingo Faustino Sarmiento,
entusiasta admirador del modelo tecnológico e industrialista norteamericano en
materia de educación e investigación. A pesar de este entusiasmo e interés político por
la ciencia en este período, no se logró alcanzar un contenido científico alto. Es decir,
hubo grandes dificultades para concretar las aspiraciones enunciadas en discursos y
proyectos. Inversamente, en el período siguiente, o sea en el siglo XX, en la etapa que
Albornoz denomina de institucionalización de la ciencia, "La actividad científica se
intensificó, mejoró su calidad y se expandió a un conjunto más amplio de actores. Las
políticas para la ciencia se hicieron más específicas y apropiadas. De este modo el
proceso adquirió un fuerte tono en lo científico pero perdió paulatinamente su
centralidad política. La política científica se fue convirtiendo en un asunto de
científicos" (Albornoz, 1990a. 185). En esta etapa la figura sobresaliente es la de
Bernardo Houssay, un científico con inquietudes por los temas de política científica y
esa preocupación se reflejaba en todos sus escritos. Conocía bien todas las variables y
problemas del proceso de institucionalización de la ciencia, salarios, institutos, becas,
etc., por eso pudo desarrollar normativas respecto de esas cuestiones: los requisitos
que deben cumplir los institutos para funcionar adecuadamente, cantidad y carácter de
las becas, etc. Albornoz considera a Houssay uno de los representantes más notorios
del positivismo argentino. De todas las citas que el primero toma del segundo, la que
mejor refleja esa tendencia del ilustre biólogo, a nuestro entender, es la siguiente:
"[...] Por lo tanto la ciencia es bienhechora. Acorta o suprime las distancias, permite
distribuir la riqueza, disminuye el esfuerzo material, facilita la supresión de todas las
esclavitudes, asegura una vida sana y más larga, con más confort; da las bases para
mejorar el bienestar general, distribuir las ventajas y disminuir las desigualdades
excesivas" (Albornoz, 1990a. 187). Todo este optimismo respecto del poder de la
ciencia es trasladado por Houssay a sus planteos de política científica. En la segunda
posguerra se consolida definitivamente la institucionalización de la ciencia, ya
considerada en concomitancia con la tecnología y el desarrollo económico. Marcó el
comienzo de un segundo ciclo dentro de este período de institucionalización
caracterizado por la preponderancia del paradigma científico, de la "gran ciencia",
especialmente de la ciencia física. En el plano tecnológico, en los años 60
aproximadamente, en el resto de América Latina también comienzan a tomar fuerza las
teorías del desarrollo centradas en la investigación aplicada y en la problemática de la
transferencia de tecnología. Estas cuestiones vistas preponderantemente desde el lado
político, se analizaron bajo la oposición "centro-periferia" en la que se puso el acento
en las relaciones políticas de dominación ejercidas por los países centrales en
detrimento de los periféricos en virtud de su poder económico, político y militar
posibilitado por el desarrollo tecnológico. Un rasgo propio de este período es la falta de
claridad respecto de los vínculos entre ciencia y cambio tecnológico, explicable en
razón de la estrecha relación que el desarrollo tecnológico tiene con la vida económica.
Este problema no resuelto se presentó tanto en el plano conceptual como en el
organizativo. El modelo descrito comienza a mostrar su agotamiento en la década que
va de los 70 a los 80. El centro de este cambio está constituido por las llamadas
"nuevas tecnologías", es decir, la tecnología de las comunicaciones, de la información y
la ingeniería genética, cuya característica principal es el acercamiento que establecen
entre la investigación básica, el desarrollo tecnológico y el aparato productivo. En
nuestro país el reacomodamiento que se debe operar en el plano de las políticas a raíz
de las nuevas formas de conocimiento y difusión de estas tecnologías sofisticadas, esto
es, la revisión de la racionalidad de las prioridades, la distribución de recursos, el tipo
de institución requerida y el perfil adecuado del profesional, tardó mucho en
producirse. Recién avanzada la década del 90 y estos primeros años del siglo XXI, se
vieron los frutos del trabajo de políticos, técnicos e investigadores plasmados en
proyectos y prospectivas que buscan armonizar el sistema argentino de I+D al
concierto internacional y propiciar de ese modo el tan ansiado despegue político, social
y económico del país que le otorgue al mismo tiempo un lugar destacado en el
progreso de las naciones. El ejemplo más destacado en este sentido es el Plan
Estratégico Nacional de Ciencia, Tecnología e Innovación "Bicente-nario" 2006/2010.
La lectura de sus metas y objetivos muestra a las claras que en el país se ha asumido
la necesidad de actualizar los planes de acuerdo con los conceptos vertidos en este
Documento: (i) alcanzar al final del período una inversión consolidada en CTI del 1%
del PIB, con una participación paritaria estatal y privada, y un total de investigadores
equivalentes a tiempo completo de 2,3%o de la PEA; (ii) desarrollar capacidades,
recursos humanos y acciones de investigación e innovación en nueve "áreas-problema-
oportunidad" y en diecinueve "áreas temáticas prioritarias" (disciplinarias y
tecnológicas), en ambos casos en coordinación con los ministerios sectoriales; (iii)
desarrollar el Sistema de Información de Ciencia y Tecnología Argentino (SICyTAR);
(iv) evaluar externamente las instituciones nacionales del sistema nacional de CTI e
impulsar programas estratégicos de fortalecimiento institucional y cambio estructural;
(v) desarrollar y consolidar la federalización de la CTI a nivel de provincias y regiones;
(vi) promover actividades de divulgación y alfabetización en ciencia y tecnología y, (vii)
fortalecer los mecanismos de coordinación del sistema y de programación a corto,
mediano y largo plazo. (Del Bello y Abeledo, 2007) La investigación
científico-tecnológica en la Universidad En relación con las exigencias que plantea la
nueva orientación en materia de investigación, Ciapuscio, en su trabajo comentado
más arriba, señala las deficiencias que padece el sistema de investigación en nuestro
país, y recomienda una rápida reconversión del mismo si queremos evitar una situación
de atraso irrecuperable frente a la velocidad del cambio tecnológico. Entre muchas
carencias, señala que "ramas enteras de las nuevas ingenierías y de las tecnologías
están desiertas" (Ciapuscio, 1990. 259). Podría esperarse que hoy las cosas hayan
mejorado notablemente si se llevaran a la práctica los proyectos establecidos, los
cuales, como el denominado "Bicentenario", están orientados a promover la
innovación. Para que esto ocurra, las instituciones de investigación tendrían que
presentar un número alto de investigaciones dedicadas a solucionar los problemas
científico-técnicos más urgentes del país, es decir, las que ponen en marcha a las
nuevas tecnologías. Pero las conclusiones de los analistas de políticas públicas en
materia de I+D e innovación no son positivas: el CONICET ha alterado sus objetivos
iniciales que apuntaban a apoyar las investigaciones de los docentes que trabajaban en
las universidades y otras instituciones de investigación científica y técnica. En la
década del 70 se modificó el estatuto de la carrera de investigador y los investigadores
pasaron a tener una relación directa con el CONICET, de modo que el organismo
dedicó más a atender la carrera de investigador de su propio personal abandonando el
objetivo de formación de investigadores en relación con las necesidades del país.
Abeledo afirma que hay una diferencia sustancial con lo que sucede en los países
industrializados: La comparación de los datos estadísticos con los de países de mayor
desarrollo muestra, además de las diferencias cuantitativas de la cantidad de
investigadores en relación con la PEA (población económicamente activa), una
distribución con una muy baja proporción de investigadores en empresas. En los
sistemas de innovación de mejor desempeño, la proporción de investigadores en
empresas es comparable a la que se desempeña en universidades y en institutos
públicos. Un aumento en las tasas de formación de nuevos investigadores debería
estar acompañado por un aumento en la apertura de nuevas plazas, especialmente en
las empresas privadas. (Abeledo, 2006b) Albornoz, por su parte, en su artículo
"Universidad y nuevas tecnologías" atribuye a la Universidad la función de "crear,
atesorar y difundir el conocimiento científico, tanto a través de la formación de
profesionales, como de la prestación de servicios a la sociedad" (Albornoz, 1990b.
263). Dicho en otras palabras, debe asumir en todos los niveles el fenómeno de las
nuevas tecnologías. Pero este ideal es por el momento difícil de alcanzar para los
países latinoamericanos: el cambio tecnológico y la respectiva adaptación del sistema
educativo comenzaron en los países industrializados y desde allí se siguen produciendo
ondas expansivas de innovaciones que abren con los países de la periferia brechas
tecnológicas y problemas en la división internacional del trabajo. Otra consecuencia
para los países en vías de desarrollo por esta falta de protagonismo en la producción
de tecnologías es la dificultad para constituirse en culturas tecnológicas; existe más
bien un dualismo tecnológico provocado por la presencia de empresas de tecnología de
punta, por lo general pertenecientes a empresas multinacionales, y otras retrasadas y
obsoletas. También en este orden pueden mencionarse la infrautilización de la
capacidad productiva, la falta de incentivos en la inversión local y la ausencia de
alternativas tecnológicas viables. La Universidad refleja estas tensiones políticas,
económicas y sociales dando lugar a falsas dicotomías como "masividad" versus
"excelencia", o a una situación de incapacidad para adaptar sus estructuras a la
incertidumbre del cambio tecnológico (¿qué carreras deben priorizarse, cuál es la
duración real de las carreras?). La Universidad de Buenos Aires es la institución
argentina que, pese a sus deficiencias estructurales, cuenta con una importante
tradición de investigación creativa en ciencia, con la mayor cantidad de investigadores
y con el prestigio conferido por los Premios Nobel a sus graduados (B. Houssay,
medicina, 1947; L. F. Leloir, química, 1970 y C. Milstein, medicina, 1984). No ha
avanzado, sin embargo, en relación con las urgencias planteadas por las nuevas
tecnologías. Para alcanzar el nivel adecuado, Albornoz entiende que es necesario
entender la investigación básica como investigación estratégica, de acuerdo con la
proximidad que hoy tiene la ciencia básica con la industria. En el momento en que
Albornoz publica su texto, existe una escasa cantidad de investigadores dedicados a
disciplinas estratégicas básicas (biotecnología, biología celular y molecular,
microbiología y genética) o a las aplicadas (microbiología aplicada, inmunología, etc.).
¿Ha cambiado esta situación en la actualidad? Como dijimos antes, ha cambiado en los
proyectos pero no en los hechos, o no lo suficiente. Un caso ilustrativo: en 2007 el
Ministerio de Economía lanzó el Plan Productivo Nacional. En él se destaca un concepto
clave para la cuestión del desarrollo nacional que estamos examinando: el concepto de
competitividad. Transcribimos las afirmaciones de Juan C. Del Bello y C. Abeledo sobre
el significado de este concepto: La competitividad es entendida como el incremento
sistemático del nivel de productividad de la economía y la transformación de la
estructura productiva, a nivel de sectores, cadenas de valor y conglomerados
productivos, que combinen un alto grado de innovación y que no sean islas de
modernidad. El Plan afirma: "El reto de la política productiva para la Argentina es,
entonces, promover un cambio que transforme la estructura productiva a favor de
actividades que incorporen más innovación y que contribuyan al desarrollo de un tejido
productivo más denso, en forma consistente con los ejes rectores de aumento de las
exportaciones, la inversión y la competitividad."
En cuanto a los lineamientos sectoriales se promueven las actividades de "alto
impacto potencial": biotecnología, nanotecnología y energías renovables, los "sectores
innovadores": agroalimentos, farmo y agroquímica, TICs, tecnología nuclear,
aeroespacial y satelital, bienes de capital y servicios de alto valor agregado (turismo,
industrias culturales, diseño, etc.) y los "sectores tradicionales". El documento explicita
la necesidad de la "coordinación interministerial", específicamente del Plan Estratégico
Nacional de Ciencia, Tecnología e Innovación, el Plan Estratégico Territorial y el Plan
Nacional de Inversión Pública. (Del Bello y Abeledo, 2007) Lamentablemente, si bien la
finalidad que el plan enuncia es la correcta, falla en la especificación de cuestiones
básicas que debería contemplar. No establece, por ejemplo, "la institucionalidad
necesaria para asegurar una buena gobernanza del Plan", es decir, no indica mediante
qué mecanismos se van a relacionar las actividades de investigación con el sector
productivo y con el de financiamiento. Como hemos señalado a propósito del
funcionamiento del CONICET, los institutos de investigación no se han adecuado a los
lineamientos establecidos por los planes y proyectos gubernamentales. Dicen los
autores: [Los organismos estatales] Padecen de baja autonomía respecto de la
administración central, cuentan con rígidos sistemas de personal e inflexibles
regímenes de remuneraciones, no tienen flexibilidad y agilidad para generar recursos
propios y disponer de ellos, así como de su patrimonio. En fin, son instituciones
pensadas para el enfoque superado de la "transferencia de tecnología", que supone
una situación de generación y oferta de tecnologías por parte del Estado, y un sector
empresarial pasivo "demandante". El enfoque de los sistemas nacionales de innovación
ha superado esa visión, pero las instituciones continúan funcionando con el viejo
enfoque. Los "involucrados/actores" (stakeholders) no tiene participación en los
gobiernos de las instituciones, o la participación es de naturaleza corporativa
(representantes del sector privado en el Directorio del CONICET sin empresas, por
ejemplo). Mientras que otros países de la región han reestructurado instituciones o
generado nuevas, con un enfoque basado en la asociatividad y en el desarrollo de
redes, los institutos estatales argentinos permanecen congelados en el tiempo. (Del
Bello y Abeledo, 2007) Otra condición que puede favorecer el crecimiento en materia
de investigación, según Albornoz y los demás autores que estamos viendo, es la
integración de la investigación con la estructura docente, es decir, integrar la
investigación en la Universidad y proyectarla realmente en los planes de estudio. Hacer
esto significaría adecuar la Universidad, institución rígida por naturaleza, al dinamismo
del cambio tecnológico, es decir, tornarla flexible y cambiante, significa pedirle que "no
enseñe tanto conocimientos objetivos, como que entrene para la discusión de
alternativas, opciones, riesgos y ganancias potenciales", que reconozca que "el cómo
que transfiere el conocimiento se ha convertido en algo más importante que el propio
conocimiento transferido" (Albornoz, 1990b. 271). Conclusiones Hasta 1990, la
Argentina, y América Latina en general, presentaban un atraso radical respecto de los
países industrializados en materia de políticas educativas orientadas al desarrollo y el
crecimiento socio-económico. Durante la década de los 90 se comienza el intento de
recuperar el terreno perdido pero con políticas equivocadas, tal como lo señalan
Chudnovsky y López (1996), por eso no se produjo el cambio estructural esperado. En
la década siguiente, que llega a nuestros días, se intenta dar nuevo impulso a la
investigación científica y tecnológica orientada a la innovación, ya no dependiendo de
la inversión extranjera como en la década anterior, sino fortaleciendo la infraestructura
existente mediante la creación de organismos adecuados y planes conceptual y
técnicamente a la altura de las exigencias internacionales. El contenido del Proyecto de
Plan Nacional de Ciencia, Tecnología e Innovación Productiva de 2003, 2004 y 2005,
elaborado por la Secretaría de Ciencia, Tecnología e Innovación Productiva, corrobora
lo que acabamos de decir. Sin embargo, los cambios que tales propuestas de avanzada
introducen no se han concretado en las acciones correspondientes. Como se ha
expresado a lo largo del presente documento, la inercia de los organismos de
investigación, el academicismo universitario nacional, la desvinculación de ambos con
las necesidades sociales y las empresas, a lo que se suma la no intervención del Estado
en estas cuestiones, impiden realizar las modificaciones necesarias. Para finalizar, se
transcribe una reflexión de Mario Albornoz que reitera todo lo dicho hasta aquí de
forma muy contundente: Hoy, en cierta medida, en ciencia y tecnología vivimos del
capital acumulado en las décadas de los cincuenta y los sesenta, ya que ellas fueron
pródigas en esfuerzos y resultados. En aquellos años se consolidó la capacidad
científica del país, se crearon sus principales instituciones (como el CONICET, el INTA,
el INTI, la CNEA) y las universidades nacionales alcanzaron un alto nivel en materia de
investigación y se constituyeron en los principales núcleos impulsores de la
investigación científica. El panorama actual es un reflejo empobrecido del pasado
esplendor. Buenos ejemplos actuales, como los de la empresa INVAP (que es una
sociedad del Estado perteneciente a Río Negro) y la Comisión Nacional de Actividades
Espaciales (CONAE), son excelentes casos de capacidades tecnológicas, aunque
lamentablemente aisladas, en un escenario con un perfil productivo de escaso valor
agregado. El modelo neoliberal, pero también la incapacidad puesta de manifiesto por
los sectores más progresistas para generar un nuevo pensamiento movilizador, carece
de motivos para formular políticas que tiendan a aprovechar al máximo las capacidades
disponibles. De este modo, un país que décadas atrás pudo producir premios Nobel y
desarrollar tecnología propia, hoy no puede sostener el esfuerzo científico-tecnológico
que requiere el mundo de fin de siglo (Albornoz, 2003. 2). Bibliografía Abeledo, C.
(2004a). La investigación en la Universidad: ¿Creación de conocimiento o desarrollo
tecnológico? En Universidad, Sociedad y Producción. (J. C. Pugliese, Ed.). Buenos
Aires. Abeledo, C. (2006b). Las necesidades de recursos humanos para el desarrollo
del sistema nacional de innovación. Albornoz, M. (1990a). Consideraciones históricas
sobre la política científica y tecnológica en la Argentina. En Albornoz, M. y Kreimer, P.
Ciencia y tecnología: estrategias y políticas de largo plazo. Buenos Aires: Eudeba.
Albornoz, M. (1990b). Universidad y nuevas tecnologías. En Albornoz, M. y Kreimer, P.
Albornoz, M. Ciencia y tecnología en la Argentina: capacidades sin sustento, 2003.
<http://www.escenariosalternativos.org> Bunge, M. (1997). Ciencia, técnica y
desarrollo. Buenos Aires: Sudamericana. Chudnovsky, D. (1999). Políticas de ciencia y
tecnología y el Sistema Nacional de Innovación en la Argentina. Revista de la CEPAL.
N° 67. Chudnovsky, D. y López, A. (1996). Política tecnológica en la Argentina: hay
algo más que laissez faire? Redes, revista de estudios sociales de la ciencia (vol. III).
Universidad Nacional de Quilmes. Ciapuscio, H. (1990). Formación intensiva de
recursos humanos: una necesidad imperiosa. En Albornoz, M. y Kraimer, P. Ciapuscio,
H. (1994). El fuego de Prometeo: tecnología y sociedad. Buenos Aires: Eudeba. Del
Bello, J. C. y Abeledo, C. (julio de 2007). Reflexiones sobre cuestiones pendientes de la
Agenda de Política Pública en Ciencia, Tecnología e Innovación de Argentina. Primer
Congreso Argentino sobre Estudios Sociales de la Ciencia y la Tecnología. Universidad
Nacional de Quilmes. Yúfera, E. P. (1994). Introducción a la investigación científica y
tencnológica. Madrid: Alianza.

LA RESPONSABILIDAD SOCIAL DEL CIENTÍFICO. CRÍTICA DE LA TEORÍA


SOBRE LA NATURALIDAD VALORATIVA DE LA CIENCIA Y DE LA ACTIVIDAD
CIENTÍFICA.
POLÉMICA ENTRE CIENTIFICISMO Y ANTICIENTIFICISMO
María Beatriz Contratti

Introducción La relación entre ética y ciencia deriva principalmente de la imperiosa


necesidad de la sociedad contemporánea de poner en orden o contener los resultados
de las investigaciones científicas que se hacen tangibles para la mayor parte de los
habitantes del planeta a través de la tecnología. Desde luego, la ética no es el único
modo a través del cual puede llevarse a cabo esa tarea de ordenamiento. También lo
hacen la jurisprudencia, la política y la religión, por ejemplo. Pero el fenómeno de la
ciencia y tecnología ha desbordado las posibilidades de esos tres ámbitos que se han
ocupado tradicionalmente de generar o mantener orden social. Por qué esto parece ser
así, se irá viendo a medida que se avance en el desarrollo de los temas. Una de las
primeras tareas que se abordarán aquí, entonces, será la de esbozar brevemente los
rasgos de la sociedad actual que, precisamente al estar caracterizada por la impronta
de la ciencia y la tecnología, se suele denominar sociedad del conocimiento o sociedad
de la información. En segundo lugar, se hará una breve síntesis de lo que es la ética,
en tanto marco normativo de la conducta humana y en tanto disciplina filosófica, para
referirnos después a la ética aplicada, terreno al cual pertenece en parte el tema de
este texto. Estas nociones permitirán incursionar en la cuestión específica que nos
ocupa: los aspectos éticos de la ciencia y/o de la tecnología. Sobre todo, se intentará
hacer hincapié en el modo de reflexión que opera en este campo, con el propósito de
poner en evidencia los problemas o cuestionamientos que más comúnmente se
formulan a la ciencia y la tecnología desde diversos sectores sociales. Se verá que es el
concepto de responsabilidad el que predomina en los juicios éticos que tienen por
objeto a la tecnociencia. Una vez aclaradas estas cuestiones básicas preliminares, es
posible examinar uno de los aspectos de la ética de la ciencia: la investigación
científica. En ésta se pone de relieve la conducta del investigador a la luz de la
corrección o incorrección de sus acciones en relación con la producción de
conocimiento. Como todo trabajo o profesión, la tarea del investigador debe ajustarse
a ciertos estándares establecidos, algunos generales y otros peculiares a su campo,
para que el resultado de su tarea sea el adecuado. Se analizarán algunos de esos
estándares o reglas de conducta. Pronto se verá que la ética de la ciencia no puede
reducirse a juzgar la tarea del científico como si de eso sólo dependiera la obtención de
resultados confiables de las investigaciones, entendiendo por resultado confiable
teorías ciertas y objetivas o altamente probables. La conducta del científico ajustada a
normas éticas es imprescindible en una investigación seria, pero el problema de la
ética de la ciencia es más complejo y difícil de tratar. Incluye el análisis y crítica del
conocimiento mismo surgido de las investigaciones, aun de aquellas que respetan
todos los protocolos requeridos por la labor científica. El análisis epistémico de la
investigación y las teorías científicas mostrará que la pretensión de obtener
conocimiento altamente confiable, objetivo, es una empresa mal encaminada. El
conocimiento, desde este análisis, resulta ser más bien "construido" que "obtenido".
Las conclusiones de esta forma de ver el conocimiento de la ciencia tienen importantes
implicancias en el tema que nos ocupa. Esto lleva, finalmente, a la distinción entre
cientificismo y anticientificismo, términos estos desde los que se ha planteado la
polémica entre dos posturas opuestas: quienes piensan que la ciencia es neutral
respecto a valores, es decir, es conocimiento "obtenido" acerca del mundo por
procedimientos confiables, no contaminado con intereses o tendencias personales,
políticas o ideológicas (cientificismo), y quienes creen que la ciencia es conocimiento
"construido", lo que hace inevitable que todos esos sesgos e intereses confluyan en su
producción (anticientificismo). Es preciso aclarar que la postura cientificista, que
proporciona una imagen de la ciencia como conocimiento imparcial y objetivo, tiene
aún, a pesar de que los recientes estudios sobre la ciencia han mostrado que esa
imagen es inadecuada, una influencia muy grande en las capas dirigentes de la
sociedad, lo que conlleva una unión íntima entre ciencia y poder, o conocimiento y
poder. Como cierre, se intentará brindar un panorama del mundo actual que recoja
todos los aspectos de la relación ciencia-ética que se han ido comentando. Se dejará
claro que cuando se habla de ciencia se quiere decir en realidad "tecnociencia" en
todos los casos, pues una y otra, a diferencia de lo que piensa el cientificista, son, en
la práctica, difíciles de distinguir y, a los efectos de considerar las consecuencias de la
aplicación del conocimiento científico y tecnológico en el mundo social y natural, son
indistinguibles. Caracteres de la sociedad contemporánea "Saber es poder", el
famoso aforismo enunciado por el filósofo inglés Francis Bacon[37] en el siglo XVII,
parece encontrar en la sociedad actual un significado pleno. Efectivamente, en la
segunda mitad del siglo pasado se caracterizó a la sociedad contemporánea como la
"sociedad de la información" y "sociedad del conocimiento" justamente porque la
información y el conocimiento constituyen los recursos fundamentales que nutren su
cultura y resultan condición de su mantenimiento y desarrollo. El alto valor adquirido
por el saber en el presente se relaciona con la difusión de las llamadas "tecnologías de
la información", por un lado, y, por otro, con la preponderancia adquirida por el
conocimiento científico sobre cualquier otro tipo de conocimiento. Las tecnologías de la
información presiden el desarrollo tecnológico en general puesto que toda la técnica
actual depende para su funcionamiento en forma directa o indirecta de la primera. A su
vez, la tecnología de la información ha sido posible gracias a la comprensión por parte
de la ciencia de los mecanismos que permiten a los seres vivos en general interactuar
con su medio, de los procesos cognitivos estudiados por la psicología, de los
conocimientos matemáticos avanzados y del interés en transformar los dispositivos
mecánicos en automáticos en beneficio de la producción industrial.[38] Es fácil advertir
que la complejidad y profusión del panorama que presenta la cultura científica y
tecnológica contemporánea hace que los fenómenos como la ciencia y la tecnología no
se dejen apresar en definiciones o caracterizaciones simples. Una afirmación como la
siguiente: "a la ciencia sólo le interesa ofrecer conocimiento del mundo", resulta
demasiado estrecha si tomamos en cuenta las diversas motivaciones que impulsan a
los científicos a realizar su trabajo y no considerar que el conocimiento sobre el mundo
es algo tan problemático y discutido que hace poco plausible dicho ofrecimiento. De
igual modo, sostener, como es habitual, que la tecnología tiene como objetivo no
conocer el mundo, sino "sólo transformarlo", es realizar un recorte sumamente
arbitrario en la compleja red formada por propósitos, saberes, historia, relaciones
sociales y otras cuestiones que son inherentes al proceso tecnológico. Se hace
necesario, pues, proporcionar una caracterización de la ciencia y la tecnología más
ajustada a la diversidad y complejidad de su papel en el mundo contemporáneo, y esta
necesidad no se debe sólo a razones de índole conceptual, sino sobre todo a esclarecer
las relaciones entre la ciencia y la tecnología con la sociedad. Es obvio que ellas han
sido creadas y desarrolladas por el hombre para hacer cada vez más confortable y
satisfactoria su vida, pero este propósito natural y original parece desvirtuarse en la
actualidad cuando la ciencia o la tecnología, cada una por razones a veces muy
diferentes y otras veces semejantes, se independizan de la sociedad a la que
pertenecen y cobran un poder autónomo mediante el cual dejan de servir a la
humanidad para convertirse más bien en sus amos despóticos. Cómo y por qué ha
sucedido esto está ya insinuado en la Introducción y se profundizará en el resto del
documento. Esta cuestión del poder adquirido por la ciencia y la tecnología sobre el
resto de las fuerzas sociales, poder que en última instancia tiene una raíz política, ha
sido denunciada y analizada por muchos especialistas en cuestiones socio-políticas,
filósofos de la ciencia y de la tecnología, científicos, tecnólogos y analistas
provenientes de muchas otras áreas. En este artículo se destacará el aspecto ético de
esas relaciones. Es ésta una cuestión que requiere realizar distinciones y precisiones
porque el rótulo "los problemas sociales y éticos de la ciencia y la tecnología" admite
diversas interpretaciones y enfoques que son muy discutidas o, al menos, discutibles.
La ética y sus relaciones con la ciencia y la tecnología Antes de hablar sobre las
relaciones entre ciencia, tecnología y sociedad es preciso aclarar qué se entiende por
ética en general y especialmente en el marco de los estudios sociales de la ciencia. En
términos muy abarcadores, la ética es una disciplina filosófica que se ocupa de la
justificación racional de las normas morales que regulan la conducta humana individual
y social. Las normas morales nos dicen qué es correcto hacer y qué no lo es, tanto en
relación con nosotros mismos como en relación con los demás. Dichas normas se
expresan en juicios morales que, si se consideran en conjunto, suelen revelar aspectos
importantes de la idiosincrasia de grupos o pueblos, como podría ser el ideal de vida
buena que una comunidad comparte. En relación con la acción humana, entonces, en
dichos juicios se articulan conceptos como bueno, malo, correcto, incorrecto, justo,
injusto, deber, obligación, prohibición y otros que aluden a las creencias que tiene la
gente sobre la conducta propia y ajena y lo que está corporizado en las instituciones
sobre lo que es moralmente aceptable. El concepto vinculado con las cuestiones éticas
de la ciencia y la tecnología que más peso tiene actualmente, dada la influencia que
tienen la ciencia y la tecnología sobre el destino de las sociedades y los hombres, es el
concepto de responsabilidad. Como sucede con la gran mayoría de los conceptos
filosóficos, el significado de la noción de responsabilidad es problemático. Aquí se
tomarán en cuenta sólo aquellos sentidos que sean relevantes a las cuestiones de la
ética de la ciencia. En primer lugar, habría que distinguir un sentido causal y un
sentido normativo de responsabilidad: la falta de lluvias es causa de la sequía y, por lo
tanto, de la ruina de las cosechas, pero la falta de lluvias no es moralmente
responsable de esa catástrofe. Pero si alguien arrojó alguna sustancia perjudicial a los
cultivos, entonces ese alguien sí es responsable moral y/o legalmente de su ruina.[39]
Parece, pues, que un elemento de intencionalidad es necesario para adjudicar
responsabilidad por la consecuencia de una acción, por eso en el lenguaje del derecho
se dice que quien causa daño a otro tiene la obligación de "responder" o es
"responsable" o tiene "responsabilidad" por el daño sufrido por la otra persona. Desde
luego, es preciso realizar posteriores especificaciones respecto de las condiciones
psicofísicas que debe reunir una persona para ser considerada responsable de sus
actos, pero ésta es una cuestión que urge dilucidar más desde el sentido jurídico que
desde el moral. Es necesario además aclarar que estos dos tipos de evaluaciones del
comportamiento humano no siempre coinciden sobre las distinciones de los actos
(correctos o incorrectos) y su correspondiente noción de responsabilidad. Un ejemplo:
desde el punto de vista jurídico no es imputable mentir a un allegado por piedad o
conmiseración, pero desde un punto de vista moral puede crear un dilema, y desde la
perspectiva ético-filosófica, es posible plantearse si la regla que dice "no mentir"
admite excepciones. De acuerdo con estas consideraciones, cabría preguntarse por qué
y en qué medida se puede hablar de la responsabilidad moral de la ciencia y la técnica.
Si, como se ha dicho, los sujetos de la responsabilidad son las personas o los
individuos, son los científicos o tecnólogos quienes pueden ser imputados en ese
sentido. Pero la responsabilidad moral no es sólo atribuible a los individuos, también es
lícito plantear la existencia de la responsabilidad colectiva. Justamente, los problemas
socio-ambientales ocasionados por la ciencia y la tecnología actual han llevado a
considerar el concepto de responsabilidad más allá de los individuos que intervienen
cir-cunstancialmente en la producción de un fenómeno dado; los desastres ecológicos
producidos por derrame de petróleo, las transfusiones realizadas con sangre
contaminada con el VIH y muchos otros casos conocidos ilustran la cuestión de la
responsabilidad colectiva. Es indudable que el concepto plantea muchos interrogantes
difíciles de solucionar,[40] pero su empleo viene exigido por las peculiares situaciones
conflictivas de la vida contemporánea. Por otra parte, a raíz de los diversos perjuicios
en la salud y el medioambiente derivados de la implementación de las teorías
científicas y tecnológicas, es que ha nacido en los últimos tiempos una disciplina
filosófica denominada "ética aplicada". En principio, la ética aplicada sería, valga la
redundancia, la aplicación de la ética teórica, es decir, aquella que analiza
filosóficamente las normas morales, a casos particulares. Cómo se realiza tal
aplicación, o sea, los métodos y procedimientos, si los hay, que permitirían "bajar" al
campo de los hechos concretos los principios teóricos, es materia de ardua discusión.
Lo que interesa en este texto es mostrar cómo el uso generalizado de la ética aplicada
está señalando la íntima relación que hay entre ciencia, tecnología y ética, puesto que
son esos dos primeros fenómenos típicamente contemporáneos los que han elevado la
necesidad de apelar a la ética para resguardar los derechos a la vida de los habitantes
de la Tierra expuestos a diversos peligros por el accionar de aquellas. Precisamente, el
tema tratado en este artículo se encuadraría en una reflexión general sobre cuestiones
de ética aplicada a la ciencia y la tecnología en tanto productores de conocimiento y
las prácticas que ello implica. Existen dentro del campo de la ética aplicada otras
disciplinas que se ocupan de los problemas específicos ocasionados por la ciencia y la
tecnología en ciertos sectores del mundo natural y social. Así se puede hablar de "ética
del medioambiente", "ética médica" o "bioética", "ética aplicada al tratamiento de los
animales (y plantas)", etc. Todos estos ámbitos tienen a la investigación científica o al
conocimiento científico y tecnológico como marco de referencia, pero no cuestionan
directamente el conocimiento mismo o el accionar de la ciencia y la tecnología como
instituciones generadoras de saber, como se hace aquí, sino sobre todo a los conflictos
sociales y morales que provoca su aplicación. Es preciso aclarar, por otro lado, que
ambas perspectivas, la que considera las consecuencias de la aplicación del
conocimiento y la que se interesa por el conocimiento mismo, interaccionan en forma
continua en los estudios de ética aplicada. Otra distinción aplicable a la cuestión de la
responsabilidad moral de la ciencia y la tecnología (a las personas o al colectivo) es la
establecida entre responsabilidad retrospectiva y responsabilidad prospectiva. La
primera se refiere a los resultados de las acciones ya realizadas, por ejemplo, el haber
permitido que se arrojaran los residuos industriales en el Riachuelo, y la segunda, a la
exigencia de prever los posibles resultados de las acciones. El segundo sentido, del que
se pueden encontrar infinidad de ejemplos, tiene un interés especial pues plantea a la
ciencia y la tecnología la exigencia de conservar el planeta en buenas condiciones para
las generaciones venideras. Sin embargo, no todos acuerdan en que el "conocimiento"
científico y tecnológico pueda ser en parte responsable de los daños
medioambientales, médicos, etc. que ocurren en el mundo actual. Desde un punto de
vista por demás optimista, se suele afirmar que la ciencia y la tecnología están más allá
de posibles objeciones de naturaleza ética porque son buenas en sí mismas, lo cual
quedaría probado por las múltiples formas de progreso con que han beneficiado a la
humanidad. Efectivamente, día a día sorprenden al mundo nuevos artefactos y
procesos innovadores que salvan vidas, evitan los efectos, o al menos los atenúan, de
las catástrofes naturales, incrementan la posibilidad de la comunicación entre las
personas y, en fin, proporcionan todos los medios para lograr el bienestar material,
psicológico y espiritual que necesitan los seres humanos. Y esto es, sin duda, cierto.
Pero desde otras perspectivas se muestran los efectos adversos e irreversibles que
acompañan a los beneficios. Muchas veces las mismas técnicas y productos que
permiten salvar vidas ocasionan otros malestares y dolencias, y no pocas veces esos
malestares y dolencias terminan en daños irreversibles: la represa que proporciona la
energía eléctrica tan necesaria en un mundo superpoblado también provoca la pérdida
de valiosas especies naturales y altera el microclima de la región donde se construyó,
perturbando de ese modo el sabiamente aceptado modo de vida de las poblaciones
cercanas, al punto de causar su extinción por la ruptura del equilibrio ecológico, la
comodidad que brindan los medios de transporte modernos se ve disminuida en su
valor por la contaminación que produce, y como éstos hay gran cantidad de otros
ejemplos que muestran los efectos nocivos que en forma directa o potencial puede
acarrear la implementación de las distintas tecnologías. Un caso que ha resultado
paradigmático en años anteriores en los estudios de ciencia y tecnología es el de la
energía nuclear. En este ámbito se ve claramente el rostro dual de la ciencia y la
tecnología. Si fuera posible dominar los procesos que rigen la desintegración o fisión
del átomo de modo que su utilización fuese segura, los beneficios serían cuantiosos.
Pero la lección que dejó la catástrofe de Chernobyl[41] fue suficiente para saber que
está lejos de alcanzarse este ideal, y que los peligros que entraña la manipulación de la
energía nuclear por ahora se consideran inevitables, como también lo atestigua el
problema de los residuos nucleares. La decisión de poner en marcha una central
nuclear exige una evaluación rigurosa de los riesgos que implica. Pero, ¿es posible
realizar una evaluación tal? ¿Hay algún procedimiento establecido desde el cual se
pueda determinar con una precisión aceptable el riesgo potencial que supone el
funcionamiento de una planta nuclear? Las evaluaciones de riesgo se realizan en
general por procedimientos estadísticos, pero como la estadística opera sobre variables
preestablecidas es bueno preguntarse hasta qué punto ese método resulta totalmente
satisfactorio para los habitantes de una región donde se encuentra una planta nuclear.
La estadística nos da alguna información sobre la correlación entre ciertas variables
seleccionadas por el evaluador, pero bien analizadas las cuestiones referidas a las
evaluaciones de riesgo van mucho más allá de los porcentajes que proporciona la
estadística. Lo que la gente quiere saber para aceptar o tener cierta seguridad sobre
una determinada tecnología, tiene menos relación con números que con modos de
vida, esperanzas, temores, creencias enraizadas en la tradición y perspectivas sobre el
futuro. La consideración de estos factores por parte de quienes dirigen el aspecto
político y social de la ciencia y la tecnología significaría adoptar un modelo de
evaluación en el que se considere el riesgo, no como algo abstracto y determinable en
forma cuantitativa o técnica solamente, sino pensándolo desde una dimensión
integralmente humana. En otras palabras, este modo de encarar el control de la
ciencia y la tecnología implica asumir sobre todo una perspectiva ética y no meramente
técnica en dicho control. Silvio O. Funtowics y Jerome R. Ravetz, filósofos de la ciencia
contemporáneos, argentino e inglés respectivamente, hablan en este sentido de una
"comunidad de evaluadores extendida" que es requerida cuando la evaluación, como
en el caso de la complejidad del proyecto de construcción de una represa, "no depende
de manera esencial de la diversidad de disciplinas científicas relevantes, sino que, más
bien, consiste en la multiplicidad de perspectivas legítimas desde las que es posible
contemplar el problema en su totalidad" (1997. 156). La posibilidad de concretar este
tipo de evaluaciones se sostiene, según los autores, en lo siguiente: "En las sociedades
modernas, incluyendo tanto las ricas como las pobres, hay un gran número de gente
común que puede leer, escribir, votar y debatir. La democratización de la vida política
es algo usual hoy por hoy; sus riesgos se aceptan como un pequeño precio a pagar a
cambio de sus beneficios" (1997. 159). Estos beneficios resultan de "el logro de un
sistema que, a pesar de sus ineficacias, es el medio más eficaz de evitar desastres
originados por el prolongado silenciamiento de las críticas. Experiencias recientes han
mostrado que tal presencia crítica es tan importante para nuestros problemas
ambientales y tecnológicos como lo es para la sociedad" (1997. 159). La ética de la
investigación científica Es preciso convenir, por lo pronto, que el proceso de
producción de conocimiento científico y tecnológico involucra cuestiones éticas que
comprometen al científico en tanto ser humano que tiene intereses, porta valores
personales, incurre en parcialidades y errores y otros rasgos conductuales comunes a
todos. Algunas normas de conducta exigidas a los científicos, en tanto individuos, son
las exigibles a cualquier profesional o trabajador, como la honestidad, la veracidad,
etcétera, y otras están asociadas a la puntillosa observación de los pasos del método
científico y las condiciones de su aplicación. A raíz de este nexo entre regla
metodológica y regla moral, que sería exclusivo del ámbito de la ciencia, muchos
piensan que más que hablar de códigos de conducta o normas a las que el científico
debe ajustarse, es suficiente poner de relieve los valores que son propios de las
actividades inherentes a la investigación. Estos valores serían de diversos tipos:
cognitivos, metodológicos y prácticos o morales, y el tenerlos en cuenta y darles
cumplimiento en el proceso de investigación daría forma por sí mismo a la
responsabilidad ética del investigador. Otros, en cambio, piensan que sólo los valores o
reglas cognitivas y metodológicas son de posible consideración en la ciencia, puesto
que ésta se ocupa de conocer el mundo únicamente y no realiza acciones que apunten
a cambiar ese mundo.[42] Los valores prácticos o morales quedarían fuera de toda
consideración. Pero esta perspectiva resulta bastante simplificadora y supone la
aceptación de criterios que han sido muy objetados, como la distinción tajante entre
ciencia y tecnología. Es, pues, pertinente hablar de una ética de la investigación
científica en la que se entienda al investigador científico como una "persona" que
investiga, sujeto por lo tanto a una constelación de restricciones de orden moral que
quedaría formulada en códigos deontológicos como en cualquier otra actividad
profesional. En un documento de la National Academy of Science de Estados Unidos:
On Being A Scientist: Responsible Conduct in Research,[43] documento redactado y
supervisado por academias y comités científicos norteamericanos, se examina el
problema de la investigación científica en la actualidad y los errores que se pueden
cometer en la investigación dañando gravemente no sólo el resultado de la misma,
sino también la confianza que la sociedad puede otorgar a la ciencia. Una primera cosa
que los autores del documento aclaran es que cuando se habla del científico, no sólo
se alude a un individuo particular, sino también a una comunidad de científicos, sin la
cual la ciencia actual en su complejidad y entrecruzamientos de campos y disciplinas
no sería posible. Esta comunidad científica tiene que enfrentar en forma constante
numerosos problemas: el crecimiento de la ciencia ha sido muy grande y rápido, y esto
ha creado dificultades con el financiamiento, con los medios para alcanzar las metas y
nuevas y problemáticas relaciones con la sociedad de la que forma parte. El peso que
la ciencia y la tecnología tienen sobre la sociedad contemporánea ha despertado el
interés de los no científicos en evaluar las afirmaciones de los científicos, ya que su
bienestar hoy depende en alto grado de la actividad de los últimos. Entre las
cuestiones que atañen a la responsabilidad personal del investigador relacionadas con
el cumplimiento de las normas institucionales explícitas o implícitas, o de las que se
desprenden de las prescripciones metodológicas, se encuentran las siguientes:
Errores cometidos en el transcurso de la investigación: pueden ser "errores
honestos" que se producen inevitablemente por razones circunstan-ciales (falta de
tiempo o recursos financieros); en la medida en que sean detectados en publicaciones
o artículos, deben ser reconocidos por su autor, lo cual seguramente será considerado
con tolerancia por la comunidad científica. Otro tipo de error posible se puede deber a
la "negligencia" del investigador; el error surge por precipitación, falta de cuidado,
inatención, etc. En este caso, la reacción de los pares será más severa, y las
consecuencias que pueden tener los errores debidos a la negligencia son más difíciles
de revertir que los primeros. Pero los errores más graves son los que involucran
"engaño" por parte del científico: inventar datos o resultados o falsificarlos y el plagio
constituyen los más conocidos. Según los autores del documento On Being a Scientist,
éstos no sólo socavan el progreso de la ciencia, sino el conjunto completo de valores
sobre los cuales se sustenta la misma. Las consecuencias de tales acciones no es
asunto interno solamente, pues su corrección no se dirime exclusivamente en el
ámbito de la comunidad científica, sino también en tribunales externos.
La adjudicación de créditos o recompensas:[44] según este documento de la
National Academy of Science, en el sistema estándar de la ciencia la adjudicación del
crédito por el mérito de los trabajos realizados es reconocido en tres lugares: en la lista
de autores, en el reconocimiento por las contribuciones de otros y en la lista o
referencias de las citas. En estos lugares pueden surgir conflictos por una inadecuada
atribución del crédito o reconocimiento. Varias son las consecuencias cuando las citas
no se realizan como es debido: perjudican tanto al investigador en su prestigio y su
carrera como al sistema de publicaciones científicas. Los nombres de los autores de un
proyecto de investigación, el primero de los lugares mencionados, debe
preestablecerse al desarrollo de la investigación, para no crear conflictos posteriores.
La contribución de cada miembro suele determinarse de acuerdo con el rango que
ostenta: graduado, estudiante, etc., pero una cuestión importante es que estén
mencionados "todos los colaboradores".
El tratamiento de las técnicas experimentales: las observaciones realizadas por
los científicos a través de las técnicas experimentales estandarizadas permite la
verificación independiente de los datos. En la medida en que el científico usa esas
técnicas, los resultados de su trabajo pueden ser reproducidos por otros científicos y
esto favorece su confiabilidad. Pero tanto los métodos como el conocimiento obtenido
mediante ellos no son infalibles, deben someterse a continua revisión y control, de lo
contrario se puede caer en fatales errores. El escepticismo es por esta razón una
característica de la ciencia presente en todos los momentos de la investigación.
La publicación y la apertura: la ciencia no es una experiencia individual sino
conocimiento compartido basado en la comprensión común de algunos aspectos del
mundo físico o social. Para garantizar la confiabilidad de este conocimiento la ciencia
cuenta con ciertas convenciones sobre la forma en que deben difundirse los resultados
de la investigación. La principal es la publicación de esos resultados. Se pueden
originar conflictos sobre la prioridad en los descubrimientos, ya que el primero que
publica es el que obtiene el crédito, no el primero en descubrir algo nuevo. Quien
publica es reconocido por sus pares a través de la cita en otras publicaciones que usan
sus hallazgos, pero debe cumplir una condición básica: su trabajo tiene que haber
pasado el control de calidad efectuado por sus colegas. De lo contrario, puede caer en
errores o dar lugar a interpretaciones equivocadas.
Conflictos de intereses: el científico tiene que emplear su propio juicio para
interpretar datos, para encontrar problemas de investigación y cuándo concluir un
experimento. Para ello se necesita desarrollar ciertas habilidades que se pueden
aprender, pero no totalmente. Cuenta mucho la creatividad y la intuición, que son
condiciones personales. En estos casos los juicios empleados por los científicos
comportan valores. Algunos de estos son de naturaleza epistémica, como la
consistencia, la precisión en las predicciones, la capacidad de unificación de
observaciones diferentes, la simplicidad y la elegancia. Otros son valores personales,
religiosos, filosóficos, culturales, políticos o económicos, que forman el juicio del
científico de manera fundamental. Un punto importante desarrollado por los autores
del Documento es el de la influencia positiva o negativa de los valores en el
desempeño del científico en tanto tal. Piensan que a veces favorecen la investigación y
otras la entorpecen; un ejemplo de lo primero es el deseo de realizar una buena
investigación, así como aceptar los estándares de honestidad y objetividad propios de
la ciencia y un ejemplo de lo segundo es cuando desde el campo de la eugenesia
(aplicación de las leyes biológicas de la herencia al perfeccionamiento de la especie
humana), se usan las técnicas de la ciencia para intentar mostrar la inferioridad de
ciertas razas. Los autores creen que cuando la investigación corre peligro de distorsión
por la influencia de los valores de los científicos, los mecanismos correctivos de la
ciencia, como el control empírico de las afirmaciones y el consenso de la comunidad
científica, ponen límites a esa posibilidad. Además, la formación del científico debe ser
lo suficientemente amplia a fin de que pueda tomar conciencia de las suposiciones y
creencias que subyacen a las elecciones y decisiones que debe enfrentar en su trabajo
y así, en lo posible, neutralizarlos cuando afectan negativamente los objetivos de la
ciencia. Esa formación debe incluir el aprendizaje de cuestiones no científicas, como las
que proporcionan las ciencias sociales, la religión y la ética.
Las consideraciones que se acaban de formular se relacionan con dos cuestiones que
posiblemente sean las que más influyen en los problemas éticos de la ciencia: el
aislamiento del científico del resto de la sociedad y la percepción exclusivamente
técnica de los conflictos que suscita su actividad. Pero por sí solas, estas
consideraciones no agotan la cuestión de la responsabilidad de la ciencia, razón por la
cual deben ser ajustadas y profundizadas. Si bien la ética de la investigación tal como
la expone la National Academy of Science destaca aspectos importantes de la conducta
responsable de los científicos, es de lejos insuficiente para comprender los aspectos
éticos de la ciencia. La razón de esta insuficiencia es que mide la adecuación moral de
la conducta del científico sobre el trasfondo de "la ciencia" como una institución
autosuficiente e indiscutiblemente certera en todos sus presupuestos. Bastaría
entonces con dar al científico una formación amplia que le permita reflexionar sobre
sus inclinaciones e intereses, como recomendaba antes el documento mencionado,
para lograr un conocimiento objetivo y neutral respecto de valores éticos. Sin
embargo, en primera instancia, se puede decir que este enfoque es muy defectuoso
porque supone que tal conocimiento es posible, logrando solamente la
"deshumanización" del investigador. Si bien es cierto que es necesario despojar a la
investigación de elementos espurios, como ciertos intereses o inclinaciones, no es algo
indiscutido que todos los intereses o inclinaciones sean negativos en la labor científica.
Quienes sostienen a ultranza la idea de que el conocimiento científico es, y debe ser,
formalmente racional, objetivo y éticamente neutral, es decir, no atravesado por
intereses o valoraciones de ningún tipo, han sido denominados "cientificistas". Una de
las estrategias del anticientificista para desactivar las bases de la posición cientificista
es apelar al argumento de la naturaleza problemática del conocimiento. Se verá a
continuación cuáles son las características de estos dos enfoques, sus proponentes, los
problemas conceptuales, históricos y políticos que entrañan, así como las alternativas
de la disputa en la que están involucrados. El cientificismo, el anticientificismo y
la ética Se considera al cientificismo como un punto de vista sobre la naturaleza de la
ciencia, la tecnología y sus relaciones con el resto de la sociedad caracterizado por
considerar a la ciencia, no sólo independiente del resto de la sociedad, sino también su
rectora. La preeminencia de la misma sobre los demás ámbitos sociales se supone
derivada de su capacidad de obtener y formular un conocimiento de la realidad
absolutamente racional y objetivo. Las teorías científicas, como resultado de ese
proceso altamente preciso y seguro, proporcionarían una descripción y explicación de
los fenómenos reales totalmente cierta y confiable. En los estudios epistemológicos
sobre la ciencia, fueron las corrientes denominadas positivismo y empirismo lógico[45]
las que trasmitieron esta imagen del conocimiento científico. Sin embargo, desde otras
perspectivas epistemológicas se fueron planteando objeciones a esa imagen y sus
implicancias, de modo que el positivismo o empirismo lógico fueron perdiendo vigencia
y terminaron siendo superados por otras perspectivas que, lejos de idealizar el
conocimiento científico, mostraron sus raíces históricas y humanas. Estos cambios en
el enfoque sobre la naturaleza de la ciencia provinieron también desde otros estudios
filosóficos, desde la epistemología, tales como la historia de la ciencia y la filosofía de
la tecnología, y desde otros ámbitos no filosóficos como la sociología y la antropología.
Se puede decir que hoy día una caracterización ajustada de lo que es la ciencia no
puede prescindir de alguno de estos distintos puntos de vista. Los estudios sobre
ciencia, tecnología y sociedad, justamente, representan un enfoque integral del
fenómeno tecnocientífico.[46] Es preciso aclarar, no obstante, que si bien el positivismo
lógico ha sido reemplazado en su presentación primitiva por otras concepciones que
son opuestas en sus presupuestos fundamentales, los aportes que aquellos realizaron
al análisis de la ciencia fueron tan importantes y fecundos que constituyen la condición
necesaria de toda la reflexión posterior. El control empírico de las hipótesis y las
cuestiones lógicas que éste implica, entre muchas otras, que fueron muy elaboradas y
estudiadas por los miembros de aquellas corrientes, deben ser tenidos en cuenta en
cualquier enfoque epistemológico o metodológico que se precie de tal, aunque se
discrepe con los positivistas sobre su rol y alcance. Continuando con el tema del
cientificismo, se puede decir que uno de sus más famosos exponentes es el físico y
filósofo argentino Mario Bunge (n. 1919) que se encuentra trabajando en Canadá
desde 1966. Desde su punto de vista, la ciencia es el único modo de saber que
proporciona una comprensión fundamentada de la realidad, por eso está totalmente
justificado su papel central en la cultura contemporánea. Pero no solamente la ciencia
predomina sobre cualquier otro aspecto del sistema cultural sino que la ciencia misma
es un modelo de ética. Afirma: "La ciencia es un modelo de producción con una
modalidad ética bien precisa: no puede haber ciencia deshonesta, ciencia en búsqueda
deliberada del error, o que eluda la crítica, o que suprima la verdad. La búsqueda de la
verdad objetiva impone una recta conducta [...]" (Bunge, 1996. 54). Y más aún: "De
aquí la posibilidad de moralizar por la ciencia; esta posibilidad puede realizarse si se
adapta el código de conducta deseable en la vida diaria al código de conducta deseable
en el campo de la ciencia" (Bunge, 1996. 56). Las palabras de Bunge son
suficientemente elocuentes como para no dejar dudas sobre el modo de pensamiento
caratulado como cientificista. Profundizando las críticas y objeciones ya señaladas
recibidas por el cientificismo, se podría decir, en primer lugar, que la influencia que ha
tenido la imagen de la ciencia construida por el Positivismo y sus defensores ha sido y
es muy grande. Como sería absurdo pensar que los teóricos fundadores del
Positivismo, y otros filósofos que adhieren a esa corriente, hayan obrado de acuerdo
con oscuros propósitos, resulta claro que es en el terreno político desde donde se ha
logrado instalar la idea de autonomía y "santidad" de la ciencia11 de modo de sacar
partido de su poder potencial. Es preciso aclarar que se entiende aquí el término
"política" en sentido amplio, referido descripción de los lazos políticos, económicos y
culturales y las teorías científicas. Ver González García, M., López Cerezo, J. y Luján
López, J. (1996). 11 La expresión es de Vandana Shiva (1993). al modo en que una
sociedad y sus instituciones gestionan sus recursos, proyectos, ámbitos de poder, etc.,
y no significando las actividades partidarias del sistema político formal. Al hacer de la
ciencia no una institución más en interacción con otros sistemas sociales, sino un
santuario encerrado en una torre de marfil desde la cual sólo se emite verdades
indiscutidas, se ha impedido al resto de la sociedad participar en un diálogo con la
comunidad científica sobre la conveniencia de proseguir investigaciones propuestas o
de conservar teorías que se han mostrado perjudiciales, aun cuando las actividades de
esta comunidad ponen en riesgo sus intereses y hasta su supervivencia. Habría que
preguntarse, entonces, si el conocimiento científico contiene los caracteres que el
cientificista dice que tiene y que justificarían su posición. Si se encuentra que no los
tiene, o son imposibles de definir claramente, entonces la posición anticientificista,
cuya tesis es opuesta a la del cientificista en la medida en que considera a la ciencia
como conocimiento construido, provisorio y negociable, histórica y culturalmente
contextualizada y éticamente comprometida junto con la tecnología, encontraría
razones de peso para rechazar la posición cientificista, que, como se ha dicho,
pretende que el conocimiento científico tiene preponderancia sobre todo tipo de
conocimiento y por eso es ajena a todo requerimiento de orden social y ético. En
Ciencia, tecnología y sociedad (González García, López Cerezo y Luján López. 1996) se
ha sistematizado el conjunto de objeciones epistémicas que comúnmente se formula
contra la idea de ciencia como conocimiento univer-salmente válido, racional y objetivo
que es la que sustenta la postura cientificista; esas objeciones son las siguientes:
La fragilidad del conocimiento inductivo: remite al problema ya planteado por
David Hume (filósofo británico, 1711-1776) en el siglo XVIII, según el cual ningún
número finito de enunciados singulares puede justificar concluyentemente un
enunciado universal. Ninguna de las dos versiones de la inducción, ni la verificacionista
ni la confirmatoria, se salva de la condena de este argumento. Las observaciones son
limitadas y las enunciados de la ciencia son generales, abiertos, de modo que el
conocimiento que brindan no es totalmente racional (en el sentido de tener un
fundamento absoluto), sino sólo "razonable", dependiente más del acuerdo
intersubjetivo que de la certeza objetiva. Tampoco la refutación resulta un proceso
concluyente, aunque el esquema lógico correspondiente lo sea, pues en ciencias
fácticas las hipótesis no se ponen a prueba aisladamente sino en "red" con otras
creencias e hipótesis auxiliares, lo cual hace imposible determinar su falsedad. Tomas
Kuhn (1922-1996), sociólogo e historiador de la ciencia, asestó un golpe definitivo a la
concepción del conocimiento empírico tradicional al introducir nociones impensables
hasta ese momento en el hallazgo y evaluación de ese conocimiento: a través del
concepto de paradigma postuló que una amalgama de factores tecnológicos,
pedagógicos, psicológicos, sociológicos, considerados no epistémicos por la tradición
empirista, era la responsable de las teorías aceptadas. La historia interna de la ciencia
se muestra así insuficiente para explicar el modo en que la ciencia debe resolver sus
problemas.
Relativismos: a partir de Kuhn se desarrollan con intensidad los estudios
sociológicos de la ciencia. Con la entrada de la sociología, la frontera entre ciencia y
sociedad parece desdibujarse. La ciencia no es ya un dominio privilegiado donde se
produce el conocimiento, sino que éste surge de las mismas fuentes que alimentan
cualquier creencia común. La calificación de buena o mala ciencia ya no depende sólo
de criterios epistémicos sino también de intereses y convenciones sociales. Michael
Mulkay (n. 1936), sociólogo inglés que trabaja actualmente en la Universidad de York,
por ejemplo sostiene que la ciencia toma muchas veces del ámbito cultural externo
ciertos términos o conceptos cuando sus recursos interpretativos fracasan o que
también es receptora de las demandas de intereses de grupo, y esto le permite afirmar
que "el contenido de la ciencia se afecta por factores sociales y culturales originados
fuera de la ciencia" (Mulkay, 1994. 347). Por lo tanto, el valor cognitivo de una teoría
científica resultaría en aras de su justificación relativa a los contextos particulares, o
sea, a los criterios correspondientes a aquellos sectores externos, o bien carecería por
completo de justificación.

La carga teórica de la observación: esta cuestión alude a que lo que se ve o


percibe depende tanto de las impresiones sensibles como del conocimiento previo, las
expectativas, los prejuicios y el estado interno general del observador. Este modo de
concebir la observación complica la explicación del cambio teórico (cuando una teoría
es sustituida por otra que le es incompatible) o bien la elección entre teorías rivales
incompatibles. El concepto de paradigma de Kuhn expresa en forma radical este
problema, pues científicos que trabajan bajo paradigmas diferentes también ven el
mundo de modo diferente.[47] La cuestión de la carga teórica profundiza el problema de
la relatividad.
Infradeterminación: es un problema ocasionado por la posibilidad de establecer
teorías alternativas incompatibles para explicar un mismo conjunto de fenómenos. Otra
manera de expresar esto es afirmar que la evidencia empírica es insuficiente para
determinar la solución de un problema dado. La infradeterminación se relaciona con el
problema de la carga teórica y la inconmensurabilidad.
Clausura metodológica: a raíz de los problemas anteriormente mencionados se
produce un vacío epistémico entre la evidencia empírica y las teorías respectivas. Como
no hay recursos de ese orden para llenar esa brecha se recurre a factores no
epistémicos, que, según los autores, pueden ser"-Factores técnico-instrumentales,
como tradiciones técnicas y disponibilidades instrumentales, que canalizan el tipo de
resultados obtenidos y la interpretación de los mismos; -Factores sociales, es decir,
factores económicos, profesionales, políticos o ideológicos (el 'contexto social', en
pocas palabras), que tienen un papel crucial en la interpretación de los resultados
teóricos y la promoción selectiva de ciertas tecnologías" (González García, López
Cerezo y Luján López, 1996. 46).
De este modo, parece clara la relación del contenido de las teorías con los factores
externos considerados no epistémicos por la perspectiva tradicional. En lugar de
considerar este modo de clausura como un procedimiento espurio, se piensa que
favorece la consecución de los objetivos de la ciencia y disminuye la incertidumbre
ocasionada por la infradeterminación. Son importantes en la medida en que permiten
tomar decisiones sobre asuntos de interés general. Este conjunto de restricciones al
alcance epistémico de las afirmaciones del conocimiento científico es objeto de amplio
consenso entre los estudiosos de la ciencia y ofrece, más allá de lo que pueda implicar
desde el punto de vista estrictamente lógico o epistemológico de las teorías, una
imagen de la ciencia como una actividad humana más, lejos de la idealizada
perspectiva del cientificista. Y esto no es un rasgo secundario, externo a la ciencia
misma, sino una condición necesaria en la producción de su saber. Las cuestiones
puntualizadas más arriba están apuntando a una concepción del conocimiento
científico como algo construido, no dado, sus resultados son consensuados, no
impuestos por "la realidad"; los científicos, que son seres humanos, de algún modo
deciden con qué cosas han de tratar (ya sean teóricas o empíricas), no las
"encuentran" aquí y allí. Este modo de concebir el mundo de la ciencia no significa que
sus conceptos y objetos sean libres invenciones. Hay límites respecto de la posibilidad
de construcción del conocimiento. Como dicen los autores: "La naturaleza, realidad,
mundo externo, o como queramos llamarlo, es interpretable de diversas maneras (en
este aspecto reside el componente social del conocimiento) pero no de cualquier
manera (de aquí su componente lógico y empírico).[...] No decidimos, en un sentido
fuerte, cuales son los hechos del mundo, aunque sí asumamos o alcancemos un
consenso acerca de cómo describirlo o acerca de cómo manipularlos dados ciertos
presupuestos y ciertos fines "extracientíficos" (González García, López Cerezo y Luján
López, 1996. 51). Esto significa decirle "no" al mantenimiento de una postura
positivista ya sin fundamento, pero también "no" a la anticiencia. La idea es
desenmascarar a la ciencia en su pretensión hegemónica injustificada para acercarla a
la sociedad y ponerla a su servicio, es decir, invertir la dirección que actualmente se da
entre ambas. Ciencia básica, ciencia aplicada, técnica y tecnología En los
tópicos que hacen a la ética de la ciencia, tratados hasta el momento, no se ha hecho
ninguna alusión a las posibles diferencias entre ciencia y tecnología. Ahora es preciso
referirse al tema porque otro de los factores que favorecen el mantenimiento de una
idea de ciencia desvinculada de la sociedad y productora de conocimiento universal y
necesario, tal como la ve el cientificista, es justamente la diferenciación que se suele
establecer entre ciencia o ciencia aplicada y tecnología.[48] La distinción da pie muchas
veces a la creencia de que la ciencia es valorativa o éticamente neutral porque sólo se
ocupa de producir conocimiento, el cual es indiferente respecto a valores o normas
morales, y que es la tecnología, en tanto aplicación de ese conocimiento, la que tiene
propósitos o fines que pueden ser intrínsecamente malos. Este argumento general del
cientificista suele tener mucho peso en la sociedad por la propia consideración de los
científicos respecto de su actividad; según Stewart Richards, científico inglés
contemporáneo dedicado al estudio de la epistemología y la historia de la ciencia,
"Muchos científicos practicantes, que ignoran o desprecian la filosofía de la ciencia, se
afe-rran a una concepción idealizada de su profesión y propagan una opinión de la
'verdad científica' que implica absoluta certeza, objetividad y desprendimiento. Tal
punto de vista podría sostenerse con conocimiento total de que muchos tipos de
ciencia pueden practicarse solamente en virtud del apoyo financiero proporcionado por
los gobiernos o las compañías industriales con fines que frecuentemente son poco
claros, y casi siempre dirigidos por intereses políticos o económicos" (Richards, 1987.
172). La distinción entre ciencia y tecnología suele deberse, además, a que la gente
tiene mayormente contacto directo con técnicos o tecnólogos y no con los científicos,
quienes permanecen a salvo aislados en gabinetes o laboratorios, y es a aquellos a
quienes atribuyen toda la responsabilidad (causal y moral) por los daños y perjuicios
en el medioambiente social y natural. Es momento de preguntarse qué razonabilidad
tiene la afirmación sobre la distinción tajante entre ciencia y tecnología, como si fueran
dos empresas con propósitos cognitivos, sociales, políticos y económicos tan diferentes
que, en tanto objetos de una evaluación ética, según los cientificistas, deben ser
tratadas como ámbitos independientes. Uno de los autores internacionalmente
conocido que apoya la distinción es Mario Bunge, filósofo al que ya se ha hecho
referencia. Bunge distingue entre ciencia básica, ciencia aplicada y tecnología. Las dos
primeras tienen como objetivo la producción de conocimiento aplicando el método
científico, pero mientras la primera "trabaja en los problemas que le interesan (por
motivos puramente cognoscitivos), el investigador aplicado estudia solamente
problemas de posible interés social" (Bunge, 1997. 42). Por eso, mientras la
investigación básica no se puede planear, la aplicada sí. En la ciencia básica el
científico debe ser absolutamente libre en la elección de los temas a investigar, y en el
tiempo y los procedimientos que utiliza. Si no se entorpece este modo de trabajo del
científico básico, el mismo producirá resultados beneficiosos a la ciencia aplicada, a la
técnica y a la sociedad en forma automática. La técnica se diferencia de las dos formas
de ciencia porque mientras éstas buscan las leyes que explican la realidad, el propósito
de la primera es controlar la realidad empleando principalmente el conocimiento y el
método científico, pero también otros tipos de conocimientos. Se encarga de solucionar
problemas prácticos, no cognoscitivos. La diferencia en los resultados de la
investigación básica/aplicada y la técnica también es inequívoca: en la primera el
producto final es conocimiento y en la segunda es un artefacto o plan de acción con
valor práctico. El cuadro trazado por Bunge no deja duda acerca de la concepción del
autor sobre la ciencia, la técnica y sus relaciones. Y explica por qué sostiene la
neutralidad valorativa de la ciencia y no de la tecnología: la primera es totalmente
independiente de cualquier interés externo u objetivo que trascienda el puro
conocimiento, en cambio la segunda produce efectos reales en el mundo. Una de sus
conclusiones sobre el tema de la ética, la ciencia y la técnica, es la siguiente: "A
diferencia de la ciencia básica o pura, que es intrínsecamente valiosa o, en el peor de
los casos, carente de valor, la tecnología puede ser valiosa o disvaliosa, según sean los
fines a los cuales sirve. Por consiguiente es preciso someter la tecnología a controles
morales y sociales" (Bunge, 1996. 125).[49] En la misma vena que Bunge, Nicolas
Rescher (n. 1928), filósofo alemán radicado en EE.UU., en su obra Razón y valores en
la Era científico-tecnológica afirma que "Las ciencias de la Naturaleza y la tecnología
van juntas como piezas de un mismo cuerpo. Ninguna puede ir lejos sin la otra" (1999,
100). Esta unidad, sin embargo, se debe al sofisticado desarrollo científico-tecnológico
actual y no a la función intrínseca de cada una. En este último sentido, considera que
el cometido de la ciencia es puramente cognitivo: "la representación y racionalización
'desinteresadas' del hecho objetivo" (1999, 108). Mientras el conocimiento científico es
sólo descriptivo, el conocimiento tecnológico presenta una vertiente cognitiva y otra
normativa: la primera corresponde al "saber cómo hacer" y la segunda al "saber si"
hacer algo concreto es razonable dadas las circunstancias. El saber hacer es
específicamente técnico, pero generalmente está enraizado en el conocimiento
científico. Por ser la ciencia descripción (y explicación) de lo que es, considerarla
"antiética" respecto de los valores e intereses humanos es "irracional e inapropiado".
"Las cuestiones normativas de valor, importancia, legitimidad y similares, quedan
simplemente 'fuera del tema' en este proyecto" (1999, 111). Otros autores encuentran
más compleja la cuestión de la distinción entre ciencia y tecnología y la consiguiente
atribución de responsabilidad moral. Evandro Agazzi (1934), filósofo italiano, considera
que ciencia y tecnología conforman un fenómeno de la modernidad que no tiene
precedentes en épocas anteriores. La constitución de la tecnología, afirma, es una
consecuencia de la ciencia moderna; es esta ciencia la que proporciona los
fundamentos teóricos de la acción eficaz. Por eso la tecnología no es la forma moderna
de la técnica sino una rama de la técnica, la que se puede entender como "ciencia
aplicada". La técnica actual sigue el camino de la técnica antigua: la fabricación de
dispositivos útiles descubiertos empíricamente, sin usar, ni necesitar, conocimiento
científico. Una diferencia entre técnica y tecnología que tiene relación directa con el
tema de la responsabilidad moral es el vínculo que resulta establecido, por parte de
cada una, entre la humanidad y el mundo artificial. El surgimiento de la técnica de
algún modo produjo una bifurcación en el mundo humano al producir lo artificial, pero
esto significó una domesticación del mundo natural para satisfacer las necesidades de
la naturaleza humana; además, el crecimiento de ese mundo artificial fue lento y
fragmentario, permitiendo así la integración paulatina de los artefactos al ambiente
natural y al contexto de las condiciones humanas existentes. En el caso de la
tecnología, la naturaleza no se domestica sino más bien es sustituida por los
artefactos, y el mundo artificial crece con una rapidez, una amplitud y una complejidad
tal que le confiere las características de una autonomía creciente. Aunque la
producción de cada tecnología es local, su impacto y consecuencias, que son
imprevisibles, inmediatamente se globalizan. El modo en que cada uno de estos
procesos, el técnico y el tecnológico, de acuerdo con Agazzi, se relaciona con el tema
de la ética, se da a través de la necesidad de regulación requerida por el mundo
artificial que generan. En el caso de la técnica, se suponía que ésta se ponía al servicio
del hombre adaptando la naturaleza a sus necesidades, pero se lo hacía "obedeciendo
a la naturaleza", motivo por el cual la necesidad de regulación estaba implícita en las
"reglas del oficio". Pero en el caso de la tecnología, el mundo artificial es creado por
los seres humanos en forma independiente de la naturaleza, por eso necesita ser
regulado por los propios seres humanos en forma de normas que aseguren la armonía
y sabiduría que antes se confiaba a la naturaleza. La característica de la ciencia y la
tecnología consistente en conllevar la posibilidad de su uso incontrolado hace que las
mismas no contengan las directivas de su propio uso. El campo de la ética constituye
el campo apropiado desde donde se fijen las directivas para su ejercicio positivo. Es
preciso tener en cuenta que para Agazzi la posibilidad de distinción entre ciencia y
tecnología es a lo sumo de carácter conceptual, pero no existe entre ellas ninguna
distinción real, de ahí la pertinencia del término "tecnociencia" que refiere a un
fenómeno unitario. No obstante, para el autor, la cuestión de la responsabilidad exigida
a la tecnociencia no debe entenderse como la de un asunto externo a la propia ciencia
o como la posible existencia de un tribunal moral que juzgue sus producciones. Es al
científico individual, en tanto investigador, a quien compete el control responsable de
la tecnociencia, pues guiándose por los principios de la moralidad, puede evaluar cada
etapa de la investigación de modo que el resultado final pueda ser visto por la
humanidad como un beneficio y no como una amenaza. Sin embargo, el enfoque de
Agazzi puede ser cuestionado. La formación ética de los científicos es necesaria, y
también lo es seguir las normas establecidas para la regulación de la actividad
científica. Pero de ninguna manera es suficiente. Los peligros inherentes a la
tecnociencia, como dice el propio Agazzi, son incontrolables, y los conflictos actuales y
potenciales que genera exceden los límites personales de responsabilidad y pide el
control ético-político y social. Éste sería el modo más adecuado para que la ciencia y la
tecnología logren la confianza y la cooperación de la sociedad toda en el cumplimiento
de los objetivos de la ciencia, que no serían otros, de modo mediato, que los de la
propia sociedad. Un ejemplo de armonización de los intereses sociales y los científicos
se ha visto en el presente documento en el tema de la evaluación de riesgo de las
tecnologías. León Olivé (n. 1950), filósofo mexicano, rechaza la idea de que la
neutralidad valorativa sea una característica de la ciencia, porque tanto ella como la
tecnología están presentes en el desarrollo de una tecnología dada. Tanto en una
como en otra hay que aplicar lo que científicos y tecnólogos, así como instituciones
ecologistas, han llamado el "principio de precaución", que exhorta a suspender la
implementación de determinadas tecnologías si existen bases razonables, "aunque no
se tenga evidencia conclusiva de que existen relaciones causales" (Olivé, 2000. 66), de
que una innovación puede provocar daños en el medioambiente. A pesar de que es
difícil establecer cuáles son las bases razonables cuando intervienen actores de cuño
muy distinto como empresarios y funcionarios de Estado en la puesta en marcha de
una determinada tecnología, la única solución justa es la discusión y participación de
todos los afectados por esa tecnología, incluso la comunidad más amplia. Si parece
difícil la aplicación del principio de precaución en el campo de la tecnología por la
diversidad de actores intervinientes en las decisiones y/o evaluaciones, más difícil
resulta pensar que un científico, antes de obtener la evidencia conclusiva, sobre bases
sólo "razonables", deba, por razones éticas solamente, suspender un determinado
desarrollo. Sin embargo, como documenta Olivé, ese caso existió: Mario Molina, Premio
Nobel de Química (1995) junto a su colega Sherwood Rowland, enfrentaron lo que
Molina llamó "un problema de ética superior", a principios de la década del 70, cuando
sospecharon que los clorofluorocarburos (CFC), presentes en muchos productos
industriales (aerosoles, aire acondicionado, etc.), provocaban la destrucción de la capa
de ozono de la atmósfera terrestre. No es necesario reproducir aquí la historia de lo
sucedido; lo importante es el énfasis de Olivé sobre el hecho de que las consecuencias
dañinas de la sustancia podían ser detectadas en el "contenido" mismo de la creencia
científica. Por eso afirma tajantemente: "No es cierto que los únicos problemas morales
de la ciencia y la tecnología los constituya el uso posterior (bueno o malo) que se haga
de los conocimientos" (Olivé, 2000. 75). Reflexiones generales sobre el tema Los
beneficios que la ciencia y la tecnología proporcionan a la sociedad son cuantiosos,
pero también están rodeados de riesgos. Aquí se ha defendido la idea de que la
responsabilidad por los riesgos actuales y potenciales de la empresa tecnocientífica
abarca diversos aspectos de las actividades de investigación y también los resultados
de la misma. Científicos y tecnólogos comparten esa responsabilidad en la medida en
que ciencia y tecnología no constituyen ámbitos separados con su propia lógica, sino
son extremos de un continuo que los incluye. Antes de la explosión tecnológica del
siglo XX, la ciencia guardaba cierta distancia del resto de la sociedad, pero con el
surgimiento de tecnologías de uso cotidiano, la gente comienza a tomar conciencia de
la participación en los riesgos que tiene la ciencia. El cuestionamiento por los
"descubrimientos" científicos se fue extendiendo y llevó a que muchos sectores sociales
tomaran cartas en el asunto. Los comités de ética en hospitales y centros de salud, los
acuerdos internacionales sobre el cuidado de los recursos disponibles en el planeta,
como el protocolo de Kyoto de 1997 y la recientemente finalizada Cumbre de
Copenhague sobre la reducción de gases que causan el efecto invernadero, y otros
acuerdos regionales sobre el uso cauteloso de los bienes naturales compartidos, son
ejemplos de la búsqueda de límites a las actividades de la ciencia y la tecnología. Aun
así los controles políticos y sociales son muy resistidos, y lo son en nombre de una
ciencia que muchas veces se autocalifica como autónoma y reclama independencia
completa de los demás sectores sociales. Ya se vio que algunas posiciones filosóficas
sobre la ciencia respaldan tal pretensión argumentando que la ciencia sólo produce
conocimiento objetivo universal-mente válido, que por eso mismo es neutralmente
valorativo, no pudiéndosele imputar ningún tipo de responsabilidad moral. Pero esto no
es otra cosa que el enfoque que hemos llamado "cientificista" que es cerrado y
determinante respecto del papel jerárquico y hegemónico que otorga a la ciencia y los
valores que porta sobre otros ámbitos de interés social. Se ha tratado de mostrar aquí
la insuficiencia de esta perspectiva, ya sea desde el punto de vista epistemológico,
ético o social. El conocimiento científico no tiene la precisión ni la objetividad que se
creía al comienzo, tampoco está libre de las influencias externas en la elección de
problemas y la justificación de las soluciones. Además, como hija de su tiempo, puede
estar sujeta a la dirección que le marcan las fuentes de financiación o los fines
políticos. En el terreno de los efectos y resultados de las investigaciones, la falta de
responsabilidad ha sido y es frecuente: la sofisticación de los armamentos es cada vez
mayor, los experimentos con personas no informadas en lugares del Tercer Mundo aún
ocurren, siempre aparecen sustancias nuevas que son contaminantes, etc. La solución
propuesta consiste en que los proyectos de investigación y de desarrollo tecnológico
sean puestos a consideración de los interesados o afectados por ellos. A muchos puede
parecerles al menos utópica este tipo de sugerencia, pero no hay soluciones
indiscutibles, salvo en lo que hacen al mantenimiento del respeto y dignidad de las
personas. Además, una ciencia no contaminada por los intereses económicos, políticos
y sociales no deja tampoco de ser una idea utópica. La dimensión ética la atraviesa,
como lo hace con todo quehacer humano. Es preciso aclarar que al tratar el tema de
ética de la ciencia y sus aplicaciones derivadas, que es objeto de enfoques
controvertidos e interpretaciones contrapuestas, se incurre en simplificaciones. Algunas
veces ocurre involuntariamente, pero la mayoría de las veces es necesario realizar
recortes con el fin de lograr concretar algunas ideas, aunque con ello se cercene la
profundidad que tiene este tema. Queda pues, como tarea siempre pendiente, rescatar
la riqueza y complejidad que le es inherente al estudio de un fenómeno típicamente
humano como es el de la ciencia y su relación con la ética.

Bibliografía Agazzi, E. (1997). Equivalence or Separation Betwen Science and Technology? En From
Technique to Technology: The Role of Modern Science. Society for Philosophy & Technology. Vol. 4, n° 2.
Ávila, H. (2002). Ciencia y ética. En Temas de pensamiento científico. Buenos Aires: Eudeba. Bonilla, A.
(2003). La ética de la responsabilidad de Hans Jonas en su discusión contemporánea. (Conferencia sin
editar). Pontificia Universidade Catolica Do Rio Grande Do Sul. Brasil. Bunge, M. (1996). Ética, Ciencia y
técnica. Buenos Aires: Sudamericana. Bunge, M. (1997). Ciencia, técnica y Desarrollo. Buenos Aires:
Sudamericana. Ciapuscio, H. (1994). El fuego de Prometeo. Buenos Aires: Eudeba. Echeverría, J. (1998).
Filosofía de la ciencia. Madrid: Akal. Funtowicz, S. y Ravetz, R. J. (1997). Problemas ambientales, ciencia
postnormal y comunidades de evaluadores extendidas. En González García, M. I., López Cerezo, J. A. y
Luján López, J. L. Ciencia, tecnología y sociedad. Buenos Aires: Ariel. González García, M. I., López
Cerezo, J. A. y Luján López, J. L. (1996). Ciencia, tecnología y sociedad. Madrid: Tecnos. Guariglia, O.
(1996). Moralidad. Ética universalista y sujeto moral. Buenos Aires: Fondo de Cultura Económica. Laudan,
L. (1984). Science and Values. Berkeley: Univ. of California Press. Mulkay, M. (1994). La ciencia y el
contexto social. En La explicación social del conocimiento. México: UNAM. National Academy of Sciences.
(1995). On Being a Scientist: Responsible Conduct in Research. Washington DC. Olivé, L. (2000). Ciencia y
tecnología. En Los linderos de la ética. Madrid: Siglo Veintiuno. Rescher, N. (1999). Razón y valores en la
Era científico-tecnológica. Barcelona: Paidós. Richards, S. (1987). Filosofía y sociología de la ciencia.
México: Siglo Veintiuno. Shiva, V. (1993). Colonialism and the Evolution of Masculinist Forestry. En
Harding, S. (ed.). The "Racial" Economy of Science. Indiana University Press.

Notas
[1]
1 Samaja, J. (1993). Epistemología y Metodología (329). Buenos Aires: Eudeba. [2]2. Recomendamos
para el conocimiento de la obra de Lewis Carroll la edición de: Eduardo Stilman (comp. y trad.). (1998).
Lewis Carroll. Los libros de Alicia. Buenos Aires: Ediciones de la Flor y Best Ediciones, con prólogo de Jorge
Luis Borges. En la misma, se encuentra la edición crítica de las Aventuras de Alicia en el país de las
maravillas, A través del espejo y lo que Alicia encontró allí, La avispa con peluca, La caza del Snark y una
cantidad importante de cartas y fotografías. [3]3. Las citas de la obra de Lewis Carroll, Alicia en el país de
las maravillas, corresponden a la edición de Alianza, Buenos Aires, 1990. [4]4. En 1856, a pedido del editor
de sus primeros poemas, Charles Dodgson debe elegir un seudónimo por lo que propone varios: 1) Edgar
Cuthwellis, por transposición de Charles Lutwidge, 2) Edgar U. C. Westhill, 3) Louis Carroll, derivado de
Lutwidge=Ludovic y Charles=Carolus, 4) Lewis Carroll. El editor escogió la última opción. En adelante hará
una tajante separación entre las obras publicadas por Dodgson, las publicadas por Carroll y otras
totalmente anónimas. [5]1 Eco, U. (1993). El nombre de la rosa. Barcelona: RBA Editores. [6]2 Bertelloni, F.
(1997). Para leer El nombre de la rosa de Umberto Eco. Sus temas históricos, filosóficos y políticos.
Buenos Aires: Oficina de publicaciones del C.B.C. [7]3 Eco, U. (1993, 198). [8]4 Borges, J. L. (1969). El
Golem. En El otro, el mismo. Buenos Aires: Emecé. [9]5 Morris, C. (1985). Fundamentos de la teoría de los
signos. Barcelona: Paidós. [10]1 Pitcher, G. (1964). The Philosophy of Wittgenstein. Englewood Cliffs:
Prentice-Hall. [11]2 Devereux, G. y Weiner, F. (1950). The Ocupation Status of Nurse. En American
Sociological Review. N° 5 (vol. 15). [12]3 Weininger, O. Sexo y Carácter (286). Buenos Aires: Losada. [13]4
Tapia, A. El árbol de la retórica. [en línea]. [consulta: 26 de julio de 2010].
<http://elarboldelaretorica.blogspot.com/2007/02/mapas-argumentativos.html> [14]1 Mosterín, J. (2000).
Conceptos y teorías en la ciencia (208-209). Madrid: Alianza. [15]2 El acertijo "MU" extraído de Hofstadter,
D. R. (1979). Gódel, Escher, Bach: an Eternal Goleen Braid (34-41 y 260-261). N.Y.C.: Basic Books. [16]3
Borges, J. L. (1979). El libro de arena. En El libro de arena. Madrid: Alianza-Emecé. [17]4 Martínez, G.
(2001). Borges y la matemática. Buenos Aires: Eudeba. [18]5 Martínez, G. y Piñeiro, G. (2009). Gódel para
todos. Buenos Aires: Seix Barral. [19]1 Peri Rossi, C. (1999). El amor es una droga dura. Barcelona: Seix
Barral. [20]2 "La Esfericidad de la Tierra 2: De la Antigüedad a Colón". (2009, enero 6). Odisea cósmica.
Blog de actualización astronómica y espacial. [en línea]. [consulta: 10 de junio de 2010]
<http://www.odiseacosmica.com> [21]1 Elster, J. (2000). El cambio tecnológico. Barcelona: Gedisa. [22]2
Elster, J. (1991). Tuercas y tornillos (67-68). Barcelona: Gedisa. [23]1 Se entiende por monismo
metodológico a la corriente epistemológica que sostiene que existe un único método aplicable a las
ciencias tanto naturales como sociales, afirmando de esta manera el monopolio del método
hipotético-deductivo, o del dialéctico. En este sentido, toda posición monista es reduccionista, ya sea que
reduzca la cuestión del método al hipotético-deductivo, o ya sea que la reduzca al dialéctico. En este
sentido, a ambos tipos de ciencias fácticas, o bien se aplica el método hipotético-deductivo, o bien se
aplica el dialéctico, reduciendo ulteriormente toda otra posibilidad. (Asti Vera y Ambrosini, 2009.189)
[24]
2 Se entiende por pluralismo metodológico a aquella corriente epistemológica que sostiene que para
cada tipo de ciencias fácticas, es decir, para las naturales y para las sociales, existe un método propio de
justificación de hipótesis o teorías para cada una de ellas; en el primer caso, podrá ser el inductivista o el
hipotético-deductivo; en el segundo, será el intencional, el genético, etc. (Ibídem) [25]3 Tradicionalmente
se denominan como "filósofos analíticos" a aquellos cuya actividad principal consiste en argumentar a
favor o en contra de distintas posiciones filosóficas, incluso pensando a la argumentación como la principal
actividad de los filósofos. Este tipo de actitud filosófica estaba encarnada en los filósofos sociales -como
Dewey-, los matemáticos o los científicos naturales -como Carnal y Reichenbach. Según Rorty,
actualmente el panorama ha cambiado, y esta actitud la detentan los abogados, y lo que definiría
entonces a la filosofía analítica hoy sería un "estilo", una "aptitud filosófica" consistente en ser capaz de
discernir en cualquier argumento "lagunas lógicas", y de producir un buen argumento en favor de
cualquier posición. Frente a esta corriente filosófica encontramos a los llamados "filósofos continentales",
como Heidegger, Nietzsche, Derrida, Foucault, Gadamer, etc., cuyo objetivo, sostiene Rorty, consiste en la
búsqueda de una reconstrucción histórica exhaustiva, de una redescripción del pasado adecuada para
distintos fines y, en este sentido, Rorty los llama "críticos de la cultura". (Comesaña, 1998. 114-115) [26]4
Asti Vera, C. y Ambrosini, C. (2009. 218-220). [27]5 Compárese con el mismo concepto y el ejemplo ya
citado en este Documento. [28]6 Phillip Melanchthon, nacido como Phillip Schwartzerd, fue un reformista
germano, colaborador de Martín Lutero, y principal jefe del luteranismo tras la muerte de su fundador. Fue
uno de los líderes de la Reforma Luterana, aunque su intención era lograr un entendimiento entre
protestantes y católicos. Su obra y lecciones han tenido una gran influencia en la educación, tal es así que
se convirtió en el autor más leído de su tiempo, y algunas de sus obras fueron textos de estudio en las
escuelas hasta 1800. [29]1 Biblioteca Escéptica. [en línea]. [consulta: 26 de julio de 2010].
<http://bibliotecaesceptica.wordpress.com/2009/05/06/%C2%BFque-son-laspseudociencias-
mario-bunge/#more-1743>
[30]
2 Kuhn, T. (1986). La estructura de las revoluciones científicas. México: Fondo de Cultura Económica.
[31]
3 Edmonds, D. J. y Eidinow, J. A. (2001). El atizador de Wittgenstein. Una jugada incompleta (222).
Barcelona: Península. [32]4 Semmelweis, I. (1988). Etiología, concepto y profilaxis de la fiebre puerperal.
En El desafío de la epidemiología, problemas y lecturas seleccionadas. Organización Panamericana de la
Salud. [33]1 Para deslindar los distintos aspectos de la planificación de la política científica, nos basaremos
sobre todo en el capítulo mencionado de E. P. Yúfera. [34]2 Florentino Ameghino (1854-1911), científico
argentino fundador de la paleontología sudamericana a fines del siglo XIX. [35]3 El adjetivo "malthusiana"
alude a la predicción de Malthus, T. R. (1766-1834), economista y demógrafo británico. Según esa
predicción, puesto que la población tiende a crecer en progresión geométrica y los alimentos en
progresión aritmética, llegará un día en que la población superará los medios de subsistencia de no mediar
obstáculos preventivos y represivos. Si se entiende el factor "población" como población de investigadores
y el factor "medios de subsistencia" en calidad de recursos financieros, la predicción resultante sería que si
el aumento de investigadores crece geométricamente y los recursos financieros lo hacen en proporción
aritmética, la cantidad de investigadores superará el volumen de los recursos. Pero el caso ya mencionado
de Venezuela contaría como un contraejemplo de esa predicción. [36]4 El destacado es del autor.
[37]
1 Bacon, F. (1961). Novum Organum, Buenos Aires: Losada. [38]2 Esta referencia a las fuentes diversas
de las cuales surge la teoría y la tecnología de la información puede servir como ejemplo de lo que se
sostendrá más adelante sobre la construcción del conocimiento, tanto el científico como el técnico. [39]3
Los distintos aspectos del significado del concepto de responsabilidad señalados en este artículo son
tratados por Bonilla, A. (2003). [40]4 El concepto de responsabilidad colectiva genera muchos
cuestionamientos, por ejemplo: ¿cómo se puede adjudicar responsabilidad a un grupo por daños que
provocaron unos pocos, sin violar los principios de la libertad individual?, ¿se puede atribuir
intencionalidad al grupo, considerado distinto de sus miembros, para atribuirle responsabilidad?, etc. [41]5
La explosión ocurrió en un reactor de la planta nuclear de Chernobyl, ex Unión Soviética, el 26 de abril de
1986. [42]6 Larry Laudan, por ejemplo, afirma que la filosofía de la ciencia sólo se puede ocupar de valores
epistémicos o cognitivos, pero no debe preguntarse por el deber ser de los científicos. [43]7 Washington
DC: National Academy of Science, 1995. [44]8 Generalmente se cree que la recompensa de la que gozan
los científicos es de naturaleza intelectual u honorífica, en tanto la del tecnólogo es más bien económica,
pues los inventos tienen un valor comercial del que carece la producción científica. [45]9 El origen del
Positivismo lógico se remonta a la constitución del Círculo de Viena en la Universidad de Viena en 1922.
Sus representantes principales son L. Wittgenstein y M. Schlick. Sostienen una forma de empirismo
estricto: las proposiciones científicas pueden ser verificadas en forma completa por la experiencia. El
Empirismo lógico que continúa esta corriente, representado principalmente por R. Carnap, C. Hempel y H.
Reichembach, encarna un empirismo más moderado: la experiencia sólo puede mostrar que las
proposiciones científicas son probables. [46]10 A partir de la obra de Kuhn (1962) surge una profusión de
trabajos sobre la ciencia señalando especialmente su relación con la sociedad actual; constituyen un
conjunto heterogéneo, pero suelen agruparse bajo la sigla CTS (Ciencia, Tecnología y Sociedad). Se ha
distinguido entre la tradición europea de Science and Technology Studies y la tradición americana: STS
(Science, Technology and Society). Aunque comparten presupuestos generales sobre la dimensión social
de la ciencia y la tecnología, la primera enfatiza el origen de las teorías científicas, es decir, en la ciencia
como proceso. La segunda se centra en la descripción de los lazos políticos, económicos y culturales y las
teorías científicas. Ver González García, M., López Cerezo, J. y Luján López, J. (1996). 11 La expresión es
de Vandana Shiva (1993). [47]12 La tesis de la incomensurabilidad entre paradigmas y el relativismo que
implica fue radical en La estructura de las revoluciones científicas (Kuhn, 1962), pero luego, ya en la
Posdata de La estructura... y en las obras posteriores, fue debilitando ese relativismo haciendo hincapié en
la inconmensurabilidad lingüística. [48]13 Ciapuscio (1994) desarrolla un panorama completo de las teorías
sobre las diferencias entre ciencia y tecnología. [49]14 Es preciso aclarar que para Bunge, el científico sí es
moralmente responsable pues se puede prestar a cualquier tipo de conducta corrupta o a alterar el
correcto proceso de investigación cediendo a presiones externas. Ver Bunge (1996), capítulo V.

También podría gustarte